Location via proxy:   [ UP ]  
[Report a bug]   [Manage cookies]                

Courseware Module 4 VA

Download as pdf or txt
Download as pdf or txt
You are on page 1of 134

CATapult Courseware

Module 4
Verbal Ability
Published by IMS Learning Resources Pvt. Ltd. in the Year 2020

Registered Office: 6th Floor, NCL Building, ‘E’ Block, Near Bandra Family Court,
Bandra Kurla Complex (BKC), Bandra (E), Mumbai - 400051
Tel.: +91 22 66170000
Toll Free: 1800-1234-467
CIN : U80220MH1999PTC121823
E-mail : support@imsindia.com Website: www.imsindia.com

Copyright © IMS Learning Resources Pvt. Ltd.

All copyrights to this material vests with IMS Learning Resources Pvt. Ltd. No part of this material
either in part or as a whole shall be copied, reprinted, reproduced, sold, distributed or transmitted
in any form or by any means, electronic, mechanical, photocopying, recording or otherwise, or
stored in any retrieval system of any nature without the permission of IMS Learning Resources
Pvt. Ltd., and any such violation would entail initiation of suitable legal proceedings.

The views of and opinions expressed in this book are not necessarily those of the publishers. While
every effort has been made to ensure that all facts are stated correctly, the publishers regret their
inability to accept responsibility for any inadvertent errors or inaccuracies. Readers are advised in
their own interest to reconfirm facts before acting upon them.

The publishers shall endeavour, wherever possible to remedy all errors of commission and omission
which are brought to their attention in subsequent editions.

This book is sold subject to the condition that it shall not, but way of trader or otherwise, be
lent, resold, hired out, or otherwise circulated without the publisher’s prior written consent in any
form of binding or cover other than that in which it is published and without a similar condition
including this condition being imposed on the subsequent purchaser and without limiting the rights
under copyright reserved above.
CATapult
VERBAL REASONING

VA-4.1 SYLLOGISMS
THEORY

Standard Form of Syllogisms


Introduction

In which of the three sets A, B and C, do the first two statements logically lead to the third
statement?
A B C
All planets revolve around All planets revolve around
Earth is a planet.
the Sun. the Sun.
The Earth revolves The Earth revolves around
Earth is a planet.
around the Sun. the Sun.
All planets revolve The Earth revolves around
Earth is a planet.
around the Sun. the Sun.

SET A:
The third statement in Set A — ‘All planets revolve around the Sun’ — does not follow from the
first two. One cannot deduce that ‘all’ planets revolve around the Sun merely based on the fact
that planet Earth revolves around the Sun.
Set B:
In Set B, the third statement does not follow from the first two. It is not explicitly stated that
‘only’ planets revolve around the Sun. Other objects may also revolve around the Sun.
SET C:
Only in Set C does the third statement follow logically from the first two statements: All planets
revolve around the Sun, Earth is a planet, and hence the Earth revolves around the Sun.
An Aristotelian syllogism is a set of three statements, like Set C, where one statement (Conclusion)
is logically derived from the other two statements (Premises).
For example:
Premise: All men are mortal.
Premise: Socrates is a man.
Conclusion: Socrates is mortal.

1
CATapult
VERBAL REASONING

Each of the premises can be in one of the four standard forms:

NAME FORM SYMBOL EXAMPLE


Universal Affirmatives All A are B A All Men are Mortal
Universal Negatives No A are B E No Man is Perfect
Particular Affirmatives Some A are B I Some Men are Wise
Particular Negatives Some A are not B O Some Men are not Clever

In the above forms A and B are called categorical terms. In each of the forms listed above, A
is the subject (S) and B is the predicate (P). The predicate of the Conclusion is called the major
term (P); the subject of the conclusion is called the minor term (S). In the example above, Mortal
is the major term and Socrates is the minor term.
The premise containing the major term is called the major premise and premise containing the
minor term is called the minor premise.
In syllogism of three statements there will be a third term that links the major and the minor
term, the middle term (M). In the example above, Man is the middle term.
Each of the premises has one term in common with the conclusion: in a major premise, this is
the major term (i.e., the predicate of the conclusion); in a minor premise, it is the minor term (the
subject) of the conclusion. For example:

Major premise: All men are mortal.


Minor premise: Socrates is a man.
Conclusion: Socrates is mortal.
As seen from the above example, the middle term always appears twice in the premises.

Using Rules and Fallacies to Test the Validity of Syllogisms

Aristotle first expressed the idea that valid syllogisms conform to certain rules. There are 5 general
rules that can be used to test the validity of syllogisms. If any one of these rules is violated, a
specific formal fallacy is committed and, accordingly, the syllogism is invalid. Conversely, if none
of the rules are broken, the syllogism is valid.

Of the five rules presented in this section, the first two depend on the concept of distribution, the
last three on the combination of different forms of propositions.

A categorical term is said to be distributed if all individual members of that category are accounted
for.

In a statement like ‘All A are B’ the term A is distributed, because all elements of the set A are
pinpointed (All of them are B). The term B is not distributed because there might be Bs that are
not A and about which we have no information.

In a statement like ‘Some C are D’, neither C nor D are distributed because nothing is said about
the remaining Cs (that are not D) and nothing about the Ds that are not C.

2
Chapter VA
SYLLOGISMS 4.1

In a categorical syllogism the distribution of terms depends on the subject:

THEORY
 In A: All A are B propositions, the subject (A) is distributed.
 In E: No A is B propositions, both the subject (A) and the predicate (B) are distributed.

PRACTICE EXERCISE CLASS EXERCISE


 In I: Some A are B propositions, neither the subject nor the predicate are distributed.
 In O: Some A are not B propositions, the predicate (B) is distributed (all points of B are
outside the points of A which are not B)

Rule 1: The middle term must be distributed at least once.


Fallacy: Undistributed middle.

Examples
Premise: All eagles are birds.
Premise: All doves are birds
Conclusion: All doves are eagles
In this standard-form categorical syllogism the middle term is ‘birds’. In both premises ‘birds’ occurs
as the predicate of an A type proposition and therefore it is not distributed in either premise.
Thus, the syllogism commits the fallacy of undistributed middle and is invalid. If the major premise
were rewritten to read ‘All birds are eagles’, then ‘birds’ would be distributed in that premise and
the syllogism would be valid.

Rule 2: If a term is distributed in the conclusion, then it must be distributed in a premise.


Fallacies: Illicit major/minor.
Examples
1. All donkeys are animals.
Some foxes are not donkeys.
Some foxes are not animals.

2. All lions are mammals.


All mammals are animals.
All animals are lions.
In the first example the major term, ‘animals’, is distributed in the conclusion but not in the major
premise, so the syllogism commits the fallacy of illicit major. In the second example the minor
term, ‘animals’, is distributed in the conclusion but not in the minor premise. The second example
therefore commits the fallacy of illicit minor, or ‘illicit process of the minor term’.

In applying this rule, one must always examine the conclusion first. If no terms are distributed in
the conclusion, Rule 2 cannot be violated. If one or both terms in the conclusion are distributed,
then the appropriate premise must be examined. If the term distributed in the conclusion is also
distributed in the premise, then the rule is not violated. But, if the term is not distributed in the
premise, the rule is violated and the syllogism is invalid. In applying Rule 2 (and also Rule 1),
you may find it helpful to begin by marking all the distributed terms in the syllogism—either by
circling them or by labeling them with a small letter ‘d’.

3
CATapult
VERBAL REASONING

Rule 2 becomes intuitively plausible when it is recognized that distribution is a positive attribute.
Granting this, an argument that has a term distributed in the conclusion but not in the premises
has more in the conclusion than it does in the premises and is therefore invalid. Of course, it is
always permissible to have more in a premise than appears in the conclusion, so it is perfectly
all right for a term to be distributed in a premise but not in the conclusion.

Rule 3: Two negative premises are not allowed.


Fallacy: Exclusive premises.
Example
No birds are mammals.
Some wolves are not birds.
Some wolves are not mammals.
This syllogism may be seen to be invalid because it has true premises and a false conclusion. The
defect is attributable to the fact that it has two negative premises. Upon reflection, Rule 3 should
be fairly obvious. Let ‘S’, ‘P’ and ‘M’ once again designate the minor, major, and middle terms.
Now, if the P class and the M class are separate either wholly or partially, and the S class and
the M class are separate either wholly or partially, nothing is said about the relation between the
S class and the P class. These two classes may either be distinct or identical in whole or in part.

Rule 4: A negative premise requires a negative conclusion, and a negative conclusion requires a
negative premise.
Fallacy: Drawing an affirmative conclusion from a negative premise.
OR
Drawing a negative conclusion from affirmative premises.
These arguments may be seen to be invalid because each has true premises and a false conclusion.
The first draws an affirmative conclusion from a negative premise, and the second draws a negative
conclusion from affirmative premises.

An alternate formulation of Rule 4 is: Any syllogism having exactly one negative statement is invalid.
Thus, if the conclusion alone is negative, or if one premise is negative while the other premise
and the conclusion are affirmative, the syllogism is invalid.

Conversely, a negative conclusion asserts that the S class is separate either wholly or partially from
the P class. But if both premises are affirmative, they assert class inclusion rather than separation.
Thus, a negative conclusion cannot be drawn from affirmative premises.

Rule 5: No valid syllogism can have two particular premises.

As a result of the interaction of these first four rules, it turns out that no valid syllogism can have
two particular premises. This result is convenient to keep in mind, because it allows us to identify
as invalid any standard-form syllogism in which both premises start with ‘Some’.

4
Chapter VA
SYLLOGISMS 4.1

THEORY
Summary of Rules

Rule 1:
The middle term must be distributed in one of the premises.

PRACTICE EXERCISE CLASS EXERCISE


Rule 2:
If a term is distributed in the conclusion it must be distributed in one of the premises.
Rule 3:
Two negative premises (No + No, No + Some Not, Some Not + Some Not) are not allowed.
Rule 4:
A negative premise requires a negative conclusion, and a negative conclusion requires
a negative premise (No or Some Not in the premise must be followed by No or Some
Not in conclusion and vice-versa)
Rule 5: No valid syllogism can have two particular premises (Some + Some, Some + Some Not).

Note
 There is no fixed order in which the rules have to be applied.
 The order depends on the form of the premises and conclusions. In case the premises or
conclusion are in the form of Some A is not B or No A is B, Rule 4 should be applied first.
 The only rules that need to be applied are 1 and 2, violation of the rest of the rules can be
seen just by observation.

While syllogisms can be solved equally accurately using Venn-diagrams, a proper understanding of
the rules will prove to be a faster method.

Please remember that syllogisms need not be literally true - i.e. statements like ‘all black is white’
or ‘no fish can swim’ are perfectly valid as premises or conclusions, as long as they follow the
rules listed above.

5
CATapult
VERBAL REASONING

Venn Diagrams

The validity of the conclusion depends on the form of the premises. One of the best ways of testing
the validity of syllogisms is by representing the form of the premises using Venn diagrams. The
following is the representation of the four standard forms of the propositions using Venn-Diagrams.

Each type of premise implies more than one possibility. The Venn diagrams shown below represent
all possibilities arising from a particular proposition. The valid conclusions are those that will apply
to all the possibilities.

Men = M & Liars = L VALID CONCLUSION INVALID CONCLUSION


All M are L Some Liars are Men All Liars are Men
All Liars may be Men Some Liars are not Men
L L
Some Liars may not be Men

M M

Some L are M Some Liars are Men Some Liars are not Men
L M L, M All Liars may be Men All Liars are Men
Some Men may not be Liars
All Men may be Liars

L
M
L
M

Some L are not M Some Men may not be Liars Some Men are Liars
L M L All Men may be Liars Some Liars are Men
M No Men may be Liars
No Liars may be Men

L M

6
Chapter VA
SYLLOGISMS 4.1

No L are M No Men are Liars

THEORY
M L

PRACTICE EXERCISE CLASS EXERCISE


Difference Between Some and All

The difference between the quantifiers Some and All as used in logic as opposed to ordinary
language needs to be clearly understood. In logic, Some A is B does not mean Only some A is B,
it represents the fact that there is information only about some parts of A and that they are in B;
the rest of the A might or might not be in A and nothing can be said with certainty about them.

7
CATapult
VERBAL REASONING

CLASS EXERCISE

Directions: Use the five rules to determine in which of the following sets of statements the third
statement logically follows from the first two statements.

1. All wines are expensive.


All expensive things are exclusive.
Some exclusive things are not wines.

2. No commercial movies are art-movies.


No art-movies are big-budget movies.
No big-budget movies are commercial movies.

3. All popular novels are bestsellers.


Some literary novels are not best sellers.
Some literary novels are not popular.

4. All high-calories diets are diets high in cholesterol.


Some diets high in cholesterol are not healthy diets.
Some healthy diets are not high-calorie diets.

5. All plants have chlorophyll.


All plants have leaves.
Some leaves have chlorophyll.

6. Some convicted are not offenders.


All offenders are guilty.
Some guilty are not convicted.

7. All companies that overprice their products are unethical.


Some unethical businesses are owned by investors.
Some companies owned by investors overprice their products.

8. All dinosaurs are extinct.


All extinct animals lived before the ice age.
Some of the animals living before the ice age were dinosaurs.

9. No wire-tapping of phones based on suspicion is unlawful.


Some wire-tapping of phones is unlawful.
Some wire-tapping of phones is not based on suspicion.

8
Chapter VA
SYLLOGISMS 4.1

10. All persuaders are masters at eye contact.

THEORY
All masters at eye contact are confident.
Some confident people are persuaders.

PRACTICE EXERCISE CLASS EXERCISE


Directions: Represent the four statements given below using Venn diagrams.

11. All men are liars.

12. Some men are liars.

13. Some men are not liars.

14. No men are liars.

Using rules and fallacies to test the validity of syllogisms:

15. Rule 1:
Fallacy:
Example:
P1: All eagles are birds. P2: All doves are birds. C: All doves are eagles.

16. Rule 2:
Fallacies:
Example 1:
P1: All dogs are animals. P2: Some foxes are not dogs.
C: Some foxes are not animals.
Example 2:
P1: All lions are mammals. P2: All mammals are animals.
C: All animals are lions.

17. Rule 3:
Fallacy:
Example:
P1: No birds are mammals. P2: Some wolves are not birds.
C: Some wolves are not mammals.

18. Rule 4:
Fallacy:

19. Rule 5:

9
CATapult
VERBAL REASONING

Directions: In each question below, three statements (I, II and III) are followed by four conclusions
(A, B, C and D). You have to take the given statements to be true even if they seem to be at variance
with commonly known facts. Read all the conclusions and then decide which logically follow/s from
the given statements, disregarding commonly known facts.

20. Statements:
I. Some drivers are technicians.
II. All technicians are engineers.
III. Some engineers are lecturers.
Conclusions:
A. Some technicians are lecturers. B. Some lecturers are drivers.
C. All engineers are technicians. D. Some engineers are drivers.
1) Only C follows. 2) Only D follows.
3) Only C and D follow. 4) None of these

Directions: This question contains six statements followed by four sets of combinations of three
statements. Choose the set in which the statements are logically related. (Past CAT Questions)

21. A. All roses are fragrant. B. All roses are majestic.


C. All roses are plants. D. All roses need air.
E. All plants need air. F. All plants need water.
1) CED 2) ACB 3) BDC 4) CFE

22. A. All men are men of scientific ability.


B. Some women are women of scientific ability.
C. Some men are men of artistic genius.
D. Some men and women are people of scientific ability.
E. Some men of artistic genius are men of scientific ability.
F. Some women of artistic genius are women of scientific ability.
1) ACD 2) ACE 3) DEF 4) ABC

23. A. No fishes breathe through lungs. B. All fishes have scales.


C. Some fishes breed up-stream. D. All whales breathe through lungs.
E. No whales are fishes. F. All whales are mammals.
1) ABC 2) BCD 3) ADE 4) DEF

24. A. All those who achieve great ends are happy.


B. All young people are happy.
C. All young people achieve great ends.
D. No young people achieve great ends.
E. No young people are happy,
F. Some young people are happy.
1) ADE 2) ABF 3) ACB 4) ADF

10
Chapter VA
SYLLOGISMS 4.1

25. A. Some of my closest friends disapprove of me.

THEORY
B. Some of my closest friends are aardvarks.
C. All of my closest friends disapprove of me.
D. All who disapprove of me are aardvarks.

PRACTICE EXERCISE CLASS EXERCISE


E. Some who disapprove of me are aardvarks.
F. Some of my closest friends are no aardvarks.
1) BCD 2) ABD 3) BCE 4) ABE

26. A. Some mammals are carnivores. B. All whales are mammals.


C. All whales are aquatic animals. D. All whales are carnivores.
E. Some aquatic animals are mammals. F. Some mammals are whales.
1) ADF 2) ABC 3) AEF 4) BCE

27. A. First-year students of this college like to enter for the prize.
B. All students of this college rank as University students.
C. First-year students of this college are entitled to enter for the prize.
D. Some who rank as University students are first-year students.
E. All University students are eligible to enter for the prize.
F. All those who like to enter are entitled to enter for the prize.
1) AEF 2) ABC 3) BEC 4) CDF

28. A. All candid men are persons who acknowledge merit in a rival.
B. Some learned men are very candid.
C. Some learned men are not persons who acknowledge merit in a rival.
D. Some learned men are persons who are very candid.
E. Some learned men are not candid.
F. Some persons who recognize merit in a rival are learned.
1) ABE 2) ACF 3) ADE 4) BAF

29. A. Some beliefs are uncertain. B. Nothing uncertain is worth dying for.
C. Some beliefs are worth dying for. D. All beliefs are uncertain.
E. Some beliefs are certain. F. No belief is worth dying for.
1) ABF 2) BCD 3) BEF 4) BDF

30. A. No lunatics are fit to serve on a jury.


B. Everyone who is sane can do logic.
C. None of your sons can do logic.
D. Some who can do logic are fit to serve on a jury.
E. All who can do logic are fit to serve on a jury.
F. Everyone who is sane is fit to serve on a jury.
1) BDE 2) BEF 3) BDF 4) ADE

11
CATapult
VERBAL REASONING

31. A. Different hues are obtained from primary colours.


B. A rainbow consists of several hues.
C. Blue and red can give different hues.
D. Red is a primary colour.
E. Blue can give different hues.
F. Red can give different hues.
1) ACE 2) AEF 3) ADF 4) CDF

32. A. Mathew and Paul are brothers. B. Siblings are known to quarrel often.
C. Mathew and Paul do not quarrel. D. All those who quarrel are siblings.
E. Paul and Mathew quarrel often. F. Mathew and Paul cannot be siblings.
1) BDE 2) ADF 3) CDE 4) ABE

33. A. Painting and music are forms of art. B. Art is a symptom of culture.
C. Culture and art are complementary. D. Music is a form of art.
E. Painting is a form of art. F. Music shows culture.
1) BDF 2) AEF 3) ACE 4) CEF

34. A. Metal is good material for desks. B. Desks are made of metal.
C. This object is not a desk. D. This object is a desk.
E. This object is not made of metal. F. This is made of metal.
1) ADF 2) BCE 3) ABD 4) BDF

35. A. Some men are bald. B. Bald people are intelligent.


C. Raman is a man. D. Raman is bald.
E. Raman is intelligent. F. All men are intelligent.
1) ABF 2) BDE 3) BCD 4) BEF

36. A. No barbarian is a gentleman. B. Some gentlemen are barbarians.


C. Some gentlemen are rude. D. No gentlemen are rude.
E. Some barbarians are not rude. F. All barbarians are rude.
1) ABE 2) BCE 3) ADF 4) BDE

37. A. All leaves are green. B. All leaves have chlorophyll.


C. Chlorophyll is green. D. All plants have leaves.
E. All plants have chlorophyll. F. Only leaves have chlorophyll.
1) BDE 2) BEF 3) BDF 4) AEF

12
Chapter VA
SYLLOGISMS 4.1

38. A. All good bridge players play good chess.

THEORY
B. Many good chess players are not bridge players.
C. Goren is a good bridge player.
D. Goren plays chess well.

PRACTICE EXERCISE CLASS EXERCISE


E. Spassky plays chess well.
F. Spassky plays bridge badly.
1) ABD 2) BEF 3) ACE 4) ACD

39. A. All snakes are reptiles. B. All reptiles are not snakes.
C. All reptiles are cold blooded. D. All snakes lay eggs.
E. All reptiles lay eggs. F. Snakes are cold blooded.
1) ADE 2) BDE 3) ABE 4) ACF

40. A. College students are intelligent. B. Intelligence is a collegian’s attribute.


C. Ram’s sister is a college student. D. Ram goes to college.
E. All intelligent persons go to college. F. Ram is an intelligent person.
1) ADF 2) BCD 3) ABF 4) ABD

13
CATapult
VERBAL REASONING

PRACTICE EXERCISE-1

Directions: Use the five rules to determine in which questions the third statement logically follows
from the first two statements.

1. Some juries are not unbiased.


All unbiased are just.
Some just are not juries.

2. All celebrities who endorse unhealthy products are unethical.


Some unethical people are driven by commercial motives.
Some celebrities driven by commercial motives endorse unhealthy products.

3. All religions are antiquated.


All antiquated beliefs predate science.
Some of the antiquated beliefs that predate science were religious.

4. No crash diets based on starvation are healthy.


Some crash diets are healthy.
Some crash diets are not based on starvation.

5. All masqueraders are skilled at voice modulation.


All skilled at voice modulation are charming.
Some charming people are masqueraders.

Directions: Use Venn diagrams to determine in which questions the third statement logically follows
from the first two statements.

6. No frogs are toads.


All toads are amphibians.
No frogs are amphibians.

14
Chapter VA
SYLLOGISMS 4.1

7. Some books are mystery.

PRACTICE EXERCISE CLASS EXERCISE THEORY


No mystery is adventure.
Some books are adventure.

8. All social are networks.


Some networks are not media.
Some media are not social.

9. All bands are ribbons.


No ribbons are scarves.
Some scarves are bands.

10. All wires are plastic.


All wires are copper.
Some plastic is copper.

Directions: In each question below, three statements (I, II and III) are followed by four conclusions
(A, B, C and D). You have to take the given statements to be true even if they seem to be at variance
with commonly known facts. Read all the conclusions and then decide which logically follow/s from
the given statements, disregarding commonly known facts.

11. Statements:
I. Some barbers are fashion designers.
II. No fashion designers are businessmen.
III. Some businessmen are traders.
Conclusions:
A. No fashion designers are traders. B. Some traders are not fashion designers.
C. Some fashion designers are traders. D. Some barbers are not businessmen.
1) Either A, B and D or C, B and D follow.
2) Either A and D or C and D follow.
3) Either A and B or C and B follow.
4) None of the above

15
CATapult
VERBAL REASONING

Directions: Each question contains six statements followed by four sets of combinations of three.
Choose the set in which the statements are logically related.

12. A. Reena and Meena are sisters. D. All those who quarrel are sisters.
B. Sisters are often known to quarrel. E. Reena and Meena quarrel often.
C. Reena and Meena do not quarrel. F. Reena and Meena cannot be sisters.
1) BDE 2) ADF 3) CDE 4) ABE

13. A. All crows are birds. D. All crows lays eggs.


B. All birds are not crows. E. All birds lay eggs.
C. All birds are warm blooded F. Crows are warm blooded.
1) ADE 2) BED 3) ABE 4) ACF

Directions: Each of the questions below contains four arguments of three sentences each. Choose
the set in which the third statement is a logical conclusion of the first two. (Past CAT Questions)

14. A. All Ts are square; All square are rectangular; All Ts are rectangular.
B. Some fat are elongated; Some elongated things are huge; Some fat are huge.
C. Idiots are bumblers; Bumblers fumble; Idiots fumble.
D. Water is good for health; Health foods are rare; Water is rare.
1) D only 2) C only 3) Both A and C 4) A, B, C and D

15. A. Some Xs are Ps; Some Ps are Ys; Some Xs are Ys.
B. All Sonas are bright; Some bright are crazy; Some Sonas are crazy.
C. No faith is strong; No strong have biceps; No faith has biceps.
D. All men are weak; Some weak are strong; Some strong are weak.
1) A and D 2) C only 3) D only 4) None of these

16. A. Some icicles are cycles; All cycles are men; Some icicles are men.
B. All girls have teeth; No teeth are yellow; No girls are yellow.
C. No hand is foot; Some foot are heads; Some hands are heads.
D. Every man has a wife; All wives are devoted; No devoted has a husband.
1) A, B and C only 2) A and B
3) C and B 4) A, B, C and D

17. A. No sun is not white; All moon is sun; All moon is white.
B. All windows are open; No open space is allocated; All window is closed space.
C. No German can fire; All Americans bombard; Both, Germans and Americans can fight.
D. No X is Z; No Z is Y; No X is Y.
1) A only 2) B only 3) C only 4) D only

16
Chapter VA
SYLLOGISMS 4.1

18. A. No cowboys laugh; Some who laugh are sphinxes; Some sphinxes are not cowboys.

PRACTICE EXERCISE CLASS EXERCISE THEORY


B. All ghosts are fluorescent; Some ghosts do not sing; Some singers are not fluorescent.
C. Cricketers indulge in swearing; Those who swear are hanged; Some who are hanged
are not cricketers.
D. Some crazy people are pianists; All crazy people are whistlers; Some whistlers are pianists.
1) A and B 2) C only 3) A and D 4) B only

19. A. All good people are knights; All warriors are good people; All knights are warriors.
B. No footballers are ministers; All footballers are tough; Some ministers are players.
C. All pizzas are snacks; Some meals are pizzas; Some meals are snacks.
D. Some barkers are musk-deer; All barkers are sloth bears. Some sloth bears are
musk-deer.
1) C and D 2) B and C 3) A only 4) C only

Directions: Each question consists of five statements followed by options consisting of three statements
put together in a specific order. Choose the option which indicates a valid argument, that is, where
the third statement is a conclusion drawn from the preceding two statements. (Past CAT Questions)

Example:
A. All cigarettes are hazardous to health.
B. Brand X is cigarette.
C. Brand X is hazardous to health.
ABC is a valid option, where statement C can be concluded from statements A and B.

20. A. All software companies employ knowledge workers.


B. Tara Tech employs knowledge workers.
C. Tara Tech is a software company.
D. Some software companies employ knowledge workers.
E. Tara Tech employs only knowledge workers.
1) ABC 2) ACB 3) CDB 4) ACE

21. A. Traffic congestion increases carbon monoxide in the environment


B. Increase in carbon monoxide is hazardous to health.
C. Traffic congestion is hazardous to health.
D. Some traffic congestion does not cause increased carbon monoxide.
E. Some traffic congestion is not hazardous to health.
1) CBA 2) BDE 3) CDE 4) BAC

17
CATapult
VERBAL REASONING

22. A. Apples are not sweets. B. Some apples are sweets.


C. All sweets are tasty. D. Some apples are not tasty.
E. No apple is tasty.
1) CEA 2) BDC 3) CBD 4) EAC

23. A. Some towns in India are polluted. B. All polluted towns should be destroyed.
C. Town Meghana should be destroyed. D. Town Meghana is polluted.
E. Some towns in India should be destroyed.
1) BDE 2) BAE 3) ADE 4) CDB

24. A. No patriot is a criminal B. Bundledas is not a criminal.


C. Bundledas is a patriot. D. Bogasdas is not a patriot.
E. Bogasdas is a criminal
1) ACB 2) ABC 3) ADE 4) ABE

25. A. Ant eaters like ants. B. Boys are ant eaters.


C. Balaram is an ant eater. D. Balaram likes ants.
E. Balaram may eat ants.
1) BCA 2) ADC 3) ABE 4) ACD

26. A. All actors are handsome. B. Some actors are popular.


C. Ram is handsome. D. Ram is popular actor.
E. Some popular people are handsome.
1) ACD 2) ABE 3) DCA 4) EDC

27. A. Modern industry is technology driven. B. BTI is a modern industry.


C. BTI is technology driven. D. BTI may be technology driven.
E. Technology driven industry is modern.
1) ABC 2) ABD 3) BCA 4) EBC

28. A. All Golmal islanders are blue coloured people.


B. Some smart people are not blue coloured people.
C. Some babies are blue coloured.
D. Some babies are smart.
E. Some smart people are not Golmal islanders.
1) BCD 2) ABE 3) CBD 4) None of these

29. A. MBAs are in great demand.


B. Ram and Sita are in great demand.
C. Ram is in great demand.
D. Sita is in great demand.
E. Ram and Sita are MBAs.
1) ABE 2) ECD 3) AEB 4) EBA

18
Chapter VA
SYLLOGISMS 4.1

PRACTICE EXERCISE-2

Directions: From the six statements choose the set of three statements where the third statement
can be logically drawn from the first two statements.

1. A. Trout lay eggs. B. Mammals have hair.


C. Trout are fish. D. Platypuses are mammals.
E. All fish lay eggs. F. Platypuses have hair.
1) ACE 2) DFB 3) AEF 4) CEA

2.
A. A little power corrupts a little. B. Power corrupts.
C. A dictator has absolute power. D. A dictator is absolutely corrupt.
E. Corruption is absolute power. F. Absolute power corrupts absolutely.
1) BAF 2) EDF 3) BFC 4) FCD

3. A. Some men are dancers. B. Some men are not dancers.


C. Sheela may be a dancer. D. Some women are dancers.
E. Some women are not dancers. F. Sheela is a woman.
1) ABD 2) DFC 3) DEF 4) ECF

4. A. Jumbo is a cricketer. B. All cricketers play ball.


C. All cricketers bat. D. Jumbo plays ball.
E. Cricketers are fond of chocolates. F. Chocolates affect the teeth.
1) ABC 2) AEF 3) EFD 4) BAD

5. A. Keto likes to look adult. B. Keto is a boy.


C. Boys play a lot. D. Keto likes swimming.
E. Keto likes to wear long pants. F. All adults wear long pants.
1) AEF 2) AFE 3) BCD 4) FEA

6. A. All exits are unceremonious.


B. No exits are vagrants.
C. Some unceremonious things are vagrants.
D. No vagrants are uncouth.
E. All uncouth things are unceremonious.
F. All exits are uncouth.
1) CED 2) DFB 3) AEF 4) FAC

7. A. All apes have hair. B. All gorillas have hair.


C. All apes are gorillas. D. All primates have hair.
E. All apes are primates. F. All gorillas are grey.
1) DBE 2) CAB 3) ABC 4) DEA

19
CATapult
VERBAL REASONING

8. A. Akhet is a ship’s crew. B. Ashet is a ship’s crew.


C. All ship’s crew are submarine crew. D. Akhet and Ashet are friends.
E. Ashet is a submarine crew. F. Akhet is a submarine crew.
1) BCE 2) BCA 3) EFD 4) BEC

9. A. All green is white. B. All white is black.


C. All green is black. D. All black is white.
E. All white is green. F. All black is green.
1) CDA 2) ADC 3) EFB 4) DEB

10. A. Some cricket players are golfers. B. All golfers are cricket players.
C. Susheel is a cricket player. D. Sohail is a golfer.
E. Susheel is a golfer. F. Sohail is a cricket player.
1) ACE 2) BCE 3) DBC 4) EBC

11. A. Some Sioux are Indians. B. Some Sioux do not carry Tomahawks.
C. All Mohicans carry Tomahawks. D. All Indians carry Tomahawks.
E. No Sioux are Mohicans. F. No Indians carry Tomahawks.
1) CEB 2) ADB 3) FAB 4) DEC

12. A. All monoliths are caged. B. All monoliths are menageries.


C. Some menageries are monstrous. D. Some monoliths are monstrous.
E. Some caged are menageries. F. Some monoliths are caged.
1) BDC 2) FED 3) AEF 4) BFD

Directions: Each question has a set of three or four statements. Each statement has three segments.
Choose the option where the third segment in the statement can be logically deduced using both
the preceding two, but not just one of them.

13. A. All chairs are desks. Some tables are desks. Some chairs are desks.
B. Some umbrellas are pencils. All chocolates are pencils. Some chocolates are umbrellas.
C. Some watches are pins. No cap is a pin. No watch is a cap.
1) B only 2) A and C 3) A and B 4) None of these

14. A. All singers can dance. Some animals can dance. Some animals are singers.
B. Only heroes act in a movie. Gautam acts in movies. Gautam is a hero.
C. Some brave are famous. All famous are handsome. Some brave are handsome.
1) B and C 2) B only 3) A and C 4) A and B

20
Chapter VA
SYLLOGISMS 4.1

15. A. No doctor is an engineer. Some managers are engineers. Some managers are not

PRACTICE EXERCISE CLASS EXERCISE THEORY


doctors.
B. All black are green. Yellow are black. Some yellow are not green.
C. All companies are making profits. Werner is making profits. Werner is a company.
1) A and B 2) B and C 3) A only 4) C only

16. A. No yankees have brains. Some who have brains are donkeys. Donkeys are not yankees.
B. All movies are watchable. Some movies do not have songs. Movies without songs are
not watchable.
C. All players play cricket. Ravi plays cricket. Ravi is a player.
D. No bus stops have queues. Bandra bus stop has a queue. Bandra bus stop does not
have buses.
1) A and B 2) C only 3) C and D 4) None of these

17. A. Many juices are concentrates. Some juices are not fresh. No fresh are concentrates.
B. No lattes are espressos. Some cappuccinos are not lattes. Some cappuccinos may be
espressos.
C. Some sodas are ice-creams. All milkshakes are ice-creams. All milk-shakes are sodas.
1) A only 2) B only 3) A and C 4) B and C

18 A. A few cucumbers are zucchinis. All zucchinis are not asparagus. Some asparagus
are cucumbers.
B. Some broccolis are cauliflowers. Some cabbages are cauliflowers. Some cabbages are
broccolis.
C. Some carrots are not radishes. No potatoes are carrots. Some potatoes are not rad-
ishes.
1) A only 2) C only 3) A and B 4) None of the above

19. A. Some eggs are boiled. Some eggs are poacheD. No boiled are poached.
B. All eggs are omelettes. Some omelettes are not fried. Some eggs are not fried.
C. A few eggs are coddled. All coddled are scrambled. Some eggs are scrambled.
1) A and B 2) B only 3) C only 4) A and C

20. A. Not all psychology is sociology. Only anthropology is sociology. Some psychology may be
anthropology.
B. No botany is biology. No biology is zoology. No zoology is botany.
C. Only algebra is geometry. All algebra is mathematics. All geometry is mathematics.
1) A and B 2) A and C 3) B and C 4) A, B and C

21. A. Some thieves are notorious. Many thieves are assassins. Some assassins are thieves.
B. All assassins are spies. All spies are stealthy. Only assassins are stealthy.
C. Some murderers want revenge. No murderers are assassins. Some assassins do not want
revenge.
1) A and B 2) B and C 3) A and C 4) None of the above

21
CATapult
VERBAL REASONING

Directions: Each question has a main statement followed by four statements labelled A, B, C and
D. From the options, choose that ordered pair of statements where the first statement implies the
second, and the two statements are logically consistent with the main statement.

22. The library allows you to borrow a second book only if you return the first one.
A. I borrowed a second book from the library.
B. I returned the first book to the library.
C. I did not borrow a second book from the library.
D. I did not return the first book to the library.
1) BA 2) AD 3) CD 4) DC

23. Snow falls when the temperature is below freezing.


A. It is snowing.
B. It is not snowing.
C. The temperature is below freezing.
D. The temperature is not below freezing.
1) BD 2) DB 3) AC 4) BC

24. Riddhi will go to the party if her friends Riya and Ritika go as well.
A. Riddhi goes to the party.
B. Riddhi does not go to the party.
C. Ritika goes to the party.
D. Riya goes to the party.
1) CA 2) AD 3) DB 4) None of the above

25. You must use the gift voucher to shop online or at the mall.
A. You used the gift voucher to shop at the mall.
B. You did not use the gift voucher to shop online.
C. You used the gift voucher to shop online.
D. You did not use the gift voucher to shop at the mall.
1) DB 2) CA 3) DC 4) Both (1) and (2)

26. If the clock strikes, the cuckoo will not come out.
A. The clock does not strike.
B. The cuckoo does not come out.
C. The clock strikes.
D. The cuckoo comes out.
1) CB 2) AD 3) AB 4) CD

22
Chapter VA
SYLLOGISMS 4.1

Directions: Each question has some statements followed by a set of conclusions. Choose the

PRACTICE EXERCISE CLASS EXERCISE THEORY


conclusions that logically follow from both the given statements.

27. Statements:
Some nomads are wanderers. Not all wanderers are lost.
Conclusions:
A. Some nomads are lost.
B. Not all nomads are lost.
1) Only A 2) Only B 3) Both A and B 4) Neither A nor B

28. Statements:
Most cyclists are athletic. All athletic people can run marathons.
Conclusions:
A. Some cyclists can run marathons.
B. Some cyclists are athletic people.
1) Only A 2) Only B 3) Both A and B 4) Neither A nor B

29. Statements:
All bubblegum is pink. All bubblegum is green.
Conclusions:
A. Some green is pink.
B. All bubblegum is pink and green.
1) Only A 2) Only B 3) Both A and B 4) Neither A nor B

30. Statements:
All rocks are stones. Some stones may be gems.
Conclusions:
A. Some gems are rocks.
B. Some rocks may be stones.
1) Only A 2) Only B 3) Both A and B 4) Neither A nor B

23
CATapult
VERBAL REASONING

PRACTICE EXERCISE-3

Directions: From the six statements choose the set of three statements where the third statement
can be logically drawn from the first two statements.

1. A. Only puddings are pies. B. No pies are cheesecakes.


C. No cheesecakes are waffles. D. Only waffles are pies.
E. Some cheesecakes are not puddings. F. Only waffles are puddings.
1) ADF 2) BAE 3) CFE 4) DCB

2. A. No cash is credit. B. Some cheque is credit.


C. Some debit is credit. D. Some cash is not cheque.
E. All cash is cheque. F. Some debit is not cash.
1) ACF 2) DFB 3) EAB 4) FCA

3. A. No Starks are Lannisters. B. Some Baratheons are Targaryens.


C. Some Starks are Targaryens. D. Some Lannisters are Baratheons.
E. Some Starks may be Baratheons. F. No Lannisters are Baratheons.
1) AFC 2) CBE 3) DAF 4) EBC

4. A. Some wolves are deer. B. Some deer are lions.


C. No wolves are lions. D. All lions are deer.
E. Some lions are wolves. F. No deer are wolves.
1) CBF 2) ABE 3) DCF 4) EDA

5. A. All dragons can fly. B. Some dragons cannot swim.


C. No krakens can fly. D. Some krakens are not dragons.
E. Some dragons are krakens. F. All krakens can swim.
1) CAD 2) EFB 3) FBD 4) BCE

6. A. Some wives are good. B. Some wives are politicians.


C. No politicians are good. D. Some wives may be lawyers.
E. Some lawyers may be good. F. Some lawyers are politicians.
1) ACB 2) CEF 3) FBD 4) Both (1) and (3)

7. A. Many carbon are sulphur. B. Sulphur are never phosphorus.


C. Phosphorus are always carbon. D. Some sulphur are not carbon.
E. No carbon are phosphorus. F. All phosphorus are sulphur.
1) ACF 2) BAE 3) EBD 4) None of the above

24
Chapter VA
SYLLOGISMS 4.1

8. A. Only Odins are Lokis. B. Only Lokis are Thors.

PRACTICE EXERCISE CLASS EXERCISE THEORY


C. Some Odins are Lokis. D. Some Lokis may be Odins.
E. Some Odins may be Thors. F. All Thors are Odins.
1) BFC 2) BAF 3) CED 4) Both (1) and (2)

9. A. Some rings are evil. B. Some power is evil.


C. All power is cursed. D. Some rings are cursed.
E. All rings are power. F. All evil are rings.
1) AEB 2) DCF 3) FBE 4) None of the above

10. A. Some teas are earl grey. B. No teas are lemon.


C. Only lemon are teas. D. All earl grey are lemon.
E. No earl grey are teas. F. Some lemon may be earl grey.
1) CAF 2) BFA 3) DBE 4) EDC

Directions: Each question has a main statement followed by four statements labelled A, B, C and
D. From the options, choose that ordered pair of statements where the first statement implies the
second, and the two statements are logically consistent with the main statement.

11. When the bell rings, the students leave the classroom.
A. The bell rang.
B. The bell did not ring.
C. The students left the classroom.
D. The students did not leave the classroom.
1) AB 2) CA 3) DB 4) BD

12. Harry could take up only one subject: either Transfiguration or Potions.
A. Harry took up Potions.
B. Harry did not take up Potions.
C. Harry took up Transfiguration.
D. Harry did not take up Transfiguration.
1) AC 2) AD 3) CB 4) Both (2) and (3)

13. Only when her mobile buzzes, does Kajal pick it up.
A. Kajal’s mobile is buzzing.
B. Kajal picks up her mobile.
C. Kajal does not pick up her mobile.
D. Kajal’s mobile is not ringing.
1) AB 2) BA 3) CD 4) DC

25
CATapult
VERBAL REASONING

14. The flight will be delayed if this fog does not clear up.
A. The flight is delayed.
B. The fog clears up.
C. The fog does not clear up.
D. The flight is not delayed.
1) BA 2) AC 3) DC 4) DB

15. We can order pizza from Pizza Hut or Domino’s.


A. We ordered from Domino’s.
B. We ordered from Pizza Hut.
C. We did not order from Pizza Hut.
D. We did not order pizza.
1) CA 2) CD 3) BA 4) None of the above

16. Ram does not drive when he drinks.


A. Ram drove. B. Ram didn’t drink.
C. Ram didn’t drive. D. Ram drank.
1) BA 2) CA 3) CD 4) DC

17. Only intelligent investors manage money well.


A. Harsh manages money well.
B. Harsh is an intelligent investor.
C. Harsh does not manage money well.
D. Harsh is not an intelligent investor.
1) DC 2) AD 3) CD 4) CB

18. The computer moves when the mouse clicks.


A. The computer moved.
B. The mouse did not click.
C. The mouse clicked.
D. The computer did not move.
1) AC 2) CA 3) CD 4) DC

19. Good girls go to heaven.


A. Sheela is a good girl.
B. Sheela will go to heaven.
C. Sheela is not a good girl.
D. Sheela will not go to heaven.
1) AC 2) CD 3) DC 4) None of these

26
Chapter VA
SYLLOGISMS 4.1

Directions: Each question has some statements followed by a set of conclusions. Choose the con-

PRACTICE EXERCISE CLASS EXERCISE THEORY


clusions that logically follow from both the given statements.

20. Statements:
Some leather is tanned. No hide is tanned.
Conclusions:
A. Some leather is not hide.
B. Some hide is not leather.
1) A only 2) B only
3) Both A and B 4) Neither A nor B

Directions: Use the five rules to determine in which questions the third statement logically follows
from the first two statements.

21. All philosophers are intellectuals.


All intellectuals are valuable.
Some valuable things are not philosophers.

22. No comic books are literature.


No literature is populist.
No populist books are comic books.

23. All constellations have stars.


Some stellar objects are not stars.
Some stellar objects are not part of constellations.

24. All dairy products are foods high in calcium.


Some foods high in calcium are not easily digestible.
Some easily digestible foods are not dairy products.

25. All math involves problems.


All math involves axioms.
Some axioms have problems.

Directions: Use Venn diagrams to determine in which questions the third statement logically follows
from the first two statements.

26. All apples are red.


All red are flowers.
Some apples are flowers.

27
CATapult
VERBAL REASONING

27. Some yellow is orange.


All orange is fruit.
All fruit is orange.

28. Some panes are clear.


Some clear are lucid.
Some panes may be lucid.

29. Some shirts are pants.


No pants are trousers.
No trousers are shirts.

30. Some rugs are carpets.


Some carpets may be red.
Some rugs may be red.

Directions: Each question has a set of three or four statements. Each statement has three segments.
Choose the option where the third segment in the statement can be logically deduced using both
the preceding two, but not just one of them.

31. A. Superstars have massive egos. Arnold has a massive ego. Arnold is a superstar.
B. All milkshakes are alcoholic. Beer is alcoholic. Beer is a milkshake.
C. Some baskets have oranges. Some baskets have apples. Baskets have either apples
or oranges.
D. No Briton wants to lie. Grant wants to lie. Grant is not a Briton.
1) A and D 2) B only 3) D only 4) C and D

32. A. No fruits have vitamins. All vegetables have vitamins. Vegetables are not fruits.
B. Some plants are money plants. Rubber plants are money plants. Money plants are
rubber plants.
C. Superman will destroy all evil. Satan is evil. Satan will destroy Superman.
D. All’s well that ends well. The play did not end. The play was not well.
1) A only 2) A and C 3) A and D 4) None of these

33. A. All thunderstorms are accompanied by hail. There is hail. There is a thunderstorm.
B. Some who talk do not smile. Nishi talks. Nishi does not smile.
C. All things made of glass are transparent. Photo frames are made of glass. Photo frames
are transparent.
D. No murderer is human. Ram is human. Ram is not a murderer.
1) A only 2) B only 3) C and D 4) C only

28
Chapter VA
SYLLOGISMS 4.1

34. A. All parkour is dangerous. Not all freerunning is dangerous. Some parkour is not

PRACTICE EXERCISE CLASS EXERCISE THEORY


freerunning.
B. All faiths are creeds. All creeds are religions. Some religions are faiths.
C. Some sands are grainy. Some sands are not pebbly. No grainy are pebbly.
1) A only 2) B only 3) C only 4) A and B

35. A. Only portals are orange. Only portals are blue. Some blue may be orange.
B. Some princes are Persian. All princesses are Indian. Some Persian are Indian.
C. Some Lara raid tombs. All those who raid tombs are Croft. Some Croft are Lara.
D. No Ezio are Altair. All Desmond are Altair. Some Desmond are Ezio.
1) A and B 2) A and C 3) C and D 4) B and D

Directions: Each question has some statements followed by a set of conclusions. Choose the
conclusions that logically follow from the given statements.

36. Statements:
All princes are kings. Only queens are kings.

Conclusions:
A. All queens are princes.
B. All princes are queens.
1) A only 2) B only 3) Both A and B 4) Neither A nor B

37. Statements:
All silk is satin. All cotton is satin.
Conclusions:
A. All cotton is silk.
B. Some silk may be cotton.
1) A only 2) B only 3) Both A and B 4) Neither A nor B

38. Statements:
No coconuts are pumpkins. Some papayas are not pumpkins.
Conclusions:
A. No coconuts are papayas.
B. Some papayas are coconuts.
1) A only 2) B only 3) Both A and B 4) Neither A nor B

29
CATapult
VERBAL REASONING

39. Statements:
All Jons know nothing. No Ygrittes know nothing.
Conclusions:
A. No Jons are Ygrittes.
B. Some Jons may be Ygrittes.
1) A only 2) B only 3) Both A and B 4) Neither A nor B

40. Statements:
All swords have edges.
All swords have hilts.
Conclusions:
A. All swords have both edges and hilts.
B. Some of those that have hilts have edges.
1) A only 2) B only 3) Both A and B 4) Neither A nor B

30
Chapter VA
SYLLOGISMS 4.1

PRACTICE EXERCISE-4

Directions: From the six statements, choose the set of three statements where the third statement
can be logically drawn from the first two statements. (Past CAT questions)

1. A. Smoking causes cancer.


B. All cigarettes are hazardous to health.
C. Smoking does not cause cancer sometimes.
D. One brand of cigarettes is Cham-cham.
E. Brand X causes cancer.
F. Cham-cham is bad for health.
1) ABE 2) BDF 3) ABD 4) ABC

2. A. Laxman is a man. B. Meera is Laxman’s wife.


C. Some women are islands. D. No man is an island.
E. Meera is not an island. F. Laxman is not an island.
1) ADE 2) ABE 3) ADF 4) CDE

3. A. All bosses are fragrant. B. All bosses are majestic.


C. All bosses are pious. D. All pious need air.
E. All bosses need air. F. All pious need water.
1) ABC 2) BCD 3) CDE 4) CEF

4. A. No attendants are qualified. B. Some nurses are qualified.


C. Some nurses are not qualified. D. All nurses are attendants.
E. All attendants are qualified. F. Some attendants are qualified.
1) ABF 2) CDF 3) BDF 4) BDE

5. A. Mary is John’s wife. B. Mary and John danced together.


C. Mary wears John’s ring. D. Husbands and wives danced the last waltz.
E. John loves Mary. F. John danced last with Mary.
1) ADF 2) ABD 3) ACE 4) AEF

6. A. Some college athletes are professionals.


B. No college athlete is a professional.
C. Some professionals are well-paid.
D. All professionals are well-paid.
E. All well-paid persons are professionals.
F. No well-paid person is a college athlete.
1) BEF 2) ABF 3) BDF 4) ACF

31
CATapult
VERBAL REASONING

7. A. Some intolerant are poor thinkers.


B. Some poor thinkers are intolerant.
C. All people with high ideals are intolerant.
D. No poor thinker is intolerant.
E. No poor thinker has high ideals.
F. Some people with high ideals are poor thinkers.
1) CDE 2) CDF 3) ABD 4) BCF

8. A. All engineers can sing. B. No music lover can sing.


C. All who can sing are music lovers. D. All music lovers can sing.
E. Some who can sing are engineers. F. All engineers are music lovers.
1) ACE 2) ACF 3) ABF 4) ACD

9. A. Some well-dressed people are sociable.


B. All sociable people are well-dressed.
C. Some well-dressed people are dull.
D. No dull person is well-dressed.
E. Some sociable people are dull.
F. Some dull ones are well-dressed.
1) ACE 2) BCE 3) ADE 4) BEF

10. A. Iran and Iraq are members of the UN.


B. Iran and Iraq are not friends.
C. Iran and Iraq are neighbours.
D. Some UN members are friends.
E. Not all members of the UN are friends.
F. All neighbours are not friends.
1) ABE 2) ABD 3) CDF 4) AEF

11. A. Good managers are intuitive. B. Some managers are women.


C. Supriya is a good manager. D. Supriya is a woman.
E. Some women are intuitive. F. Supriya is intuitive.
1) BCE 2) ABD 3) ACF 4) ADF

12. A. All who are sincere are graduates. B. Some graduates are not sincere.
C. All who are sincere are dull. D. All graduates are dull.
E. Some who are dull are graduates. F. No one who is dull is sincere.
1) BEF 2) ADF 3) ABF 4) ADC

13. A. Sham won a lottery.


B. Sham lost in a chess game.
C. Sham is not intelligent.
D. One need not be intelligent to win a lottery.
E. One need not be intelligent to win a chess game.
F. Sham plays chess.
1) BEF 2) ACD 3) BDE 4) BDF

32
Chapter VA
SYLLOGISMS 4.1

14. A. All boys are good. B. Some girls are bad.

PRACTICE EXERCISE CLASS EXERCISE THEORY


C. Good people are educated. D. Boys are educated.
E. Ram is an educated boy. F. Lata is an educated girl.
1) BCF 2) ACD 3) DEF 4) ADF

15. A. All vegetarians eat meat.


B. All those who eat meat are not vegetarians.
C. All those who eat meat are herbivorous.
D. All vegetarians are carnivorous.
E. All those who eat meat are carnivorous.
F. Vegetarians are herbivorous.
1) BCE 2) ABE 3) ACD 4) ACF

16. A. All roses have thorns. B. All roses have nectar.


C. All plants with nectar have thorns. D. All shrubs have roses.
E. All shrubs have nectar. F. Some roses have thorns.
1) BEF 2) FEB 3) BDE 4) ACF

17. A. Some abra are dabra. B. All abra are cabra.


C. All dabra are abra. D. All dabra are not abra.
E. Some cabra are abra. F. Some cabra are dabra
1) AEF 2) BCF 3) ABD 4) BCE

18. A. No spring is a season. B. Some seasons are springs.


C. Some seasons are autumns. D. No seasons are autumns.
E. Some springs are not autumns. F. All springs are autumns.
1) DFA 2) BEF 3) CEB 4)DEB

19. A. All falcons fly high. B. All falcons are blind.


C. All falcons are birds. D. All birds are yellow.
E. All birds are thirsty. F. All falcons are yellow.
1) ABC 2) CDF 3) DEF 4) BCA

20. A. No wires are hooks. B. Some springs are hooks.


C. All springs are wires. D. Some hooks are not wires.
E. No hook is a spring. F. All wires are springs.
1) AED 2) BCF 3) BEF 4) ACE

21. A. No plane is a chain. B. All manes are chains.


C. No mane is a plane. D. Some manes are not planes.
E. Some planes are manes. F. Some chains are not planes.
1) ACD 2) ADF 3) ABC 4)CDF

33
CATapult
VERBAL REASONING

22. A. All rolls are nice. B. All toys are nice.


C. All toys are dolls. D. Some toys are nice.
E. Some nice things are dolls. F. No doll is nice.
1) CDE 2) CEF 3) ACD 4) BEF

23. A. Some buildings are not sky-scrapers.


B. Some sky-scrapers are not buildings.
C. No structure is a sky-scraper.
D. All sky-scrapers are structures.
E. Some sky-scrapers are buildings.
F. Some structures are not buildings.
1) ACE 2) BDF 3) CDE 4) ACF

24. A. All bins are buckets. B. No bucket is a basket.


C. No bin is a basket. D. Some baskets are buckets.
E. Some bins are baskets. F. No basket is a bin.
1) BDE 2) ACB 3) CDF 4) ABF

25. A. Some men are bad. B. All men are sad.


C. All bad things are men. D. All bad things are sad.
E. Some sad things are men. F. Some sad things are bad.
1) AFE 2) BCF 3) BDA 4) BCE

26. A. Some bubbles are not dubbles. B. Some dubbles are not bubbles.
C. No one who is rubbles is dubbles. D. All dubbles are rubbles.
E. Some dubbles are bubbles. F. Some who are rubbles are not bubbles.
1) ADF 2) DEF 3) ABC 4) BDF

27. A. All Toms are bright. B. No bright Toms are Dicks.


C. Some Toms are Dicks. D. Some Dicks are bright.
E. No Tom is a Dick. F. No Dick is a Tom.
1) ABC 2) BEF 3) ABF 4) CDA

28. A. All witches are nasty. B. Some devils are nasty.


C. All witches are devils. D. All devils are nasty.
E. Some nasty are devils. F. No witch is nasty.
1) BCD 2) CDA 3) DEC 4) FEC

29. A. No tingo is a bingo. B. All jingoes are bingoes.


C. No jingo is a tingo. D. Some jingoes are not tingoes.
E. Some tingoes are jingoes. F. Some bingoes are not tingoes.
1) ABC 2) ACB 3) DFA 4) BDA

34
Chapter VA
SYLLOGISMS 4.1

30. A. Some pins are made of tin. B. All tin is made of copper.

PRACTICE EXERCISE CLASS EXERCISE THEORY


C. All copper is used for pins. D. Some tin is copper.
E. Some pins are used for tin. F. Some copper is used for tin.
1) ABC 2) CEF 3) CDA 4) ABE

31. A. An ostrich lays eggs. B. All birds lay eggs.


C. Some birds can fly. D. An ostrich cannot fly.
E. An ostrich is a bird. F. An ostrich cannot swim.
1) BEA 2) ABE 3) DEC 4) ECB

32. A. Some paper is wood. B. All wood is good.


C. All that is good is wood. D. All wood is paper.
E. All paper is good. F. Some paper is good.
1) BED 2) BDF 3) FAB 4) FBA

33. A. All bricks are tricks. B. Some tricks are shrieks.


C. Some that are shrieks are bricks. D. Some tricks are not bricks.
E. All tricks are shrieks. F. No tricks are shrieks.
1) EAC 2) BCD 3) ABC 4) EDC

34. A. No wife is a life. B. All life is strife.


C. Some wife is strife. D. All that is wife is life.
E. All wife is strife. F. No wife is strife.
1) BEF 2) FCB 3) ABF 4) BDE

35. A. Some crows are flies. B. Some flies are mosquitoes.


C. All mosquitoes are flies. D. Some owls are flies.
E. All owls are mosquitoes. F. Some mosquitoes are not owls.
1) ABC 2) CEF 3) ADE 4) EDB

36. A. Six is five. B. Five is not four.


C. Some five is ten. D. Some six is twelve.
E. Some twelve is five. F. Some ten is four.
1) ADE 2) ABC 3) AEF 4) EDC

37. A. Poor girls want to marry rich boys.


B. Rich girls want to marry rich boys.
C. Poor girls do not want to marry rich girls.
D. Rich boys want to marry rich girls.
E. Poor girls want to marry rich girls.
F. Rich boys want to marry poor girls.
1) ADE 2) ABC 3) BCD 4) DEF

35
CATapult
VERBAL REASONING

38. A. Some sand is sandal. B. All sandal is band.


C. All band is sand. D. No sand is sandal.
E. No band is sand. F. Some band is sandal.
1) BCA 2) AFE 3) DEC 4) CED

39. A. No bird is viviparous. B. All mammals are viviparous.


C. Bats are viviparous. D. No bat is a bird.
E. No bird is a mammal. F. All bats are mammals.
1) ADC 2) ABE 3) FBA 4) AFC

40. A. No mother is a nurse. B. Some nurses like to work.


C. No woman is a prude. D. Some prude are nurses.
E. Some nurses are women. F. All women like to work.
1) ABE 2) CED 3) FEB 4) BEF

41. A. Oranges are sweet. B. All oranges are apples.


C. Some sweet things are apples. D. Some oranges are apples.
E. All sweet are sour. F. Some apples are sour.
1) DAC 2) CDA 3) BCA 4) FEC

42. A. Zens are Marutis. B. Zens are fragile.


C. Marutis are fragile. D. All stable are weak.
E. Marutis can beat Opels. F. Opels are stable.
1) ACB 2) EFD 3) CEA 4) ABC

43. A. Dogs sleep in the open. B. Sheep sleep indoors.


C. Dogs are like sheep. D. All indoors are sheep.
E. Some dogs are not sheep. F. Some open are not sheep.
1) DEF 2) DCA 3) ABE 4) FBD

Directions: Each question consists of five statements followed by options consisting of three statements
put together in a specific order. Choose the option which indicates a valid argument, that is, where
the third statement is a conclusion drawn from the preceding two statements.
Example:
A. All cigarettes are hazardous to health.
B. Brand X is cigarette.
C. Brand X is hazardous to health.
ABC is a valid option, where statement C can be concluded from statements A and B.

36
Chapter VA
SYLLOGISMS 4.1

44.

PRACTICE EXERCISE CLASS EXERCISE THEORY


A. Dinosaurs are pre-historic creatures. Water-buffaloes are not dinosaurs. Water-buffaloes are
not pre-historic creatures.
B. All politicians are frank. No frank people are crocodiles. No crocodiles are politicians.
C. No diamond is quartz. No opal is quartz. Diamonds are opals.
D. All monkey like bananas. Some GI Joes like bananas. Some GI Joes are monkeys.
1) C only 2) B only 3) A and D 4) B and C

45.
A. MD is an actor. Some actors are pretty. MD is pretty.
B. Some men are cops. All cops are brave. Some brave are cops.
C. All cops are brave. Some men are cops. Some men are brave.
D. All actors are pretty. MD is not an actor. MD is not pretty.
1) D only 2) C only 3) A only 4) A, B and C

46.
A. Citizens of Yes Islands speak only the truth. Citizens of Yes Islands are young people. Young
people speak only the truth.
B. Citizens of Yes Islands speak only the truth. Some Yes Islands are in the Atlantic. Some
citizens of Yes Islands are in the Atlantic.
C. Citizens of Yes Islands speak only the truth. Some young people are citizens of Yes Islands.
Some young people speak only the truth.
D. Some people speak only the truth. Some citizens of Yes Islands speak only the truth. Some
people who speak only the truth are citizens of Yes Islands.
1) A only 2) B only 3) C only 4) D only

47.
A. All mammals are viviparous. Some fish are viviparous. Some fish are mammals.
B. All birds are oviparous. Some fish are not oviparous. Some fish are birds.
C. No mammal is oviparous. Some creatures are oviparous and some not. Some creatures are
not mammals.
D. Some creatures are mammals. Some creatures are viviparous. Some mammals are viviparous.
1) A only 2) B only 3) C only 4) D only

37
CATapult
VERBAL REASONING

48.
A. Many singers are not writers. All poets are singers. Some poets are not writers.
B. Giants climb beanstalks. Some chicken do not climb beanstalks. Some chicken are not giants.
C. All explorers live in snowdrifts. Some penguins live in snowdrifts. Some penguins are explor-
ers.
D. Amar is taller than Akbar. Anthony shorter than Amar. Akbar is shorter than Anthony.
1) A only 2) B only 3) B and C 4) D only

49.
A. A few farmers are rocket scientists. Some rocket scientist catch snakes. A few farmers catch
snakes.
B. Poonam is a kangaroo. Some kangaroo are made of teak. Poonam is made of teak.
C. No bulls eat grass. All matadors eat grass. No matadors are bulls.
D. Some skunks drive Cadillacs. All skunks are polar bears. Some polar bears drive Cadillacs.
1) B only 2) A and C 3) C only 4) C and D

Directions: Each question consists of five or six statements followed by options consisting of three
statements put together in a specific order. Choose the option which indicates a valid argument,
that is, when the third statement is a conclusion drawn from the preceding two statements.

50.
I. All bikes are scooters.
II. Some scooters are not mopeds.
III. Some mopeds are bikes.
IV. Some mopeds are not bikes.
V. All mopeds are bikes.
VI. No bike is a moped.
1) V, IV, III 2) I, III, IV 3) VI, V, II 4) I, VI, II

51.
I. All pens are pencils.
II. All pencils are erasers.
III. All erasers are pens.
IV. All scales are pens.
V. All pencils are scales.
VI. All erasers are scales.
1) III, I, II 2) II, V, IV 3) VI, IV, III 4) V, I, IV

38
CATapult
VERBAL REASONING

VA-4.2 SUMMARY AND PARAGRAPH COMPLETION


THEORY

Summary Questions
Introduction

As an independent question construct, summary questions had first appeared in CAT in the 2003
retest (they had appeared a couple of times before as part of critical reasoning questions). There
were four questions that asked students to choose the option that best captured the essence of
the text, from the four alternative summaries. Then, five of these questions appeared in the 2004
CAT. After that, these questions recently made a reapparance in the 2014 CAT. They also make an
appearance, at times, in other exams. The skills required to solve these questions are essentially
the same as those required to solve general RC questions that ask for the main idea or title.

Certain points to keep in mind while solving these questions:

 Read the paragraph first and then highlight what you think are the most crucial points that
must appear in the correct answer.
 Remember that it is not necessary for illustrations and examples to be included in the sum-
mary. But the point that these examples are trying to make (the purpose of the argument)
must be present.
 Eliminate options that miss out on the essential information, contain information that con-
tradicts the paragraph, or introduce new information.
 Remember not to choose options that contain any new information, no matter how reasonable
or logical it may seem. In such questions, the answer should only summarize the paragraph,
not draw inferences from it.
 Sometimes, the only difference between two options may be that one is more direct and
establishes clearer relationships between the main actors and their environment.
 Remember that the answer should be a summary, so it should be pithy and to the point, not
unnecessarily long and rambling.

39
CATapult
VERBAL REASONING

Paragraph Completion
Introduction

Paragraph Completion is a question type that has appeared in the CAT from 2005 to 2013.
The question includes a short paragraph of 3 to 5 lines followed by 4 sentences. The task is to
identify the sentence that completes the theme of the short paragraph. It can be viewed as a
paragraph whose last sentence has been removed and you have to identify the deleted sentence
from a set of 4 options.
As you will see, it tests the same skills of understanding paragraphs in terms of content, style
and tone as Jumbled Paragraphs.

Sample Question

Directions: The following question has a paragraph from which the last sentence has been deleted.
From the given options, choose the option that completes the paragraph in the most appropriate way.

Economics has long spurred exploration on Earth. Medieval merchants risked the hazards of the
Silk Road to reach the markets of China; Portuguese caravels in the 15th century sailed beyond
the bounds of the known world, searching less for knowledge than for gold and spices. Historically,
the driver for opening frontiers has always been the search for resources. Science and curiosity
are weak drivers compared with wealth generation.

(1) So the best way to really encourage space exploration is to identify the economic benefits
of doing so.
(2) So the only way that space exploration will ever become a reality if there are resources to
be extracted from space.
(3) So space exploration, which does not offer enough economic benefits to compensate for the
economic costs, will never really take off.
(4) So space exploration, which is far riskier than exploration on Earth, will never be more than
an idle curiosity.
According to the paragraph, economics is a better motivator for exploration than science and
curiosity. The paragraph gives examples of exploration on Earth; all the options talk about space. So
the correct option should be one that suggests that space exploration should also be economically
motivated. (4), which does not mention economics at all, can therefore be ruled out. (3) is too
negative – it cannot be inferred from the paragraph that there are not enough economic benefits of
exploring space. Between (1) and (2), the former is a better answer, as the latter is too extreme:
according to the paragraph, ‘science and curiosity are weak drivers’ of exploration, so economics
is not the only motivation for space exploration, merely the best one. Hence, (1).

40
Chapter VA
SUMMARY AND PARAGRAPH COMPLETION 4.2

Types of Paragraph Completion Questions

THEORY
Paragraph Completion questions over the years have broadly fallen into two types based on the
types of options:

PRACTICE EXERCISE CLASS EXERCISE


 Similar Options
 Independent Options

Similar Options

This type of question usually has options that give a clear indication of the logical direction the
correct option is supposed to take. So the students’ task is simplified since the logical direction
need not be determined.

Example

Would a Digital Public Library of America solve all the other problems—the inflation of journal
prices, the economics of scholarly publishing, the unbalanced budgets of libraries, and the barriers
to the careers of young scholars? No. Instead, it would open the way to a general transformation
of the landscape in what we now call the information society. Rather than better business plans
(not that they don’t matter), we need a new ecology, one based on the public good instead of
private gain. It’s not an answer to the problem of sustainability.

(1) It’s a fight against capitalism.


(2) It’s a return to knowledge for knowledge’s sake.
(3) It’s an appeal to change the system.
(4) It is a plea from the old-fashioned.

In this question, the paragraph talks about the need for a Digital Public Library of America, the
problems it will and will not solve and the need for a new way of ecology based on public good.
The penultimate entence says that it is not an answer to the problem of sustainability. The options
clearly indicate the direction of the concluding sentence, it’s a…so the answer must sum up the
essence of the author’s demand for a DPLA. The essence is a demand to change the way we
look at things, a new ecology based on a different premise, in other words, a systemic change.
Option (3) best captures this. Option (1) has an aggressive tone that is different from that of the
paragraph. Options (2) and (4) talk about a return and about being old-fashioned, neither of which
are referred to in the paragraph. Hence, (3).

41
CATapult
VERBAL REASONING

Independent Options

Unlike the ‘Similar Options’ format, these questions have options that do not have any common
phrases. All options are completely different in terms of content and logical direction. Based on
the content, logical structure and tone of the passage the test-taker has to determine the content,
logical structure and tone that the correct answer option has to follow in order to complete the
theme of the paragraph.

So the key here lies in correctly understanding the content, identifying the logical direction and
tone of the passage.

Broadly the structure of the passages with independent options can be classified into two types
based on the thought flow.

 Main Idea to Supporting Idea


 Supporting Ideas to Main Idea

Main Idea to Supporting Idea

In such paragraphs the main idea is usually stated in the first two lines and the sentences thereafter
are supporting arguments.

Example
One of the things that photographs do is bring us close—closer than anything else I can think
of—to physical suffering and to bodily harm. People often talk about the horror of war, and about
the necessity of building a politics of human rights, in extremely abstract terms. I think we need to
engage, far more concretely, a series of questions: What does war actually do to people? What
does political oppression, defeat, physical suffering do? How are people broken? Perhaps that’s an
uninspiring and un-triumphant approach, but it may be one that we need. We need to understand
the horrific histories that we have inherited, and that continue to be made.
(1) We need to find a way to confront the crimes of our ancestors and our own as we commit
them.
(2) And for me, photographs are a way into those realities in ways that are truly of its kind.
(3) If there were any other way, we would have found it by now.
(4) Photographs are part of that horrifc history, for to photograph a crime is to not prevent it.

In the very first sentence the main idea is put forth: photographs are a medium that brings us
closest to physical harm or suffering. The next sentences talk about how war results in physical
suffering and the need to understand war crimes, our inheritance, in real terms rather than in
abstract terms. So the concluding sentence must reflect the main idea expressed in the first
sentence, the role or value of photographs, while adding to it. If the concluding sentence is not

42
Chapter VA
SUMMARY AND PARAGRAPH COMPLETION 4.2

about photographs then the first sentence has no role in the paragraph and it can be entirely

THEORY
about war crimes. Options (1) and (3) do not talk about photographs and hence do not conclude
the paragraph. Option (4) paints photography in a negative light, which is not the intention of the
author. Option (2) best completes the paragraph. Hence, (2).

PRACTICE EXERCISE CLASS EXERCISE


Supporting Idea to Main Idea

In these type of paragraphs the idea is not stated in the beginning but is slowly built up. Sometimes
they might not even contain the main idea. The option that completes the theme might state the
main idea or be a logical extension of the idea mentioned in the last sentences of the paragraph.

Example
In broadcasting your audience is conjectural, but it is an audience of one. Millions may be listening,
but each is listening alone, or as a member of a small group, and each has (or ought to have)
the feeling that you are speaking to him individually. More than this, it is reasonable to assume
that your audience is sympathetic, or at least interested, for anyone who is bored can promptly
switch you off by turning a knob.

(1) The essential point was that our literary broadcasts were aimed at the university students,
a small and hostile audience.
(2) But what use is the supposedly sympathetic audience that doesn’t appreciate finer nuances
of the art?
(3) But though presumably sympathetic, the audience has no power over you; it is just here that
a broadcast differs from a speech or a lecture.
(4) Listening is so much different from hearing, though they are often, often incorrectly used
inter changeably.

The paragraph is about the relationship between the speaker and the audience in broadcasting.
Each of the sentences is adding on to the preceding sentence and building towards a point since
there has been no main idea that has been put forth. So, the next statement has to make a major
statement about broadcasting while continuing the idea contained in the last sentence. This idea
is about the audience being sympathetic. Only options (2) and (3) continue this idea. Option (2)
does not complete the paragraph since it moves directly from broadcasting to the finer nuances of
art. There is no reference to art that the author makes in the passage. Option (3) best completes
the paragraph since it says how a broadcast is different from a lecture or a speech. The audience
has no power over the speaker, since they cannot interrupt him while he is speaking, unlike in a
speech or a lecture. Hence, (3).

43
CATapult
VERBAL REASONING

Strategies to Tackle Paragraph Completion Questions

1. Read the paragraph, and without looking at the options, try to imagine how it will contin-
ue. Ask yourself what the theme of the paragraph is, what the author is driving at, how he/
she will round off the ideas in the paragraph. This is particularly helpful in case of questions
which have a Supporting Ideas to Main Idea type structure. Once you look at the options,
you can eliminate the ones that do not fit with what you have already imagined. If none fit,
then reread the paragraph, and try to understand it better.
2. Identify the structure of the paragraph. If the structure proceeds from Main Idea to Support-
ing Idea, then the correct option will have to deal with the main idea in some way: restating
or extending the main idea with modification or emphasis or providing another supporting
argument that would complete the theme. If the structure proceeds from Supporting Ideas
to Main Idea, then the correct option will be the logical extension of the sequence of ideas
in the paragraph.
3. Identify the tone of the paragraph. You need not assign a precise term to the tone (as
in Reading Comprehension questions) but it is certainly useful to identify the tone in terms
of general categories such as ‘positive’, ‘negative’, ‘neutral’, etc. So if the paragraph has a
generally positive tone, a negative option is unlikely to be the answer; a neutral or optimistic
paragraph is unlikely to end on a pessimistic note; a straightforward or sincere paragraph
will probably not be concluded with a sarcastic quip; etc.
4. Eliminate options that introduce points not mentioned in the paragraph. If most of the
options include a certain point, but only one does not, then that one is likely to be incorrect.
Look for options that are similar in structure to sentences in the paragraph, especially the
last sentence of the paragraph. In case of questions with similar options, concentrate on the
parts of the options that are different.
5. Beware of extreme options. This is not a hard-and-fast rule, but rather a rule of thumb.
Unless the paragraph itself deals with extremes or absolutes, you should be wary of choosing
options that involve an extreme stance for or against something, or deal in absolutes with
words such as ‘always’, ‘never’, ‘all’, ‘none’, ‘only’, etc. By the same token, avoid choosing
vague options that seem to be only tangentially related to the paragraph.
6. Pay attention to the last sentence of the incomplete paragraph. The answer will be a
sentence that follows this sentence, so the two should flow together naturally. If an option
contradicts the last sentence (without the use of connectors such as ‘but’, ‘however’, ‘on the
contrary’, etc.), it can be ruled out. On the other hand, even if an option does not seem to fit
with the rest of the paragraph, if it flows logically from the last sentence, it could conceivably
be the answer.
7. The correct option should ideally be such that it completes the theme of whole paragraph
– the question type is also called ‘Complete the Theme’. The point is that if a paragraph
makes multiple dissimilar points, you should look for an option that unites them as a group,
or points out the overarching theme running through all the points. Also, if an option starts
a new topic, it can be eliminated, as it does not complete the theme of the given paragraph.

44
Chapter VA
SUMMARY AND PARAGRAPH COMPLETION 4.2

CLASS EXERCISE

Directions: Four alternative summaries are given below each text. Choose the option that best
captures the essence of the text.

1. George R.R. Martin has always maintained that his fantasy novels have been influenced at
least as much by history and historical fiction as by the traditional epic fantasy of writers
like J.R.R. Tolkien. Aficionados know that his novels are loosely based on the Wars of the
Roses, a vicious series of battles of succession that took place in 15th-century England. It
would probably surprise several generations of British schoolchildren to learn that the dynastic
politics of the 1400s could be transformed into anything coherent, let alone entertaining. (‘It’s
worse than the Wars of the Roses!’ young Lucy Pevensie cries in dismay when someone tries
to explain a particularly complicated bit of Narnian history in Prince Caspian. She speaks for
many.)
1) George R.R. Martin’s fantasy novels have been inspired by the Wars of the Roses, 15th
century English battles of succession, though schoolchildren might not consider the topic
interesting.
2) George R.R. Martin’s fantasy novels are based on history as much as fantasy, though their
historical background – the 15th century English Wars of the Roses – is not particularly
entertaining.
3) British schoolchildren such as Lucy Pevensie would be surprised to learn that the Wars
of the Roses, 15th century English battles of succession, could be interesting, as shown
by George R.R. Martin in his fantasy novels.
4) Though George R.R. Martin’s fantasy novels are inspired by the Wars of the Roses, battles
of succession in 15th-century England, British schoolchildren do not find them entertaining.

2. In general, conifer trees are light lovers – an odd thought, as you wander through the green
shade of the redwood forest, or peer through the close-set boles of some spruce planta-
tion, or contemplate the long, dark winter months that the spruces and pines endure in the
subboreal forests of the Baltic or the truly boreal forests of Alaska and Canada, Scandinavia
and Russia.
1) It is odd that conifer trees love light, given that they generally grow in dark forests or
cold, dark places.
2) Conifer trees are generally light lovers, which may seem odd given that they grow in shady
forests and dark places.
3) Conifer trees are usually light lovers, even though they grow in shady forests and places
with long, dark winters.
4) Though conifer trees love light, they can sometimes even grow in dark forests in cold
places.

45
CATapult
VERBAL REASONING

3. There’s been a gradual, yet growing sense in the last year that the golden age of TV, so
named for the recent decade of dark, cable antiheroes and intricate serialization, is coming
to an end. I’ve seen this crop up in more and more places this summer. The primary idea
driving this is that Mad Men  will be halfway through its final season, and Breaking Bad will
be long over by the time I write a fall TV season preview next year, and those two shows
are some of the last remaining links to the age The Sopranos kicked off. (Indeed, a former
Sopranos writer created Mad Men.) There are still antihero-driven shows out there, from the
good – Boardwalk Empire – to the bad – Ray Donovan – but the dominant form of the TV
drama is slowly moving away from dark men in dark times doing dark things.
1) A new golden age of TV will come about when the current crop of dark and depressing
shows about antiheroes such as Breaking Bad and Mad Men end.
2) It will be the end of an era when the decade-long run of TV shows about antiheroes,
which started with The Sopranos, concludes with Breaking Bad and Mad Men.
3) A golden age of TV began a decade ago with shows about antiheroes like The Sopranos,
and will end when Breaking Bad and Mad Men are over.
4) There is a sense that the golden age of TV, characterized by shows about antiheroes, is
coming to an end, with shows like Breaking Bad and Mad Men getting over.

4. The human race is spread all over the world, from the polar regions to the tropics. The
people of whom it is made up eat different kinds of food, partly according to the climate in
which they live, and partly according to the kind of food which their country produces. In hot
climates, meat and fat are not much needed; but in the Arctic regions they seem to be very
necessary for keeping up the heat of the body. Thus, in India, people live chiefly on different
kinds of grains, eggs, milk, or sometimes fish and meat. In Europe, people eat more meat
and less grain. In the Arctic regions, where no grains and fruits are produced, the Eskimo
and other races live almost entirely on meat and fish.
1) Food eaten by people in different regions of the world depends on the climate and pro-
duce of the region, and varies from meat and fish in the Arctic to predominantly grains
in the tropics.
2) Hot climates require people to eat grains while cold regions require people to eat meat
and fish.
3) In hot countries people eat mainly grains while in the Arctic, they eat meat and fish be-
cause they cannot grow grains.
4) While people in Arctic regions like meat and fish and those in hot regions like India prefer
mainly grains, they have to change what they eat depending on the local climate and the
local produce. (Past CAT question)

46
Chapter VA
SUMMARY AND PARAGRAPH COMPLETION 4.2

5. Physically, inertia is a feeling that you just can’t move; mentally, it is a sluggish mind. Even

THEORY
if you try to be sensitive, if your mind is sluggish, you just don’t feel anything intensely. You
may even see a tragedy enacted in front of your eyes and not be able to respond meaning-
fully. You may see one person exploiting another, one group persecuting another, and not

PRACTICE EXERCISE CLASS EXERCISE


be able to get angry. Your energy is frozen. You are not deliberately refusing to act; you just
don’t have the capacity.
1) Inertia makes your body and mind sluggish. You become insensitive to tragedies, exploita-
tion and persecution because it freezes your energy and de-capacitates it.
2) When you have inertia you don’t act although you see one person exploiting another or
one group persecuting another. You don’t get angry because you are incapable.
3) Inertia is of two types — physical and mental. Physical inertia restricts bodily movements.
Mental inertia prevents mental response to events enacted in front of your eyes.
4) Physical inertia stops your body from moving; mental inertia freezes your energy, and
stops your mind from responding meaningfully to events, even tragedies, in front of you.
(Past CAT question)

6. Modern bourgeois society, said Nietzsche, was decadent and enfeebled – a victim of the
excessive development of the rational faculties at the expense of will and instinct. Against
the liberal-rationalist stress on the intellect, Nietzsche urged recognition of the dark myste-
rious world of instinctual desires – the true forces of life. Smother the will with excessive
intellectualizing and you destroy the spontaneity that sparks cultural creativity and ignites a
zest for living. The critical and theoretical outlook destroyed the creative instincts. For man’s
manifold potential to be realized, he must forego relying on the intellect and nurture again
the instinctual roots of human existence.
1) Nietzsche urges the decadent and enfeebled modern society to forego intellect and give
importance to creative instincts.
2) Nietzsche urges the decadent and enfeebled modern society to smother the will with
excessive intellectualizing and ignite a zest for living.
3) Nietzsche criticizes the intellectuals for enfeebling the modern bourgeois society by not
nurturing man’s creative instincts.
4) Nietzsche blames excessive intellectualization for the decline of modern society and sug-
gests nurturing creative instincts instead.
(Past CAT question)

47
CATapult
VERBAL REASONING

7. A safari holiday should be on everyone’s wish list of life’s greatest adventures. In the age
of Hemingway, going on a safari meant going up-country, setting off with a tent and a rifle
into the back of beyond where the wild things are. Now the day of the hunter is done. Guns
are out, cameras are in and ecotourism is the buzzword in the bush. People have always
wanted to reach out and touch the wild, to spend time in the sun and under the stars and
come face to face with Africa’s storybook animals, not behind bars but moving free as the
wind across the savannah. Doing all this will make it a holiday like no other.
1) Everyone should try to go on a safari holiday as it will be a unique adventure. Though the
meaning of ‘safari’ has changed over the years, the reasons for going on one have not.
2) A safari is the greatest adventure you can go on. You will get to shoot animals with your
cameras instead of rifles and will be able to enjoy nature and free wildlife in abundance.
3) All of us should go on a safari holiday once in our lives as it promises to be an excellent
opportunity to enjoy Mother Nature’s gifts in all their glory. We can capture wildlife in our
cameras and be close to nature.
4) The meaning of the word ‘safari’ has evolved over the ages as hunting is not legal any-
more. People are now happy to meet animals face-to-face and enjoy nature. This is what
makes a safari one of life’s greatest adventures.

8. Once upon a time, online news priorities were different. Rather than optimizing for ‘viral’,
sharable content—and all the unique visitors that followed—outlets optimized for page views.
They aimed to build engaged, repeat readers who spent long hours on the site. Instead of
readers who moved from website to website, never staying long—like so many cattle roaming
fallow hills—they tried to build return readers who felt rewarded by their long-term relation-
ship with the site. It was the age of blogs. Then search engine optimization and the big
social sharing networks came around. They could each deliver much more traffic than the
experienced, intrepid blog readership that preceded them, and news organizations began to
shift their focus away from rewarding repeat readers to browsers.
1) Earlier online news companies had different priorities: they aimed for page views optimi-
zation, not search engine optimization and they valued repeat readers, not browsers. The
shift happened as big social sharing networks gained popularity.
2) In the erstwhile age of blogs, online news companies optimized page views and valued
repeat readers. But with the advent of search engine optimization and big social networks,
which could provide much more traffic, news companies began shifting their focus to
browsers.
3) Before the era of search engine optimization and the big social sharing networks, in the
age of blogs, online news companies engaged and valued repeat readers. Now they value
browsers, who quickly move from website to website but increase the traffic on their sites.
4) After search engine optimization and the big social sharing networks came around, online
news companies shifted their focus from page views and regular readers to increased
traffic and browsers respectively.

48
Chapter VA
SUMMARY AND PARAGRAPH COMPLETION 4.2

9. Who’s the greatest American movie critic? A lot of folks probably would say Pauline Kael

THEORY
or David Bordwell or Manny Farber; some might argue for more academic writers like Linda
Williams, Stanley Cavell or Carol Clover. For me, though, it’s an easy question. The greatest
film critic ever is James Baldwin. His book-length essay, The Devil Finds Work, is one of

PRACTICE EXERCISE CLASS EXERCISE


the most powerful examples ever of how writing about art can, itself, be art. It’s a memoir
of Baldwin’s life watching, or influenced by, or next to cinema. It’s a critique of the racial
politics of American films. What makes the essay sing, and not sadly or in bitterness, is its
sheer power of description, and its audacity in treating self, society and art as a whole, to
be argued with and lived with and loved all at once.
1) For me, the greatest American movie critic is James Baldwin. His book-length essay, The
Devil Finds Work, is an audacious and powerful work of art about racial discrimination in
the American film industry. It is a very descriptive essay that unifies the self, society and
art even though they share a love-hate relationship.
2) His book-length essay, The Devil Finds Work, makes James Baldwin the greatest Ameri-
can movie critic for me, though other people may disagree. The essay critiques the racial
politics of American films and joins self, society and art to create a new whole world that
can be quite disquieting.
3) The greatest American movie critic is James Baldwin, though others may believe different-
ly. His book-length essay, The Devil Finds Work, is a memoir in which he critiques racial
politics in American films. It is a powerful and bold work of art which associates the self
with society and art though they may not share a peaceful relationship.
4) James Baldwin’s memoir, The Devil Finds Work, is a potent critique of racial discrimination
in the American film industry. It is a sheer work of art, with a strong narrative, that treats
as one the competing elements of self, society and art. Essays such as this make him the
greatest American movie critic for me at least.

10. Present college guidance for low-income students involves a host of tasks, including help-
ing students to identify college matches that offer robust support structures and adequate
funding; working with students as they draft and revise personal essays; arranging campus
visits; and providing individualized guidance to students and families through the financial
aid process. What schools don’t realise is that the college application season is a fraught,
vulnerable time for students and families, a moment of transition, which can catalyse crises
even for students with broad networks of support. Low-income students are making choices
with even higher stakes. They require not only better information, resources and advocacy
through the process, but also need mentors and counsellors who can take the time to help
them work through the complex personal struggles they face both before and after the ac-
ceptance letters arrive.
1) Applying to colleges is a very challenging and emotional process for poorer students.
Guidance counsellors should help students not only with the operational aspects of the
process but also through the psychological ones.
2) The college application season is a psychologically difficult time for students. Schools do
not realise this and thus end up focusing on providing process-oriented college guidance
instead of providing mentors that will help the students deal with their personal struggles.

49
CATapult
VERBAL REASONING

3) Underprivileged students have a harder time while applying to colleges than their richer
counterparts and thus need more emotional guidance from their counsellors. This has
been ignored by schools till now and they really need to step up to the challenge.
4) Schools are focusing on process-oriented college guidance for low-income students but
ignoring the psychological tumult that they go through while applying to colleges. Students
need counsellors who can help them get through this difficult time.

11. It’s hard out there for the 1 percent. Okay, that’s not true at all. But they think it is. If you
talk to people on Wall Street, most of them—even, in my experience, the ones shopping for
Lamborghinis—will tell you that they’re ‘middle class’”. Their lament, the lament of the HENRY
(short for “high-earner, not rich yet”), goes something like this. You try living on $350,000
a year when you have to pay taxes, the mortgage on the house in an elite zip code, the
nanny who knows how to cook ethnic cuisine, the private school tuition from pre-K on, the
appropriately exclusive vacation expenses, and so on. There just isn’t that much cash left
over each month once you’ve spent it all!
1) The richest 1 percent of the population does not feel rich after paying for its extravagant
lifestyle that others can only dream about.
2) Many of the richest 1 percent feel that they are not rich yet. They complain that there
isn’t much money left over after they have paid their bills for sustaining the lifestyle they
are accustomed to.
3) Even the richest 1 percent of the population, especially people on Wall Street, is not rich
enough to have adequate money left over once it has paid its monthly bills.
4) Many people that earn enough to be counted in the richest 1 percent feel that they are
still not rich enough as after paying taxes, mortgages, fees, employee and vacation ex-
penses, etc. they do not have money left to invest.

12. Greek and Shakespearian tragic drama exhibits human suffering, desolation, unmotivated ruin
and even physical horror (those blindings, mutilations and murders). Yet we do not leave the
playhouse (or the reading) crushed, despairing and hoping to avoid such display in future.
On the contrary. Our sensibility is complexly enriched and, somehow, rewarded. We register,
in Nuttall’s thoughtful phrase, ‘a strange sweetness of grief and fear’. Unquestionably, this
‘strange sweetness’ has positive elements. It can provoke, at great depths of consciousness
and understanding, a kind of dynamic peace, a tensed equilibrium.
1) Though tragic drama exhibits human suffering, we do not avoid their displays in the future.
But we are rewarded with a kind of peace that we do not otherwise experience.
2) Though tragic drama exhibits human suffering, we are enriched in some positive ways by
watching it. The mixture of fear and grief can arouse a dynamic equilibrium.
3) Tragic drama helps us in a positive way by giving us an equilibrium which we cannot avoid
in the future. So we are enriched by watching human suffering and horror.
4) Though tragic drama exhibits suffering, it has positive elements to it which we hope to
avoid in the future. It gives us inner peace which is enriching.

50
Chapter VA
SUMMARY AND PARAGRAPH COMPLETION 4.2

Directions: The last sentence is missing in each paragraph. Choose the most logical option to

THEORY
complete the paragraph meaningfully.

PRACTICE EXERCISE CLASS EXERCISE


13. Cultural evolution as a theory in anthropology was developed in the 19th century, and it was
an outgrowth of Darwinian evolution. Cultural evolution presumes that over time, cultural
change such as the rise of social inequalities or emergence of agriculture occurs as a result
of humans adapting to some non-cultural stimulus, such as climate change or population
growth.
1) Today, the theory of cultural evolution is an (often unstated) underpinning for other, more
complex explanations for cultural change.
2) For the most part anthropologists believe that social changes are not only driven by biol-
ogy or a strict adaptation to change, but by a complex web of social, environmental and
biological factors.
3) However, unlike Darwinian evolution, cultural evolution was considered directional, that is,
as human populations transform themselves, their culture becomes progressively complex.
4) Cultural evolution can often be a result of genetic constitution of people that varies from
one group of people to other.

14. The rise of China can often seem inevitable. It is the world’s most populous country, now
reclaiming its long-lost power. Its economy recently passed Japan’s as the second-biggest in
the world, leaving economists to debate whether China was on pace to overtake the United
States by the year 2025 or 2030.
1) Yet China has a raft of problems to address before it can really pride itself.
2) Yet China’s rise has been anything but inevitable.
3) Yet China might not be the role model that other developing countries should emulate.
4) Yet China needs to transform its human rights landscape before it can be truly envied.

15. We should make freedom of speech, freedom of the press, and academic freedom universal,
thereby guaranteeing that citizens can be informed and can exercise their right of political
supervision. These freedoms should be upheld by a Press Law that abolishes political re-
strictions on the press. The provision in the current Criminal Law that refers to ‘the crime
of incitement to subvert state power’ must be abolished.
1) We should end the practice of viewing words as crimes.
2) Subverting state power if state power becomes tyrannical is essential.
3) If people need to be incited to rise up against tyranny then so be it.
4) The press is that which mediates between the people and the state.

51
CATapult
VERBAL REASONING

16. With the exception of the imperial offspring of the Ming dynasty and the dauphins of pre-Rev-
olutionary France, contemporary American kids may represent the most indulged young people
in the history of the world. It’s not just that they’ve been given unprecedented amounts of
stuff – clothes, toys, cameras, skis, computers, televisions, cell phones, PlayStations, iPods.
They’ve also been granted unprecedented authority. Parents want their kids’ approval, a re-
versal of the past ideal of children striving for their parents’ approval. In many middle-class
families, children have one, two, sometimes three adults at their beck and call.
1) At the other end of the scale, the children of so-called ‘primitive’ peoples in Africa and
South America are among the most self-sufficient and mature kids in the world.
2) This is a social experiment on a grand scale, and one that it isn’t likely to end too well.
3) There is no doubt about it: American children are spoiled rotten, and nothing short of a
miracle is going to change this fact.
4) However, the adults’ indulgence of their progeny’s whims is somewhat understandable,
considering the great amount of competition that American children have to face.

17. Can small, temporary cultural cues really change the way we perceive the world? You may
think a culture would affect you only if you’ve spent some time immersed in it, but that’s
not the case. Culture influences you more easily than you realise. Thanks to the Internet and
the rise of global commerce and travel over the past 15 to 20 years, we’ve all become more
aware of foreign ideas and symbols. Our cities are peppered with the latter. And as you’re
exposed to them, you’re inevitably swayed.
1) When you’re in Chinatown or at an Asian supermarket, you cannot avoid noticing how the
culture is different from your own.
2) When you’re in Chinatown or at an Asian supermarket, you feel like trying out the local
cuisine.
3) When you’re in Chinatown or at an Asian supermarket, your worldview becomes more
Asian.
4) When you’re in Chinatown or at an Asian supermarket, you cling to your culture more
strongly to avoid getting influenced.

18. College provides the right platform to cultivate the ability to find a wide-angle view. Here,
for the first time in your life, you are faced with choices and decisions. What subject should
you specialise in? Should you participate in the forthcoming event? What topic should you
speak on in the class seminar? Should you apply for the project position advertised on the
notice board or just concentrate on preparing for the semester exams? Should you look at
research and academics or go for that super-paying corporate job?
1) An intelligent person will ask himself “what will prove to be best for me in the long-run?”
2) Use these opportunities to practise the skill of getting a broader perspective.
3) While making these decisions, do not shy away from that which makes you uncomfortable.
4) If you carefully consider these questions, you will be able to introspect and know yourself
better.

52
Chapter VA
SUMMARY AND PARAGRAPH COMPLETION 4.2

19. Some people are so rude. Really, who sends an email or text message that just says “Thank

THEORY
you”? Who leaves a voice mail message when you don’t answer, rather than texting you? Who
asks for a fact easily found on Google? Don’t these people realise that they are wasting your
time? Of course, some people might think me the rude one for not appreciating life’s little

PRACTICE EXERCISE CLASS EXERCISE


courtesies.
1) But many social norms just don’t make sense to people drowning in digital communication.
2) They would be wrong. I appreciate politeness, but it is not required in non-verbal com-
munication.
3) But in this dog-eat-dog world, courtesies do not matter; rather time is of the essence.
4) Haven’t we all realised that life is too short to waste on being courteous?

20. Drugs that allow for mood manipulation are already on the market. Prozac and chemicals like
oxytocin have the ability to make some people calmer, more empathetic, and more altruis-
tic. Calm, empathetic, and altruistic people are far more likely to act morally than anxious,
callous, and selfish people. But does that mean mood manipulation is going to let us force
people to be moral?
1) And if it does, will it make more people moral?
2) And if it does, would more people like to be moral?
3) And if it does, will society as a whole benefit from it?
4) And if it does, is it moral to force people to be moral?

21. A social network is crucially different from a social circle, since the function of a social circle
is to curb our appetites and of a network to extend them. Everything once inside is outside,
a click away; much that used to be outside is inside, experienced in solitude. And so the
peacefulness, the serenity that we feel away from the Internet, has less to do with being no
longer harried by others than with being less oppressed by the force of your own inner life.
1) Your social network, unlike a social circle, endangers your calm.
2) It is thus time to choose between your social circle and your social network.
3) Shut off your computer, and your self stops raging quite as much or quite as loud.
4) Technological disruption has now entered our private lives.

22. By denigrating “foodwork”—everything involved in putting meals on the family table—we


have unthinkingly wrecked one of the nurseries of democracy: the family meal. It is at “the
temporary democracy of the table” that children learn the art of conversation and acquire
the habits of civility—sharing, listening, taking turns, navigating differences, arguing without
offending—and it is these habits that are lost when we eat alone and on the run.
1) Civility and eating are not very far off from each other.
2) Civility is not needed when one is by oneself.
3) To be civilized is to be a part of civilization.
4) It is more than food that is at stake at the dinner table.

53
CATapult
VERBAL REASONING

23. “Gentrification”: the term evokes the political and mental life of two generations of city-dwell-
ers. According to one interpretation, it was the forced displacement of the urban working
class by mobile, college-educated professionals. According to another, it was the restoration
of city life in the imagination of a West that had supposedly given it up for suburban sprawl.
An entire understanding of what cities were for and where they were going was bound up
in the ambiguous word.
1) Now it has altogether disappeared from the urban lexicon.
2) It is only after decades that its varying interpretations have been reconciled.
3) Cities, in the meantime, went on proliferating, oblivious to the meaning of the word.
4) All the energies of urban thought went into debating its meaning.

24. In Bolaño, literature is a helpless, undignified, and not especially pleasant compulsion, like
smoking. At one point you started and now you can’t stop; it’s become a habit and an iden-
tity. Nothing is so consistent across Bolaño’s work as the suspicion that literature is chiefly
bullshit, rationalizing the misery, delusions, and/or narcissism of various careerists, flakes,
and losers. Yet Bolaño somehow also treats literature as his and his characters’ sole excuse
for existing. This basic Bolaño aporia—literature is all that matters, literature doesn’t matter
at all—can be a glib paradox for others.
1) He seems to have meant it sincerely, even desperately, something one would feel without
knowing the first thing about his life.
2) He seems to have worn this paradox lightly on his sleeve, being able to produce literature
instead of getting caught in its contradictions.
3) He seems to have found a resolution to this paradox; treading along the border where
the meaning and meaninglessness of literature intersect.
4) He turned this very paradox into the raison d’être of his literature.

25. The strength of An Ideal Husband is in its dialogue. Despite some incidents of old fashioned
blackmailing, all players are true ladies and gentlemen. Even when insulting each other face
to face, an air of formality is maintained and it is in this subtle humour that the movie pre-
vails. There are no physical threats and no outwardly lewd behaviour.
1) The ornate detail of old English houses comes alive throughout the movie sets the salon,
the conservatory, the library.
2) A simple biting remark is all the retaliation anyone needs.
3) The story is somewhat Shakespearean.
4) It makes for a great test for the range of one’s sense of humour.

54
Chapter VA
SUMMARY AND PARAGRAPH COMPLETION 4.2

PRACTICE EXERCISE-1

Directions: Four alternative summaries are given below each text. Choose the option that best
captures the essence of the text.

1. The film actors’ creation is by no means all of a piece; it is composed of many separate per-
formances. In particular, the presentation of an event that, on the screen, unfolds as a rapid
and unified scene is a sequence of separate shootings which may take hours at the studio.
Thus a jump from the window can be shot in the studio and the ensuing flight, can be shot
weeks later when outdoor scenes are taken. Let us assume that an actor is supposed to be
startled by a knock at the door. If his reaction is not satisfactory, the director can resort to
an expedient: when the actor happens to be at the studio again, he has a shot fired behind
him without his being forewarned of it. The frightened reaction can be shot now and be
cut into the screen version. Nothing more strikingly shows that art has left the realm of the
‘beautiful semblance’ which, so far, had been taken to be the only sphere where art could
thrive.
1) The film actors’ creations are varied. Their sequences are shot separately though they are
seen as one shot on screen. Thus, films have an unreal appearance, which was unimag-
inable till now.
2) The scene that we see as a unified sequence is shot over different locations and periods
of time. The actors’ reactions can be changed as per the shot’s requirements. Thus the
truthful appearance of art has changed forever.
3) Film scenes that may look unified are actually shot over different locations and time pe-
riods. As film actors’ emotions can be manipulated, it goes on to show that art need no
longer resemble reality.
4) The unreal appearance of art has come about due to films, as film actors can change
their reactions as per the requirement of the scene. Films scenes are also spread over
different times and places, though we see each of them as a continuous scene.

2. In principle, a work of art has always been reproducible. Man-made artefacts could always
be imitated by men. Mechanical reproduction of a work of art, however, represents something
new. Historically, it advanced intermittently and in leaps at long intervals, but with accelerat-
ed intensity. The Greeks knew only two procedures of technically reproducing works of art:
founding and stamping. Bronzes, terra cottas and coins were the only art works which they
could produce in quantity. All others were unique and could not be mechanically reproduced.
With the woodcut, graphic art became mechanically reproducible for the first time, long before
script became reproducible by print. The enormous changes which printing, the mechanical
reproduction of writing, has brought about in literature are a familiar story. However, within
the phenomenon which we are here examining from the perspective of world history, print
is merely a special, though particularly important, case.

55
CATapult
VERBAL REASONING

1) Art has always been reproducible and mechanical reproduction allows reproducing it in
large quantities. The Greeks knew only two ways of producing art. Then came woodcut
and printing. The latter is most important from the point of view of world history.
2) While the Greeks knew only two ways of reproducing art, mechanical reproduction of art
increased over time. Though there were other art works like bronzes and terracottas that
could be reproduced, printing was one of their most important discoveries as it brought
about enormous changes in literature.
3) Mechanical reproduction of art has increased exponentially and steadily over time. Through-
out world history, printing is an important case of mechanical reproduction as it brought
about significant changes in literature, though woodcut graphic art came before printing.
4) Mechanical reproduction of art is relatively new but it has advanced faster over time,
though sporadically. The Greeks knew only two ways of doing so. Though woodcut was
used to reproduce graphic art before printing, the latter is more significant as it brought
about great changes in literature.

3. Translation of poetry is believed to be impossible, for any unfaithful elements would be tak-
en as failure, be it content or form. The arguments include linguistic elements and cultural
elements. Most importantly, the myth of untranslatability looks upon poetry as beauty itself
which is untouchable for once it is touched it is destroyed. But as translation of poetry has
never been stagnant though sometimes vigorous and sometimes not, there is strong evidence
in both translation history and present day practice that poetic translation, a literary form as
distinguished from fiction, drama and prose, is translatable.
1) Though poetry cannot be translated as its beauty is untouchable, poetic translations have
been practised as a different literary form.
2) Even with the problems of untranslatability of poetry, it has been translated in fits and
bounds throughout history as a separate literary form.
3) Due to its linguistic and cultural elements, poetry is believed to be untranslatable. But,
poetic translations have been carried out as a separate literary form.
4) The translation of poetry is a separate literary form. Though it is untranslatable due to
its linguistic and cultural elements, it has been practised.

4. The story of Confucianism does not really begin with Confucius, nor was Confucius the
founder of Confucianism in the same way that Buddha was the founder of Buddhism. Rather,
Confucius considered himself a transmitter who consciously tried to retrieve the meaning of
the past by breathing vitality into seemingly outmoded rituals. Confucius’ love of antiquity
was motivated by his strong desire to understand why certain rituals, such as the ancestral
cult, reverence for Heaven, and mourning ceremonies, had survived for centuries. He had
faith in the cumulative power of culture. Confucius’ sense of history was so strong that he
saw himself as a conservationist responsible for the continuity of the cultural values and the
social norms that had worked so well for the civilization of the Chou dynasty.

56
Chapter VA
SUMMARY AND PARAGRAPH COMPLETION 4.2

1) Confucius thought of himself as a conservationist and transmitter responsible for the

PRACTICE EXERCISE CLASS EXERCISE THEORY


continuity of culture and rituals that had worked for the Chou dynasty’s civilization.
2) Confucius was not the founder of Confucianism like Buddha was of Buddhism. He was a
historian who transmitted the ancestral culture of the Chou dynasty.
3) Confucius was a transmitter of history. He conserved the cultural values which had become
outmoded and also preserved the Chou dynasty’s civilization.
4) Confucius loved antiquity and this love led him to bring to life the cultural values and
social norms of the outmoded rituals.

5. The Vakyapadiya expounds on the idea that the spoken word appears to have differentiation
but it really does not - it exists in the mind of the speaker as a ‘unitary gestalt’, or sphota.
Bhartrhari does not subscribe to this idea fully; he says that the ‘spoken words serve only
as the stimulus to reveal or uncover the meaning which was already present in the mind of
the hearer’. Bhartrhari explains this idea by suggesting that the mind understands sound in
two aspects: word-sound (dhvani) and word-meaning (artha). Sphota is the undifferentiated
whole, of which dhvani and artha are two sides of the same coin. There is a deep spiritual
connection between the communicable word and the thought that inspired it; they develop
simultaneously according to this philosophy.
1) The Vakyapadia and Bhartrhari relate to the spoken word in different ways. The former
does not divide sphota but the latter mentions that it is made of two distinct parts - dh-
vani and artha - both necessary to understand sphota.
2) The Vakyapadia universalizes sphota, while Bhartrhari distinguishes it as dhvani and artha.
The deep spiritual connection between these two parts developed simultaneously and is
necessary to understand the spoken word.
3) Bhartrhari considers the spoken words as a stimulus with two aspects - dhvani and artha
- which has a deep spiritual connection with the communicable word. The Vakyapadia
does not make this distinction and considers it to be a unitary pattern.
4) The Vakyapadia and Bhartrhari distinguish between spoken word and communicable word.
While the former considers it as a whole meaning understood by the speaker, the latter
believes that it has two distinct parts with a deep spiritual connection.

6. Sometimes called action painting, abstract expressionism was influenced by European art
movements such as surrealism and cubism, and rejected the aesthetics of social realism.
Abstract expressionism was not distinguished by a cohesive style and the painters associated
with the movement rejected the idea that they were a ‘school’ of art. Despite the variety of
styles and techniques, there were some unifying characteristics to abstract expressionism.
The work was mostly but not exclusively non-representational and typically features strong
colours and large canvases. Abstract expressionism celebrated the very act of painting, free-
dom of expression and aspired to convey pure emotion visually.

57
CATapult
VERBAL REASONING

1) Abstract expressionism was influenced by European art movements but rejected their aes-
thetics of social realism. It was non-representational and celebrated painting the freedom
of the painters. They expressed it on large canvases with strong colours.
2) Abstract expressionism celebrated the freedom of the painters from the aesthetics of
social realism and conveyed this emotion visually. Though it was influenced by European
art movements, it had its own distinct styles and techniques with some unifying charac-
teristics.
3) Abstract expressionism was influenced by European art movements. It is not distinguished
by a single style but has some unifying characteristics. It is more about conveying emo-
tions visually rather than being restricted by aesthetics of social realism.
4) Abstract expressionism was influenced by European art movements but did not restrict
itself to social realism. It was meant to convey emotion on large canvases with strong
colours with unifying techniques of style.

7. Stipple launched in 2010, when founders Rey Flemings and Michael Dungan set out to pro-
vide a solution to the Internet’s ‘orphaned image problem’. The fact that when an image is
moved from one location to another on the Web, it loses all of its metadata, or information
associated with that image. That’s a significant problem considering that images account for
two-thirds of all content on the Web, that 70% of all Facebook activity is image-based, and
that articles with images are viewed 94% more than articles sans image. Stipple not only
enables information associated with an image to travel with it across the Web, but it also
enables users to access additional content, such as a YouTube video or links to a Facebook
page or Twitter account, by simply rolling their mouse over the image.

1) Stipple provided a solution to the loss of metadata of images as they account for a ma-
jority of Web content. Thus, the information associated with the image is accessible to
viewers.
2) Stipple has tried to solve the problem of losing the metadata of images since they ac-
count for two-thirds of all content on the Web. It also allows users to access additional
information.
3) Stipple succeeded in solving the problem of loss of metadata of images. It also enabled
viewers to get additional information about these images on their Facebook and Twitter
accounts.
4) Stipple was successful in stopping the loss of metadata and giving viewers additional
content about images that account for nearly two-thirds of the content on the Web.

8. The Fall is famous for more than its interesting narrative technique. For one, it was written
by Albert Camus, a French thinker known for his philosophy of the absurd, a close cousin
to existentialism, and his frenemy status with Jean-Paul Sartre, another French philosopher
of the mid-1900s. Camus is most famous for three big novels: The Stranger, The Plague and
The Fall. Through these three novels, Camus establishes and explores several ideas of his
philosophy. It’s fascinating to see the way his ideas grow over time. In many ways, The Fall
can be seen as the high point of Camus’s thinking. His ideas increase in complexity over
the course of his novels. You’ll probably notice that interpreting and analysing The Plague is
more difficult than taking on The Stranger, and likewise, The Fall is more challenging than
the works which precede it. But don’t take our word for it.

58
Chapter VA
SUMMARY AND PARAGRAPH COMPLETION 4.2

1) Albert Camus was a French thinker known for existentialism and also his three fa mous

PRACTICE EXERCISE CLASS EXERCISE THEORY


novels which are known for their complex philosophy.
2) Albert Camus was a French thinker who was famous for three novels in which he explores
his ideas of the philosophy of the absurd in an increasingly complex manner.
3) The three novels of Albert Camus, known for his philosophy of the absurd, are a gateway
to his increasingly complex philosophy of existentialism.
4) Albert Camus was the trendsetter of the philosophy of the absurd which can be read in
his three famous novels in increasing complexity.

9. The Large Hadron Collider is, essentially, a super-microscope that will use the largest ener-
gies ever generated to examine trillionth-of-a-millimetre bits of matter and record evanes-
cent blinks of energy that last for only trillionths of a trillionth of a second. It’s also a kind
of time machine, in the sense that it will reproduce the conditions that prevailed 14 billion
years ago, giving scientists a look at the universe as it existed a trillionth of a second after
the big bang. The goal – and it’s a hope, a dream, a set of strong suspicions, rather than a
certainty – is to achieve a deeper, better, truer understanding of the fundamental structure
and nature of existence.
1) The Large Hadron Collider is a time-machine which will give scientists a look at the con-
ditions of the universe a trillionth of a second after the big bang and help to understand
its structure.
2) The Large Hadron Collider is essentially a super-microscope, which will enable scientists to
learn more about the fundamental structure and nature of the existence of the universe.
3) The Large Hadron Collider has cutting edge intensity which will allow it to fulfil the goal
of the scientists to understand the fundamental structure and nature of the universe.
4) Scientists believe that the Large Hadron Collider would certainly give them a deeper un-
derstanding of the fundamental structure and nature of the universe.

10. Depending on where you are on the spectrum of epicurean cultural politics, you may con-
sider Whole Foods to be a righteous grocer or a cynical con, a prod to self-improvement
or a gateway to decadence, a neighbourhood boon or a blight, a force for social good or a
place to pick up chicks. To the likes of Wal-Mart and Costco, it has been an impetus to carry
healthier, more judiciously sourced food. To small neighbourhood natural- or gourmet-food
shops, it has sometimes been an impetus to go out of business. It has enabled organic and
artisanal producers to scale up, and put pressure on the giants to at least pretend that they
are scaling down. It has less than a one-per-cent share of the American grocery market, yet
it has unquestionably transformed the way Americans produce, buy and eat food.
1) Whole Foods has captured the grocery market and changed the way Americans produce,
buy and eat food. Depending on where you are in the food chain supply chain, you may
have to scale down or go out of business.
2) Whole Foods has changed the epicurean cultural politics of America and transformed the
way Americans source their food. It gave the big business houses a reason to scale down
(or pretend to do so) while giving an impetus to the neighbourhood shops to improve or
shut shop.

59
CATapult
VERBAL REASONING

3) The Whole Foods business has forced the epicurean cultural politics to change its perspec-
tive. It has changed the dynamics of the grocery market (either by scaling up or down)
and transformed the way Americans produce, buy and eat food.
4) Whole Foods was instrumental in changing the grocery market and transformed the way
food is sourced. It represented a boon or a bane depending on whether you are a small
neighbourhood shop or a big business house.

11. The cradle of civilization is a term which refers to the geographical areas home to the first
recognizable civilizations. Arguably, the first civilization was the Sumerian civilization, in modern
day Iraq, which developed the true system of writing in the third millennium BC. The Sumerian
civilization was among the first to build permanent settlements (as opposed to the hunter
gatherer lifestyle typical at the time) in order to allow extensive agricultural practices, such
as irrigation. This along with the use of pottery, the adoption of the wheel and development
of copper tools highlighted the shift from the Neolithic period to the Chalcolithic period.
1) Only the use of the copper tools in the Sumerian civilization allowed humanity to shift
from the Neolithic to the Chalcolithic period. This civilization put an end to the nomadic
way of life and developed the system of writing.
2) The Sumerian civilization is credited for starting the Chalcolithic period. It is the first
civilization to use copper in its day-to-day life. The development of writing enabled its
development and allowed permanent settlements to flourish.
3) The Sumerian is the first civilization that built permanent settlements and thus put an end
to the people’s nomadic lifestyle. It also used pottery, the wheel and copper tools, thus
starting the Chalcolithic period.
4) The Sumerian civilization is considered to be the first civilization to build permanent set-
tlements, practise irrigation and develop writing. It marked the change from the Neolithic
to the Chalcolithic period due to the use of copper tools, pottery and the wheel.

Directions: The last sentence is missing in each paragraph. Choose the most logical option to
complete the paragraph meaningfully.

12. The term ‘directly competitive or substitutable’ describes a particular type of relationship
between two products, one imported and the other domestic. It is evident from the wording
of the term that the essence of that relationship is that the products are in competition. The
context of the competitive relationship is necessarily the marketplace since this is the forum
where consumers choose between different products.
1) Competition in the market place is a dynamic, evolving process.
2) Particularly in a market where there are regulatory barriers to trade or to competition,
there may well be latent demand.
3) It is also true that consumer responsiveness to products may vary from country to country.
4) Indeed, products which are competitive, may not be actually competing with each other
in the marketplace.

60
Chapter VA
SUMMARY AND PARAGRAPH COMPLETION 4.2

13. After September 11th, as it became apparent that the United States would bomb Afghanistan,

PRACTICE EXERCISE CLASS EXERCISE THEORY


an open letter written by an Afghan appeared on the Internet. It pleaded with Americans to
realize that Afghanistan was already a devastated country.
1) The Kite Runner portrays the Afghans as an independent and proud people.
2) The letter ends with a note of optimism.
3) It needed food, not vengeance; sympathy, not hate.
4) It was a hopeless place.

14. While there is no institutional segregation, Jews and Arabs have chosen to live separately in
all but a handful of cities. Israelis all recognize that Arab villages have historically received
less funding than Jewish areas and this has affected the quality of Arab schools, infrastruc-
ture and social services.
1) Israeli Jews and Arabs have surprisingly little contact with each other.
2) Arabs are also under-represented in higher education and most industries.
3) Arabs have adopted Hebrew as a second language and Israeli culture as an extra layer
in their lives.
4) Arabs do have an advantage obtaining some jobs during the years Israelis are in the
military.

15. The environment in which people work has a tremendous effect on their level of pride for
themselves and for the work they are doing. Do everything you can to keep your equipment
and facilities up to date. Even a nice chair can make a world of difference to an individual’s
psyche. Also, if possible, avoid overcrowding and allow each employee his or her own per-
sonal space, whether it is a desk, a locker, or even just a drawer.
1) Although employees will never feel a great sense of motivation or satisfaction due to your
policies, you can decrease dissatisfaction in this area by making sure your policies are
fair and apply equally to all.
2) If you already have a manual, consider updating it (again, with staff input).
3) To decrease dissatisfaction in this area, you must begin by making wise decisions
when you appoint someone to the role of supervisor.
4) If you’ve placed your employees in close quarters with little or no personal space, don’t
be surprised if there is tension among them.

16. In her best-selling book The Beauty Myth, the American feminist Naomi Wolf famously
compared the contemporary ideal of beauty to the medieval torture device called the Iron
Maiden, which enclosed victims in a spike-lined box shaped like a woman. Wolf pointed out
how, like the Maiden, the ideal of beauty enforces conformity to a rigid shape.
1) Fifteen years on, sadly, it continues to hold true.
2) It must be a fabulous life.
3) There is a supposed link between pointless advertising and products that sell.
4) Both, the ideal and the device, cause their victims a lot of suffering.

61
CATapult
VERBAL REASONING

17. People kill each other over diamonds; countries go to war over oil. But the world’s mostex-
pensive commodities are worth nothing in the absence of water. Fresh water is essential
for life, with no substitute.
1) Hence, there is so much of it found all over the world.
2) Yet we find that water is mostly low priced due to its relative abundance.
3) Knowing this, it is baffling that countries go to war over oil.
4) Isn’t it evident then, what people and countries should be trying hard to acquire?

18. Any article on fear or phobia will inform you that in case of social phobia a person feels
awkward when present in a social get-together. The person feels too shy to eat, drink or
even to speak to anyone while present among several individuals. Agoraphobia is the most
common instance of feeling afraid particularly when one has to depart from familiarities and
settle among a group of unknown individuals.
1) A person suffering from agoraphobia finds it difficult to speak to people around and
get along with those individuals.
2) The fear aggravates if the person is asked questions or becomes the centre of attention.
3) A person suffering from agoraphobia is cursed for life; without hope of ever making friends.
4) The fear goes away in moments and the person soon becomes comfortable.

19. Although the vast majority of people around the world are raised within the structure of an
established religion, there are few who can honestly say that they are completely satisfied
with all that their religion teaches. People who seek to understand the world around them
inevitably come into contact with people whose religious beliefs are entirely different from
their own.
1) Those who are narrow-minded strongly criticize such people.
2) They think of such people as agnostics.
3) They deem these people to be either misguided or fanatical.
4) In time many such people also try and understand alternative belief systems.

20. Mobile phones are becoming an increasingly popular way to make all sorts of payments.
In America, fans of the Atlanta Hawks have been testing specially adapted handsets linked
to their credit cards to enter their local stadium and to buy refreshments. In fact, you can
comfortably pass the entire day in many European cities without carrying cash, credit or debit
cards by paying for everything, including consumer goods, with a mobile phone.
1) It is estimated that the worldwide payments using mobile phones should grow at a easy
300% over the next 3 years.
2) Most payments by mobile phones are considered safer than those by cash by users.
3) Studies show that the average American or European consumer is more likely to carry his
mobile phone than he is to carry cash.
4) To live a credit card-free life is yet a distant dream!

62
Chapter VA
SUMMARY AND PARAGRAPH COMPLETION 4.2

21. The environmentalist’s vision is a world with less of carbon dioxide and more windmills,

PRACTICE EXERCISE CLASS EXERCISE THEORY


forests, smart grids and the like. In the distant future – in 2050 – the world is expected to
look very different indeed. Overall, carbon dioxide emissions would be less than half of what
they are today and in today’s developed countries they would have fallen by 90% or so.
1) This means that the use of automobiles should be reduced as it decreases the carbon
dioxide emission.
2) This obviously shows that environmentalists are quite far from reality!
3) This means that our goal should be oriented towards making the earth greener by planting
more trees.
4) This means that we could expect entirely new infrastructure and technologies, and perhaps
an entirely new lifestyle, too.

22. On a video monitor, one can see a number of animals whose faces bear a mark of fear
and large unhealed scars. The animals appear in a 21-minute exposé called Cutting Edge,
shot in an International University by animal rights activists campaigning for the Abolition of
Vivisection in research. The animals had been deliberately put through pain in experiments
meant to simulate the symptoms of stroke and Parkinson’s disease.
1) It is the kind of journalistic manoeuvre that has the power to move the masses from
ignorance to action in a matter of minutes.
2) Cutting Edge is an appeal to the government for continued research in finding drugs for
stroke and Parkinson’s disease.
3) The coverage raises serious doubts about whether laws related to ethical treatment
of animals even exist.
4) This video was secretly filmed in 2001 and later received generous critical acclaim from
the film community and the audience.

Directions: A sentence is missing in each paragraph given below. Choose the most logical option
to complete the paragraph meaningfully.

23. India seems to have acquired the dubious distinction of being the front-runner in regressive
trends, be it child labour or underage marriage. Slavery is the latest addition to this list. Al-
though the practice had been abolished long ago, a new study shows that an estimated 29.8
million people live in slavery around the world. _____________________. Fourteen million
Indians, the highest number in the world, live in conditions of slavery.
1) This number isn’t expected to diminish any time soon.
2) And nearly half this number is in India.
3) Despite enacting several legislations to curb this menace, India still has a long way to go
in eradicating slavery.
4) Many countries around the world are witnessing the resurgence of slavery.

63
CATapult
VERBAL REASONING

24. A gharana indicates a comprehensive musicological ideology. The concept gained curren-
cy in the nineteenth century when the royal patronage enjoyed by performers weakened.
_______________ _______________. To retain their respective identities, they fell back on
the names of the regions they hailed from. Therefore, even today, the names of many ghara-
nas refer to places; some of the gharanas well known for singing khayals are Agra, Gwalior,
Patiala, Kirana, Indore, Mewati, Sahaswan, Bhendibazar and Jaipur.
1) Performers were then compelled to move to urban centres.
2) Due to this, performers changed their style to suit audiences unlettered in music.
3) This marked the beginning of a low phase in Indian classical music.
4) Performers found it very difficult to adapt to the changed scenario.

25. The postcards were the first pieces of mail Ruma had received from her father.___________
____________________________. In her thirty-eight years, she’d never had the opportunity
to reply. His trips were brief enough so that there was no time for Ruma to write back. Be-
sides, he was not in a position to receive any mails, even from family.
1) “Be happy, love Baba,” he signed them, as if the attainment of happiness were as simple
as that.
2) Her father’s penmanship was small, precise and slightly feminine.
3) It was a one-sided correspondence.
4) The cards were addressed to Ruma; her father never included her husband or son.

64
Chapter VA
SUMMARY AND PARAGRAPH COMPLETION 4.2

PRACTICE EXERCISE-2
Directions: Four alternative summaries are given below each text. Choose the option that best
captures the essence of the text.

1. At first glance, the disciplines of philosophy and music appear to have little in common.
Whereas philosophers strive to create clear arguments using definable concepts, musicians
compose melodies and symphonies that resist linguistic understanding. Whereas philosophy
is hyper-verbal, music is predominantly non-verbal. When commenting on music, philosophers
often speak in roundabout or simplistic ways (e.g. Schopenhauer once wrote that ‘music is
the answer to the mystery of life’). Likewise, many composers explicitly allude to philosophical
texts but few musicians would say that their philosophy-inspired compositions have added to
the substantive philosophical arguments or discussions surrounding the thinker’s work. Despite
these difficulties, philosophers and musicians have long studied each other’s disciplines for
both inspiration and new understanding.
1) Though there is nothing in common between philosophy and music, many musicians have
worked with philosophers and vice-versa to understand the nuances between these two
subjects.
2) Though philosophy is non-verbal and music is verbal, they have some common ground
which has been studied by musicians and philosophers, for better understanding as well
as inspiration.
3) Though some musicians have used philosophy in creating their music, the same cannot
be said about philosophers using music to create their compositions. Many philosophers
dismiss music as simplistic.
4) Though philosophy and music have little in common and there are difficulties in relating
the two fields, philosophers and musicians have studied each others’ disciplines for better
understanding and inspiration.

2. As for defining an American compositional style, Dvorak looked to the musical materials of
indigenous Native American populations and the spirituals and plantation songs of the Afri-
can-Americans. Unfortunately, this strategy, which had worked so well in his native country,
was not well received in America. The young United States was a country built on geograph-
ic and political unity, but was ethnically and culturally quite diverse. Many who considered
themselves American did not consider the music of Native Americans and slaves to be their
music, and were opposed to having these materials form a new American voice.
1) The songs of the Native Americans and African-Americans formed the basis of Dvorak’s
compositions. But he was opposed by the Native Americans in the United States.
2) Dvorak took inspiration from the Native American and African-American music for his
compositions. However, he was opposed by the culturally diverse Americans because of
his plagiarism of American music.
3) The songs of the Native Americans and the African-Americans were considered to be the
voice of the new America. Thus, when Dvorak used this music in his compositions, it was
opposed by the diverse Americans.

65
CATapult
VERBAL REASONING

4) Dvorak looked at the native music and songs of the Native Americans and African-Ameri-
cans for defining American style music. However, due to the cultural diversity of the United
States, many did not consider this music as the new American voice.

3. The eighteenth century was a period of economic growth as a result of new methods for
agricultural production. It was a period of individual rights, social reform and universal ed-
ucation. Scientific developments led to the idea that human reason and the observation of
nature could reveal knowledge of our world. A growing middle class with a better education
meant a larger audience for new scientific theories, literature and the arts, which had to be
addressed differently than as by the previous century’s connoisseurs. This, combined with a
cosmopolitan international culture and a preference for naturalness of expression, led to a
very different style of music.
1) The eighteenth century resulted in new methods of agriculture which in turn led to uni-
versal education. Scientific developments as well as the increasing educated middle class
forced the arts like literature, music, etc. to change their target audience.
2) The eighteenth century witnessed a lot of social upheavals. Chief amongst them was the
rise of the educated middle class. Thus, all the fields of art had to address an audience
different from the connoisseurs of the previous centuries.
3) The eighteenth century led to various reforms in science, literature, arts, etc. Due to the
increase in the number of middle class people, music was also transformed as their tastes
were different than those of the connoisseurs.
4) The eighteenth century was a period of growth and change. Scientific developments and
the increasing number of educated middle class people led to a change in audience tastes
for scientific theories, literature, arts, etc., resulting in a different style of music.

4. Since the 1980s, Smetacek has taken regular expeditions from his home port of Bremer-
haven to the Southern Ocean aboard the sturdy icebreaker Polarstern. He goes to study the
plankton that fill the sea from top to bottom, extending even into the sediments of the sea
floor. Plankton is our planet’s most prolific life form, and the food it generates makes up the
base layer of the global food chain. The variety of shapes among plankton species shames
plants on land, showing more range in size than the difference between moss and redwood
trees. There are more plankton cells in the sea than our current count of stars in the entire
universe. Indeed, it is precisely this abundance that leads Smetacek to suspect that plankton
could be used to change Earth’s environment.
1) Smetacek studied the various species of plankton and came to the conclusion that plankton
can be used to change the Earth’s environment.
2) The fact that plankton is at the base of the global food chain has the potential to change
the Earth’s environment, according to Smetacek.
3) Plankton is at the bottom of the global food chain. Due to its abundance and diversity,
Smetacek believes that it is possible to change the Earth’s environment with its help.
4) Plankton is the most prolific life form on Earth, and is at the base of the global food
chain. According to Smetacek, it is therefore ideal for changing the Earth’s environment.

66
Chapter VA
SUMMARY AND PARAGRAPH COMPLETION 4.2

5. Although the word ‘rights’ appears in ‘natural rights’ and ‘human rights’, the two concepts

PRACTICE EXERCISE CLASS EXERCISE THEORY


are profoundly different. One seeks to restrict the power of government and the other seeks
to expand it. Whereas natural rights seek freedom from the state, human rights seek the
state’s protection and assistance. More importantly, and this is the point rarely appreciat-
ed by today’s human-rights industry, whereas natural rights made democracy possible, the
human-rights discourse is securing democracy’s emasculation. Democracy can only thrive if
three conditions are satisfied: (a) man is treated as rational, (b) the state is restrained and
(c) politics is freed of legal constraints. Whereas the natural-rights advocate champions each
condition, the human-rights advocate assumes the first condition is impossible and the next
two are undesirable.
1) The two concepts of natural rights and human rights are inherently different ideologies.
The former make democracy possible while the latter’s discourse tries to weaken it.
2) Natural rights and human rights are different concepts. Advocates of the former consider
that all the three conditions needed for democracy are possible while those of the latter
think they are impossible.
3) Natural rights strengthen democracy while human rights weaken it. Advocates of the for-
mer seek freedom from the state while those of the latter think it’s impossible to obtain
freedom.
4) Natural rights seek freedom while human rights protect it. Natural rights can fulfil all the
conditions of democracy while human rights emasculate it.

6. The Government of Bombay introduced the 1878 Act partly in response to criticisms of
government alcohol policy by temperance advocates. These activists argued that colonial
excise policy had prompted an increase in alcohol production and that constitutional reform
was needed to curb this. However, the dilemma for the colonial state was that excise rev-
enues from the sale of liquor production and sale licenses represented a significant part of
the revenues of each of the presidencies. Although inimical to the ideas of the temperance
activists, the Bombay Act provided the presidency another way to increase its revenue.
1) The temperance advocates argued that colonial excise policy resulted in increase in alcohol
production. Though the 1878 Act was passed to appease these critics, it provided another
route for the Bombay presidency to increase its revenue.
2) The Government of Bombay passed the 1878 Act to curb alcohol production and con-
sumption. Though the presidency made constitutional changes in its excise policy, the
temperance activists used this Act to increase the excise revenues.
3) The colonial excise policy favoured the production and consumption of foreign alcohol
which was criticized by the temperance advocates, who passed the 1878 Act. However,
the presidency found another way to increase its excise revenue.
4) The 1878 Act was passed not only to curb the production of liquor but also to appease
the temperance activists. However, the presidency found a way to get around this act and
increase its revenue.

67
CATapult
VERBAL REASONING

7. Taoists taught that all straining, all striving are not only vain but counterproductive. One
should endeavour to do nothing (wu-wei). It means not to literally do nothing, but to dis-
cern and follow the natural forces – to follow and shape the flow of events and not to pit
oneself against the natural order of things. First and foremost, to be spontaneous in one’s
actions. In this sense the Taoist doctrine of wu-wei can be understood as a way of mastering
circumstances by understanding their nature or principal, and then shaping one’s actions in
accordance with these. On the one hand, the Taoists rejected the Confucian attempts to reg-
ulate life and society and counselled instead to turn away from it to a solitary contemplation
of nature. On the other hand, they believed that by doing so one could ultimately harness
the powers of the universe. By ‘doing nothing’ one could ‘accomplish everything’.
1) The Taoists wanted to follow the natural forces without being counterproductive. But they
were against the Confucian attempts of regulating life and believed in the spontaneity of
their actions.
2) The Taoists taught that one should follow the natural order of things and be spontaneous.
Unlike the Confucians, they advised turning away from life and society in order to harness
the powers of the universe.
3) The Taoists philosophy follows a circular reasoning. Though they wanted to harness the
powers of the universe, they wanted to achieve this without challenging the natural order
of things.
4) Unlike the Confucians, the Taoists thought that all work against the universe was counter-
productive. It was only when one stopped regulating one’s actions and was spontaneous
that the powers of the universe could be harnessed.

8. Given 64 codons and 20 amino acids plus a punctuation mark, there are 1083 possible genetic
codes. What’s so special about the one code that – with a few minor variations – rules all
life on Planet Earth? The answer to this question came from Francis Crick, who argued that
the code need not be special at all; it could be nothing more than a ‘frozen accident’. The
assignment of codons to amino acids might have been subject to reshuffling and refinement
in the earliest era of evolution, but further change became impossible because the code was
embedded so deeply in the core machinery of life. A mutation that altered the codon table
would also alter the structure of every protein molecule, and thus would almost surely be
lethal. In other words, the genetic code is the qwerty keyboard of biology - not necessarily
the best solution, but too deeply ingrained to be replaced or improved.
1) Just like the qwerty keyboard has alternatives but is the most widely used, similarly, the
genetic code is the best possible arrangement due to a frozen accident.
2) The genetic code has been subject to reshuffling and then frozen in a special manner since
the earliest era of evolution. A mutation in the code is lethal and cannot be improved.
3) Though there can be hundreds of variations in the genetic code, the one special code that
rules all life on Earth may not be special, but it has become ingrained in the machinery
of life from the earliest era of evolution.
4) The one special genetic code is nothing but a frozen accident after the earliest era of
evolution. It can be lethal if altered by the structure of the protein molecules.

68
Chapter VA
SUMMARY AND PARAGRAPH COMPLETION 4.2

9. X-rays are today a humble diagnostic tool but in their infancy were considered nothing short

PRACTICE EXERCISE CLASS EXERCISE THEORY


of miraculous. Thomas Edison at one point announced a public demonstration in which he
would take an X-ray photograph of the living brain, showing actual thoughts as they dart-
ed here and there. Within a year after the rays’ discovery, Parisian hucksters were selling
tickets to sideshows purporting to show ghosts captured as X-ray images. In 1896, a New
York newspaper reported that over at Columbia’s College of Physicians and Surgeons, X-rays
were being used to project anatomical diagrams directly into the brains of students, ‘making
a much more enduring impression’.
1) In their infancy, X-rays were not thought of as diagnostic tools, but rather considered
sources of miraculous abilities.
2) In their infancy, X-rays were considered almost miraculous, with a number of people
making extraordinary claims about their powers.
3) X-rays evoked such wonder during their early days, that people believed that they were
capable of performing miracles.
4) X-rays have been used for diagnostic purposes since their early days, but they were also
used for a number of miraculous purposes at that time.

10. It is possible for a building with no artistic accomplishment to become meaningful and sig-
nificant for large numbers of people, simply by having been there all their lives. In the same
way that people feel at home in their house, they feel a bond of recognition when they see
a familiar landmark, and some buildings have been designed with that role in mind. For
example the very prominent city hall in Philadelphia has this role for the inhabitants of the
city. Artistically it is quite an odd building, and it has not been widely imitated by architects
in other parts of the world. Its principal significance is local, but locally it is very significant
indeed.
1) The city hall in Philadelphia is, artistically speaking, an odd building, but it provides a
familiar landmark for the local people, making it significant to them.
2) Buildings such as the city hall in Philadelphia play an important role in the life of the
local people, as they are designed to be recognizable landmarks.
3) Odd-looking buildings like the city hall in Philadelphia are architecturally significant, as
they become familiar landmarks for the people living in that area.
4) Buildings that are not artistically accomplished may still become significant to the local
residents due to their familiarity, e.g. the city hall in Philadelphia.

69
CATapult
VERBAL REASONING

11. Virginia Woolf’s serious joke that ‘on or about December 1910 human character changed’ was
a hundred years premature. Human character changed on or about December 2010, when
everyone, it seemed, started carrying a smartphone. For the first time, practically anyone
could be found and intruded upon, not only at some fixed address at home or at work, but
everywhere and at all times. Before this, everyone could expect, in the ordinary course of the
day, some time at least in which to be left alone, unobserved, unsustained and unburdened
by public or familial roles. That era now came to an end.
1) Though Virginia Woolf claimed that human nature changed in December 1910, it actually
did so in December 2010, when everyone started carrying a smartphone.
2) Human nature did not change in December 1910, but it did in December 2010 when
everyone started carrying a smartphone, as now everyone was constantly connected.
3) Smartphones caused a change in human nature when everyone started carrying them
around December 2010, because now people no longer had any privacy.
4) December 2010 marked the end of an era, because at this time, everyone started carrying
smartphones, which meant that they could be reached at all times.

12. Water and other liquids that are mostly water have high surface tension. That means that the
molecules prefer to bond more strongly to each other than to most of the things you might
spill them on. A liquid with lower surface tension, like alcohol, won’t bead up on a fabric
the way water will; it soaks right in. A bead of water is a molecular huddle, a withdrawal
inward, a refusal to join hands with strangers. Confronted with air, the surface of water pulls
together powerfully enough to form a weak skin. At the far high end of the surface-tension
spectrum is mercury. Mercury beads up and rolls off pretty much any surface you drop it on
without leaving a trace.
1) Liquids can be classified as those with high surface tension, such as water and other
water-based liquids, and those with low surface tension, such as alcohol. The former roll
off a surface, while the latter soak in.
2) Liquids can be classified into two types: those that have high surface tension, like water
and mercury, and those that have low surface tension, like alcohol. The former bead up
on fabric, while the latter soak in.
3) Liquids with high surface tension, like water and mercury, bead up on fabric because their
molecules bond strongly to each other, while liquids with low surface tension, like alcohol,
soak into fabric.
4) Liquids like water and mercury have high surface tension, as their molecules bond strong-
ly to each other, while liquids like alcohol have low surface tension, and thus roll off a
surface.

70
Chapter VA
SUMMARY AND PARAGRAPH COMPLETION 4.2

Directions: The following questions have a paragraph from which the last sentence has been deleted.

PRACTICE EXERCISE CLASS EXERCISE THEORY


From the given options, choose the option that completes the paragraphs in the most appropriate way.

13. During much of the Obama administration’s first year – reversing the approach of the George
W. Bush administration – the White House withheld action on unilateral economic sanctions
and other measures seen as challenging Iran’s regime.
1) This was done with the aim of opposing the George W. Bush administration.
2) The main objective of this reversal in approach was to establish peace in Iran.
3) As part of its revised strategy, however, the State Department is funding some aggressive
institutions offering services intended to aid Iran’s opposition.
4) This was done with a hope to engage Iran diplomatically instead.

14. A nation lives contrary to nature, surely, if it is unable to perpetuate itself by bringing new
citizens into the world. So the European nations, everyone knows, are endangered by their
strangely unnatural dearth of births. Americans still replace themselves in sufficient numbers.
But if one takes a closer look at the data, then even their fertility rate is dropping or just
remaining low among all their ethnic groups.
1) The demographic facts of the European nations are also contrary to nature.
2) Even immigrant groups tend not to have lots of kids for more than a generation.
3) That fact is as important as any other for our national security.
4) By contrast, only 27 percent of those who seldom attend church want a lot of kids.

15. Cumbre Vieja normally erupts at intervals of 20 to 200 years and has not done so since
1971. The next eruption may dislodge an unstable 12-mile-long slab of rock – the size of
the Isle of Man, which will crash to the sea bed, causing a dome of water to go a mile high.
1) The trigger is a volcano on La Palma in the Canary Islands.
2) The rock is already slipping down by infinitesimal degrees and when it goes, it is likely
to collapse in 90 seconds.
3) However, nothing would be as bad as the 2004 tsunami because it was one of the first
in recent times.
4) Cumbre Vieja is most likely to erupt in 2117.

16. Is it right to use bad means for good ends or use bad means to avoid worse ends? Is
moral choice possible between kind but ineffective means and unkind but effective means?
For example, is it right to send people into unemployment and redundancy by installing a
labour-saving plant and ensure a competitive future for the corporation?
1) Is it possible to judge between what is good and what is bad?
2) We have forgotten about electronic typewriters and switched to PCs and laserprinters.
3) Is it right to wear international brands when there are competitive brands within the
country?
4) Ultimately, it all depends on one’s point of view, so there are no ‘right’ answers.

71
CATapult
VERBAL REASONING

17. The story of the movie is set in a classic drama genre. It follows Nash’s life from the be-
ginnings of his academic studies at Princeton all the way to his later years in life when he
is awarded the Nobel Prize. What makes this story unique compared to other movies in the
drama genre is that the audience sees that the protagonist and antagonist is the same per-
son.
1) That Nash is the protagonist is evident but who the antagonist is remains for the audience
to decipher.
2) Nash is the protagonist of the story but he also creates the conflict within and against
himself.
3) The director expects the audience to be intelligent enough to identify these personas in
Nash at all times during the movie.
4) The story takes quite a commonplace approach towards drama by dedicating a significant
amount of screen time to Nash’s love-life.

18. Video games are an unusual medium in that they carry a heavy stigma among non-gamers.
Not everybody likes ballet, but most non-ballet fans don’t accuse ballet of leading to violent
crime and mental backwardness. Video games aren’t so lucky. There’s a sharp divide betw-
een gamers and non-gamers, and the result is a market that, while being large and devoted
is also deeply stagnant.
1) That is the reason people complain so much about video games.
2) The medium’s borders are sharply defined, and it does not find many new takers.
3) Therefore, after a certain amount of gaming, gamers turn to other activities like ballet for
change.
4) However, this is not the case with ballet.

19. You may not think that there is anything wrong with current video discs. But electronics
engineers are busily inventing new families of smarter, more computer-like media devices
which are of same size as DVDs, yet can hold over five times more information and will not
just play movies but will also make it easier to record television shows and store music,
digital-photo albums and home videos.
1) DVDs will no longer be in the market when these products are introduced.
2) DVDs as we know them just aren’t up to the mark.
3) These new devices will definitely cost much more than DVDs.
4) Similarly, DVDs are technologically superior to CDs.

72
Chapter VA
SUMMARY AND PARAGRAPH COMPLETION 4.2

20. We define the aura of natural objects as the unique phenomenon of a distance, however close

PRACTICE EXERCISE CLASS EXERCISE THEORY


it may be. If, while resting on a summer afternoon, you follow with your eyes a mountain
range on the horizon or a branch which casts its shadow over you, you experience the aura
of those mountains, of that branch. It rests on two circumstances, both of which are related
to the increasing significance of the masses in contemporary life. Namely, the desire of con-
temporary masses to bring things ‘closer’ spatially and humanly, which is just as ardent as
their bent towards overcoming the uniqueness of every reality by accepting its reproduction.
1) The manner in which human sense perception is organized is determined not only by
nature but by historical circumstances as well.
2) The concept of aura with reference to natural objects may usefully be illustrated with
reference to the aura of historical ones.
3) Over long periods of time, the aura of human sense perception changes with humanity’s
entire mode of existence.
4) Every day the urge grows stronger to get hold of an object at very close range by way of
its likeness, its reproduction.

21. Was there a common set of rules, Ekman wondered, that governed the facial expressions
that human beings made? Tomkins said that there were. But most psychologists said that
there weren’t. The conventional wisdom of the time held that expressions were culturally
determined – that we simply used our faces according to a set of learned social conventions.
1) Ekman didn’t know whom to believe.
2) Ekman was sure now.
3) Ekman realized that conventional wisdom held the answer.
4) Tomkins had arrived at a conclusion.

22. Bars in Eastern Europe are different. There are drinks on offer that you rarely see elsewhere.
They are not unusual because of the ingenuity of their composition, though one barman in
Tallinn used to offer a ‘Molotov cocktail’, a lethal combination of vodka and locally made
whisky: it was better thrown at a Soviet tank than imbibed, he would explain.
1) On the other hand, the cocktails served in the bars of the Caribbean Islands are made
of a blend of various popular liquors.
2) The fruit juices are simply delicious.
3) The bars are different because of their architecture.
4) Instead, the tipples are unfamiliar because they are the traditional drinks of the region,
largely unknown in the outside world.

73
CATapult
VERBAL REASONING

23. Diesel engines, with their rough-and-ready, spark-plug-free method of fuel ignition, had a
reputation for being smoky and smelly. These days, that is a bit untrue. Fussy consumers and
even fussier regulations have ensured that the sophisticated diesel engines used to propel
modern luxury cars are pretty much as clean as their petrol-powered, sparked-up equivalents.
1) However, petrol is still costlier than diesel.
2) However, people still prefer cars that use diesel compared to cars that use petrol.
3) However, the heavy-duty diesel engines employed in larger vehicles still have a long way
to go.
4) However, some consumers are still reluctant to buy cars that lack spark plugs.

24. While renewed confidence in corporate-bond markets has allowed big companies to raise a
record amount of money, many are still hesitant to spend the cash on hiring or expansion
amid doubts about the strength of the recovery.
1) They are also cautious in buying new properties to expand their business.
2) Most of them are confident about the improving market conditions and are looking forward
to big investments.
3) They want to keep some cash on hand in case Europe’s debt troubles lead to a new
market freeze.
4) They want to keep some cash on hand in case the market gathers momentum and fetches
them more profit.

74
Chapter VA
SUMMARY AND PARAGRAPH COMPLETION 4.2

PRACTICE EXERCISE-3

Directions: The passage given below is followed by four alternative summaries. Choose the option
that best captures the essence of the passage.

1. In the world of consumable products, such as food and news, there is always a need for
more food and news. When the product is consumed, then the consumers want more. It’s
a never-ending cycle. In the world of services, the same applies. Someone has to cook and
serve the food in a restaurant, take care of us when we are sick, do the daily transactions
we all need. Services can be self-sustaining because the need is always there. But a business
that makes and sells durable goods faces a problem: as soon as everyone who wants the
product has it, then there is no need for more. Sales will cease. The company will go out of
business.
1) Consumable products and services are produced in a continuous cycle, but the same
cannot be said of durable goods, which are produced only once.
2) The sale of consumable products and services is self-sustaining as consumers always
need more of them, but the same is not true for durable goods.
3) Unlike in the case of consumable products and services, the sale of durable goods is not
self-sustaining, which may result in their sellers going out of business.
4) The world of consumable products and services is not like that of durable goods, as there
is no risk of going out of business in the former, given the constant demand.

Directions: Each question is a logical sequence of statements with a missing link, the location of
which is shown parenthetically. From the four choices available you are required to choose the one
which best fits the sequence logically. (Past CAT questions)

2. People arguing for a position have been known to cast the opposite in an unnecessarily feeble
light (………………..…………….). People who indulge in this fallacy may be fearful or ignorant of
a strong counter argument. Detecting this fallacy often depends on having already heard a
better refutation, or having information on which to construct one.
1) Casting the opposite as weaker than it really is, is a very effective strategy.
2) This portrayal of a refutation as weaker than it really is, is a sure way of proving your point.
3) Casting the opposite as weaker than it really is, is not a very effective strategy.
4) This portryal of refutation as weaker than it really is, is unwarranted.

75
CATapult
VERBAL REASONING

3. A deliberation is a form of discussion in which two people begin on different sides of an


issue (……….…………….). Then each decides, in the light of the other argument whether to
adopt the other position, to change his or her position somewhat or to maintain the same
position. Both sides realise that to modify one’s position is not to lose, the point is to get
closer to truth of the matter.
1) Each person argues his or her position most sincerely.
2) The prerequisite for deliberation to be productive is that persons involved must keep an
open mind.
3) The purpose is to resolve the issue to the satisfaction of both parties.
4) The trick is to state your viewpoint from a position of strength.

4. The question of what rights animals should enjoy is a vexatious one. Hundreds of millions
of animals are put to death for human use each year. Contrariwise, it can be argued that
slowing down scientific research would retard discovery of antidotes to diseases such as
cancer which kill humans and animals alike (…………………). What if super intelligent beings
from Alpha Centauri landed on earth and decided to use us for their experiments, arguing
that they could save far more of their and our lives by so doing?
1) It will benefit both in the long run.
2) Is the argument truly fair to animals?
3) But the progress of human civilization cannot be made contingent on some hypothetical
rights of animals.
4) There is no question of equating human rights with those of animals.

5. Many of us live one-eyed lives. We rely largely on the eye of the mind to form our images
of reality. It is a mechanical world based on fact and reason (…………………..). So today more
and more of us are opening the other eye, the eye of the heart, looking for realities to which
the mind’s eye is blind. This is a world warmed and transformed by the power of love, a
vision of community beyond the mind’s capacity to see. Either eye alone is not enough. We
need “wholesight”, a vision of world in which mind and heart unite.
1) It has lead to unprecedented scientific growth and material well-being.
2) In the nuclear age, our mind-made world has been found flawed and dangerous, even lethal.
3) The question is irrelevant, whether or not there can be seen and reasoned.
4) We have built our lives based on it because it seemed predictable and safe.

76
Chapter VA
SUMMARY AND PARAGRAPH COMPLETION 4.2

6. Briggs, I fancy, in his way did like it. Briggs was a full-blooded, up-apt, awkward, sanguine

PRACTICE EXERCISE CLASS EXERCISE THEORY


man, who was able to like anything, from gin and water upwards. But with how many a
wretched companion of Briggs’ are we not familiar? Men as to whom any girl of eighteen
would swear from the form of his visage and the carriage of his legs as he sits on his horse
that he was seeking honour where honour was not to be found, and looking for pleasure
……………………………...
1) in places where no honour lay for him
2) wherever he could find it
3) in places where honour led him to
4) where no pleasure lay for him

Directions: The following questions have a paragraph from which the last sentence has been deleted.
From the given options, choose the option that completes the paragraphs in the most appropriate way.

7. The more frequently language learners are exposed to foreign vocabulary, the more likely
they are to remember it. Studies suggest that most learners need between 5-16 ‘meetings’
with a word in order to retain it. Byki does an excellent job providing this repeated exposure.
Every word and phrase must be correctly identified multiple times to obtain the highest score,
while the variety of exercises and activities prevents this repetition from being boring. Lan-
guage learners are thus more likely to use and enjoy the program long enough to accomplish
a sufficient number of ‘meetings’ to master the new vocabulary terms. At the same time, a
proprietary algorithm tracks each learner’s progress and presents the words that need the
most work more often than those that have already been mastered.
1) In that way, language learners get more exposure to the words that they find most diffi-
cult.
2) In order for learners to successfully make the association between a foreign language
word and its meaning, that meaning must be conveyed in a comprehensible manner.
3) When language learners who use Byki do feel ready to form sentences on their own, they
will have a solid base of vocabulary with which to do so.
4) Language learners who take that time are far more likely to use the words correctly when
they do choose to form sentences.

8. Language is learned. As a small child you learned to listen and to speak from the example
set by parents, adults and siblings in your environment. Because the patterns of sounds held
no internal meaning initially, you began by emulating them. Perhaps you were delighted that
you could make the same sound you heard, or were amazed that others responded to your
vocalizations. Because you didn’t know that you could learn. And, because there wasn’t a
formal rigour to learning how to speak and to listen, you learned by experience. Like riding
a bicycle or driving a car, it soon became second nature.
1) Because communication is so important, time spent in developing new skills rather than
depending on these second nature reactions will have real payback.
2) Now, as a competent adult, you generally use those second nature skills without much
conscious thought.

77
CATapult
VERBAL REASONING

3) The ability to use both precise and vague language builds flexibility and creates a wide
range of options.
4) The most difficult kinds of communication became easier.

9. People tend to choose to complete tasks that they are confident in doing and choose not
to partake in tasks that are new to them or they are not confident in. We also tend to
overestimate and underestimate our abilities, which can lead to increasing or decreasing our
self-efficacy. When an individual is confident in a task that they do not have the ability to
complete, it becomes a bad experience; when an individual is not confident in a task and
therefore does not attempt it, growth and development are thereby discouraged. The best
level of self-efficacy, supported by research, is when an individual’s self-efficacy is slightly
above their ability to complete the task.
1) This means that people would tend to take a wider view of a task in order to determine
the best plan to complete it.
2) This means that people would take control of their learning and display higher motivation
than those with low self-efficacy.
3) This means that people would be encouraged to complete challenging tasks and gain
experience in the process.
4) This means that people will be stimulated to greater efforts when they encounter ob-
stacles.

10. Our very physiology as humans was created by our being predators. Most of what predators
do is simply sit and wait for opportunity. Doing nothing actively is a great way to describe
hunting. When I slip into the woods with a weapon in hand – the weapon is critical because
no camera will ever make food for you, and that is a fine but defining difference – and
settle in, several things happen physiologically. The first is an immediate heightening of all
my senses, and pointed attention to those senses. Every sound, smell, caress of wind, and
movement is scrutinized for importance, nothing is ignored. The heightening is followed by
relaxation where my breathing and heart rate both slow. This then leads to a state of mea-
sured attentiveness that is complete but relaxed.
1) I realize that sitting perfectly still in wait for game is a normal, if not necessary, aspect
of being human.
2) The essence of hunting is that meditation actually shortens time, and makes hours seem
like minutes.
3) I am like an Inuit on the ice waiting for a seal to stick his nose into an air hole for hours
on end.
4) I observe everything but steal away inside myself and let my thoughts wander.

78
Chapter VA
SUMMARY AND PARAGRAPH COMPLETION 4.2

11. The historian should describe what has happened but the poet should say what sorts of thing

PRACTICE EXERCISE CLASS EXERCISE THEORY


might happen, that is, the things possible according to likelihood or necessity. The difference
is not between fiction and non-fiction: some past events could have happened according to
likelihood and are thereby candidates for poetic representation. Rather, the poet differs from
the historian with respect to the level of abstraction at which he considers the actions and
experiences of agents. The historian should engage in accurate and thorough research by
carefully recording his or others’ observations of particular events.
1) The historian should report what happened, whether or not the events exhibit explanatory
coherence.
2) This is Aristotle’s general view of the historian but the poet’s still needs explanation.
3) The poet, on the other hand, may discern in the historian’s materials some causally related
events fit for dramatic or epic representation.
4) The poet, on the other hand, looks for causal relations among events, for he cares wheth-
er his composition has a plot with events that happen because of other events and not
merely after them.

12. We have not yet been able to establish whether poetry gives us specifically propositional
knowledge, whether it tells us facts about the world in the same way that science does.
We should be careful here, though. It might be that poetry does indeed provide us with
propositional knowledge, but that it does so in a very different way from science. The two
disciplines are obviously quite distinct, which is precisely why the suggestion that poetry
might teach us facts initially sounded so odd. The whole point of poetry, it seems, is not
to state facts, but to use language in a creative and imaginative way to express thoughts,
feelings and impressions.
1) Maybe if you read a lot of poetry you also develop another skill, namely how to write
poetry.
2) When you study poetry, presumably you develop many skills, like learning how to interpret
a poem, how to measure meter, and so on.
3) Whether or not this could ultimately lead to propositional knowledge is an open question.
4) Perhaps through its creative use of language, poetry provides a unique window into sub-
jective experience.

13. If poetic language is not an enemy but an ally of philosophy, can poetry be used for writ-
ing philosophy? Poetry is a powerful literary form that can do many things, from expressing
love, declaring war, to advertising toothpaste (some say that the best of modern poetry is in
advertisement, and this is not entirely a joke). The tradition that poetry does not engage in
reasoning is based on the understanding that logical rigour and poetic vividness are under-
mining each other. But, does poetry have to be image after image, all the time, so as to
exclude reasoning?
1) There were philosophers who wrote philosophy through poetry successfully.
2) There is no such literary rule, and what is required is that the reasoning involved should
be so interesting that it can be appreciated poetically.
3) The importance of an appropriate image to an abstract theory cannot be overestimated.
4) Poetry, as a literary form, is used for describing feelings or stories, presented through
images and imaginative language.

79
CATapult
VERBAL REASONING

14. How can a genealogical history serve perspectivism? To begin with, genealogy sets out to
demonstrate that ‘meaning’ and ‘matter’ (‘signifier’ and ‘signified’) cannot possibly exist
independently of each other. On the contrary, meanings (viz. ideas or morals) are always
embedded in processes and as such are dynamic creations, not static givens. It will always
be a certain social group who will define a specific set of values, which will be overthrown
when another group will re-evaluate these same values.
1) The material circumstances that give rise to ‘meanings’ are, therefore, the relationships
of domination and power.
2) The history of morality is itself a script which has to be subjected to the most vigorous
textual analysis if its meaning is to be deciphered.
3) The significance of a genealogical history lies in its ability to distinguish ‘origin’ from ‘pur-
pose’.
4) Genealogy demonstrates that, although procedures remain relatively unchanged through
the ages, the purposes to which they are put are fluid.

15. War was also widely seen before 1914 by the upper classes across Europe as an assertion
of masculine honour, like a duel, as it were, only on a much bigger scale. Duelling was a
common way of avenging real or imagined slights to a man’s honour in virtually every Euro-
pean country at the time. Only in Britain had it died out: the point of a duel was to vindicate
one’s manly honour by standing unmoving as your opponent fired a bullet at you at twenty
or thirty paces, and the invention of modern cricket, in which a man was required to face
down a different kind of round, hard object as it hurtled towards him from the other end of
the wicket, was a satisfactory (and comfortingly legal) substitute.
1) Such codes of male behaviour appear almost incomprehensible a century later.
2) Strength of will, self-assertion and standing firm against an enemy were all part of a code
of behaviour of the upper-class men whose actions brought Europe to war in 1914.
3) Yet, at the same time, in 1914, the leaders of almost every European nation were racked
with anxiety about the future.
4) The war appeared as a release, a liberation of manly energies long pent up, a resolution
to all the insoluble problems that had plagued European politics and society.

16. In Nietzsche’s view, Socratic culture was a response to the breakdown and fragmentation
of tragic Greek culture which it attempted to replace with a set of shared, homogeneous
ethical values, theoretical norms, and methodological procedures, based on Socratic logic
and reasoning, which would replace the warring gods of the Greeks with a more unified
rational culture. In a sense, Socratic culture thus provided a cure for a cultural emergency
with extreme rationalism coming to curb the strong, warring impulses that had been released
and that Socrates believed were out of control. The result was an equation of reason and
knowledge and virtue, making reason the instrument of both truth and morality.
1) Thus, Socratic culture replaced the tragic vision of suffering and redemption through culture
with the optimism that reason can discover truth and produce a good life.
2) Thus, Socratic culture replaced tragic Greek culture with the formative force of the modern
period, with life-changing results.

80
Chapter VA
SUMMARY AND PARAGRAPH COMPLETION 4.2

3) Thus, the triumph of Socratic theoretic man, which was counterpoised to a tragic pessi-

PRACTICE EXERCISE CLASS EXERCISE THEORY


mism, provided the origins of modern rationalism and Enlightenment optimism.
4) Thus, Socratic culture was seen as an alternative to the earlier tragic Greek culture, which
had become a symbol of decay and atrophying life-instincts.

17. The romantics made nature even more central to their work than the metaphysical poets,
treating it as an elusive metaphor in their work. They sought a freer, more personal expression
of passion, pathos and personal feelings, and challenged their readers to open their minds
and imaginations. Through their voluminous output, the romantics’ message was clear: life
is centred in the heart, and the relationships we build with nature and others through our
hearts define our lives.
1) Ironically, the poets held distinctly different religious beliefs and led divergent lifestyles.
2) Romantic poets shared an interest in metaphysical subjects and practised similar means
of investigating them.
3) The Romantic movement would have likely extended further but the premature deaths of
the younger poets brought the period to an end.
4) They anticipated and planted the seeds for free verse, transcendentalism and countless
other artistic, musical and poetic expressions.

18. Poetry, as an art form, predates literacy. In preliterate societies, poetry was frequently em-
ployed as a means of recording oral history, storytelling (epic poetry), genealogy, law and
other forms of expression or knowledge that modern societies might expect to be handled
in prose. The Ramayana, a Sanskrit epic which includes poetry, was probably written in the
3rd century BCE in a language described by William Jones as ‘more perfect than Latin, more
copious than Greek and more exquisitely refined than either’. Poetry is also often closely
identified with liturgy in these societies, as the formal nature of poetry makes it easier to-
remember priestly incantations or prophecies.
1) Preliterate societies, lacking the means to write down important cultural information, used
oral learning to preserve it.
2) Other forms of poetry include narrative poetry and dramatic poetry, both of which are
used to tell stories and so resemble novels and plays.
3) The greater part of the world’s sacred scriptures are made up of poetry rather than prose.
4) In preliterate societies, all these forms of poetry were composed for, and sometimes
during, performance.

81
CATapult
VERBAL REASONING

19. The changes in the world of macroscopic objects are caused by rearrangements of the atomic
clusters. Atoms can differ in size, shape, order and position (the way they are turned); they
move about in the void, and – depending on their shape – some can temporarily bond with
one another by means of tiny hooks and barbs on their surfaces. Thus the shape of individual
atoms affects the macroscopic texture of clusters of atoms, which may be fluid and yielding
or firm and resistant, depending on the amount of void space between and the coalescence
of the atomic shapes.
1) The atoms are unchangeable and contain no internal differentiation of a sort that would
allow for division.
2) Since the atoms are separated by a void, they cannot fuse, but must rather bounce off
one another when they collide.
3) Though the atoms themselves persist through all time, everything in the world of our
experience is transitory and subject to dissolution.
4) The relative density and fragility of different materials are also accounted for by the same
means.

20. It remains unknown precisely how children acquire the nuanced contexts for a given expletive.
But we do know that by the time a child is old enough for college, his memory for taboo
words in psychology experiments is excellent. Taboo words tend to be emotional words, and
emotional things attract our attention and keep it. The distinctiveness of taboo words also
helps us remember them. In many contexts – church, classrooms, and dare I say psychology
experiments – we simply don’t expect to encounter swearing. So when we do, it stands out.
1) Taboo words have a hold on us that goes beyond their emotional impact and we use them
as a survival strategy.
2) All taboo words are not created equal, nor are they equal-opportunity offenders.
3) Thus, changing the context in which swearing occurs can change how we experience it.
4) Alternatively, though not incompatibly, we may swear simply because it makes us feel
better.

21. On my bedside table, there’s a precarious column of half-read paperbacks that taunts me
with the evidence of my own readerly promiscuity. The reason I don’t finish books is not that
I don’t like reading enough; it’s that I like reading too much. I can’t say no. I’ll be reading a
novel and thoroughly enjoying it. Then I’ll be in a bookshop and I’ll see something I’ve been
anticipating, and I’ll buy it. I’ll start reading the new book on the bus home that evening,
and that will be the end of the original affair.
1) Abandoning books was just something I did and something I was increasingly unable to
stop myself from doing.
2) I’ll start a book, get about halfway through it, and then, even if I’m enjoying it, put it
down in favour of something else.
3) I have invested in the relationship with the book I’m currently reading, but I can’t help
myself from pursuing whatever new interest happens to turn my head.
4) This is partly due to contemporary conditions: the degenerative disorder of the at tention
span that now affects pretty much anyone with an Internet connection.

82
Chapter VA
SUMMARY AND PARAGRAPH COMPLETION 4.2

22. Fans of quaint and kitschy postcards spend their entire vacations on the lookout for some

PRACTICE EXERCISE CLASS EXERCISE THEORY


especially outrageous example to amuse their friends back home. Once they find the right
card, they are faced with the problem of what to write on the other side. A conventional
greeting won’t do. A few details about the trip and an opinion or two are okay, but even
better is to come up with something clever, since every postcard is written with a particular
person in mind. No doubt, one writes differently to one’s friends than to one’s parents, who
always fear the worst when one is away.
1) Thus, instead of a postcard of the Sistine Chapel, send them one of a neighbourhood
pizzeria’s elderly owner and his wife wiping their hands and smiling broadly.
2) Thus, it’s tempting, when one sits down to send news home, to do the unconventional
and use the small space allotted for writing to have a little fun.
3) Thus, postcards continue to be used by people of modest means even though emails are
a more popular medium of communication these days.
4) Thus, if you come across some poor soul sitting alone over a postcard and visibly strug-
gling with what to write, take pity on him or her.

23. At that very moment, Washington was planning with the French government to launch an
invasion of Vietnam, designed to overthrow the Democratic Republic of Vietnam and restore
French colonial rule. This would be a joint French-American project. The United States would
supply the weapons and the financing. A dozen US troopships would be diverted from bringing
soldiers home from Europe to carrying the French invasion army – equipped with American
weapons, tanks, warplanes and jeeps – to Vietnam. This was arguably the beginning of Amer-
ica’s Vietnam War.
1) Just as kids believed in America’s peaceful future, the Vietnamese believed that Americans
were their friends and allies.
2) Vietnam’s war for independence was at the vanguard of a global revolution that destroyed
the colonial form of imperialism.
3) As the French war against Vietnam went on, with increasing US covert involvement, op-
position to this Franco-American war grew and intensified.
4) It was also, as it turns out, the beginning of the American people’s movement against
that war.

24. For all the tadpoles that perish, some still make it to become frogs, and have been doing
so for at least 200 million years. Those that don’t are the stuff of life for countless other
creatures, from the littlest insect larvae to grand old storks. Indeed, frogs are regarded as
a keystone species, which means that the death of their multitudinous offspring, along with
the death that they themselves deal out, is crucial to the flourishing of the community of
life.
1) We too are built on a bedrock of old men’s bones: our evolution to Homo sapiens is a
product of the endless winnowing out of the unfit and the unfortunate.
2) In the language of ecology, life and death are obligate symbionts, each wholly dependent
on the other.
3) This party of death is, of course, at the same time a cork-popping party of life.
4) It seems rather an outrage; a cosmic crime; a reason to raise one’s fist and rebel against
the regime that necessitates this slaughter of innocents.

83
CATapult
VERBAL REASONING

25. Adorno’s account of import and function distinguishes his sociology of art from both herme-
neutical and empirical approaches. He argues that, both as categories and as phenomena,
import and function need to be understood in terms of each other. On the one hand, an
artwork’s import and its functions in society can be diametrically opposed. On the other-
hand, one cannot give a proper account of an artwork’s social functions if one does not raise
import-related questions about their significance. So too, an artwork’s import embodies the
work’s social functions and has potential relevance for various social contexts.
1) Social functions are primarily intellectual functions rather than straightforwardly political
or economic functions.
2) In general, Adorno gives priority to import, understood as societally mediated and socially
significant meaning.
3) Adorno claims that an artwork’s functions are social antitheses while its import is instru-
mental to reason.
4) Typically, Adorno elaborates that import and functions are categorized as polarities or
dialectical pairs.

26. For centuries, a small number of writers were confronted by many thousands of readers.
This changed toward the end of the last century. With the increasing extension of the press,
which kept placing new political, religious and local organs before the readers, an increas-
ing number of readers became writers. It began with the daily press opening space to its
readers for ‘letters to the editor’. And today, there is hardly a gainfully employed European
who could not, in principle, find an opportunity to publish comments on his work, grievances,
documentary reports, or that sort of thing, some where or other.
1) Thus, the distinction between author and public is losing its basic character.
2) Thus, literary license is now founded on polytechnic rather than specialized training.
3) Thus, the same public now responds in a progressive manner rather than in a reactionary
manner.
4) Thus, the critical and the receptive attitudes of the public coincide – they are in a position
to judge as well as be judged.

84
CATapult
VERBAL REASONING

VA-4.3 CRITICAL REASONING 1


THEORY

Critical Reasoning
Introduction

Critical Reasoning is an area that is used in national and international tests to test reasoning
skills. It is an invaluable tool for any one taking competitive exams like the CAT, to sharpen his/
her reasoning skills. Critical Reasoning questions usually include short passages of three to four
sentences followed by questions such as:

 Which of the following conclusions can be best drawn from the statements above?
 The conclusion above depends on which of the following assumptions?
 Which of the following must be true if the above statements are true?
 Which of the following strengthens/weakens the argument made above?

Structure of an argument
The components of an argument are defined as:
A premise is a statement, fact or evidence, stated in the passage, which is presumed to be true
in the context of the passage. Premises are explicitly stated.
When a premise is not stated, or the author has taken it for granted, it becomes an assumption.
A conclusion is an opinion or an assertion that the author wants to prove, using premises and
assumptions.
The strength of the conclusion/assertion depends on the validity of the premises.
I. A and B are two households with the same income.
II. Household A saves more than Household B.
III. Household A spends less than Household B.

Statement III is a logical deduction from statements I and II. Such logical deductions are called
inferences. An inference is a logical certainty that can be derived from the facts presented in the
argument.
It is very important to understand the difference between Assumptions, Inferences and Conclusions.
Many students have a tendency to get confused between the three terms, or rather they find
it difficult to understand how each one is different from the other. In order to understand the
difference between the three, you have to understand the role played by each of the three in the
construction of an argument.
Assumptions, Conclusions and Inferences follow a consecutive sequence in the construction of an
argument.

85
CATapult
VERBAL REASONING

Premise A + Premise B Conclusion


As you can see from the line diagram, it is impossible to reach the conclusion unless the premises
are valid. When the author draws the conclusion without stating one of the premises, that particular
premise becomes an assumption, i.e. the author has assumed the premise to be valid. The moment
the assumption ceases to be valid, the conclusion is invalidated.
Inferences as defined earlier are logical certainties that can be deduced based on premises
(Statements/ Facts/ Evidence).

Critical Reasoning Question Types

The following are the most frequently asked Critical Reasoning question types:
 Conclusion  Weaken
 Assumption  Parallel Reasoning
 Inference  Explain/Resolve
 Strengthen  Flawed Reasoning

Conclusion Questions

Example
Anyone familiar with the prisons knows that some inmates behave little better than brute beasts.
If the death penalty had been truly effective as a deterrent, such prisoners would have vanished
long ago.
Which of the following conclusions is best supported by the passage above?
(1) There would have been more such prisoners if it were not for the presence of capital pun-
ishment as a deterrent.
(2) The prison system has managed to reform many prisoners from brutish beasts to citizens
who can return back to society.
(3) The way capital punishment is administered might have to be made more horrific in order
to deter people from behaving like brutish beasts.
(4) Capital punishment has not been able to deter individuals from perpetrating gruesome crimes
against their fellow men.

Strategy
 Conclusion should clearly follow only from the premises stated and should not make any
additional assumptions.
 Conclusions should not be treated as questions that require exploring of all possible scenarios
arising from the premises.

86
Chapter VA
CRITICAL REASONING 1 4.3

Explanation: Option (1) cannot be concluded since there is no evidence in the premises that

THEORY
supports this; it is just an opinion but not a conclusion that can be drawn from the premises.
Evidence that shows that, since the introduction of capital punishment the crime rate has dropped
will support such a conclusion. Similarly the premises do not support options (2) and (3): much

PRACTICE EXERCISE CLASS EXERCISE


more information would be needed before either of these can be concluded from this paragraph.
Only option (4) is a conclusion that can be safely made from the given premises. Hence, (4).

Assumption Questions

Example
Since the good is that which furthers a person’s real interests and not his/her self-interest, it
follows that in any given case when the good is known, men will seek it.
The argument above is based on which of the following assumptions?
(1) Men can differentiate between their real interests and self-interests.
(2) Men would prefer to choose actions that are in their real interests to those in their self-in-
terest.
(3) Good is something that is not desirable in itself.
(4) Men should seek the good rather than the bad.

Strategy
 An assumption is something that has been considered to be true by the author while arriv-
ing at his conclusion on the basis of the facts that are available to him. It is something he
has taken for granted and hence, it is not stated. So, it is like an unstated fact. It serves to
bolster the author’s argument.
 By negating the assumption the conclusion should fail, so negate each option and see if the
conclusion still holds true. Choose the option which when negated makes the conclusion fail.

Explanation: Negating option (1) will mean that men cannot differentiate between real interests
and self-interests but it does not mean that they will not seek good (even if it is inadvertently,
not knowing that they are seeking their real interests). Negating option (2) will mean that they
would prefer self-interests to real interests thus negating the conclusion that they will seek the
good since the good caters to real interest. So option (2) is the assumption. Negating option (3)
would mean that good is something that is desirable in itself but that does not mean that people
will not seek good if they know the good. Negating option (4) would mean that men should seek
the bad instead of the good, but this is a prescription, i.e. what they should do, not what they
would do. Hence, (2).

87
CATapult
VERBAL REASONING

Inference Questions

Example
The revenues of a certain automobile company increased dramatically after they launched a new
model that consumes significantly less fuel than all of its previous models as well as those of its
competitors. Despite this, the profitability (profit as a percentage of revenues) of the company fell
by almost 10 percentage points, while its competitors maintained their profitability levels.
Which of the following can be inferred from the statements above?
(1) The company greatly increased its advertising budget to promote its new model that brought
in more revenue.
(2) The overall costs increased at a rate higher than the rate of increase in revenues.
(3) The company’s market share has not increased despite increased revenues from the new
model.
(4) The company has not been able to charge a significantly higher price for the new model over
its existing models as well as those of its competitors.

Strategy
 An inference is something implied by the text (and not explicitly stated). If you read be-
tween the lines or put two and two together, the inference will seem to be true. Unlike the
conclusion, which seems to be ‘definitely true’, an inference may seem to be ‘probably true’.
While the conclusion ought to be the main point of the argument, an inference (there can be
many) could just be a minor detail that is implied. Look for an option that you can deduce
to be reasonably true on the basis of the information provided.
 For an inference question, all statements in the paragraph become premises.
 Since an inference has to follow logically from the premises, negating the inference should
result in the premises being negated. Negate each option and see if the premises still hold
true. Choose the option which when negated makes a premise fail.

Explanation: Negating option (1) means that the company decreased its advertising budget; but
this does not mean that profitability should not have fallen since the new product could have
incurred other costs, like manufacturing, which drastically increased costs. Negating option (2)
would mean the rate of increase of costs is slower than the rate of increase of revenue, which
means that the profitability could not have fallen. So option (2) is the valid inference that can be
drawn from the premises. Negating option (3) would mean that the company’s market share has
increased but this does not mean that its profitability has also increased since market share is
share of the company’s revenue in the industry’s revenue. Negating option (4) would mean that
the company was able to charge a significantly higher price but this does not mean profitability
should also have increased since costs could also have increased correspondingly. Hence, (2).

88
Chapter VA
CRITICAL REASONING 1 4.3

Strengthen Questions

THEORY
Example

PRACTICE EXERCISE CLASS EXERCISE


In our personal lives, most of us are familiar with social software tools, but most of us have not
experienced them extensively in the enterprise. While social software may well improve relationships,
build trust and community, and tap into a greater diversity of ideas, these vague promises do little
to convince sceptical executives concerned primarily with business performance. This is a shame
because social software can be tightly linked to business performance improvement.
Which of the following strengthens the argument made in the passage above?
(1) The only way to accelerate and sustain usage of social software in the enterprise is to target
implementations against very specific operating performance levers.
(2) Social software advocates haven’t done a very good job of communicating the value these
tools bring to the enterprise.
(3) By using social software, employees have more interactions and both their explicit knowledge
and their expertise is exposed to others who need them but were not looking for them.
(4) Many executives are wary of the potential loss of confidentiality and expanded opportunity
for airing personal grievances.

Strategy
 Differentiate between options that are related to the argument and those that clearly strengthen
or weaken the argument.
 Reject options that are only related to the argument (neither strengthening nor weakening it)
in general.
 Reject options that do not specifically strengthen the conclusion in the argument with addi-
tional evidence.

Explanation: The argument is that social software can be used for business performance improvement.
So to strengthen it, the options must show how social software can add to business performance
improvement. Option (1) does not strengthen the argument since it talks about how social software
should be used, not how it is useful. Option (2) is incorrect since it talks about why its benefits
are not known and not the benefits themselves. Option (3) shows how employees can benefit
from using social software, thus strengthening the argument. Option (4) indirectly casts doubts on
social software instead of supporting it. Hence, (3).

89
CATapult
VERBAL REASONING

Weaken Questions

Example
Recently schools in a major city have decided to ban the use of mobile phones by students while
they are at school. The schools contend that usage of mobile phones is the only reason behind
the reduced attention levels of their students.
Which of the following most weakens the plan of action of the schools to combat the low attention
levels of their students?
(1) Students in schools of neighbouring cities have not been reported to have low attention levels
despite having similar mobile phone usage patterns while at school.
(2) Students in schools of neighbouring cities have been reported to have low levels of attention
despite the fact that they have significantly lower levels of mobile phone usage.
(3) Adults have also reported a decline in attention levels due to the increased usage of mobile
phones at the workplace.
(4) Most of the students of the schools in the major city spend a lot of their post-school time
watching television or playing video games.

Strategy
 Clearly identify the line of reasoning used, X causes Y or X will improve Y, etc.
 Choose options that invalidate this line of reasoning. For example, If X then Y, is weakened
by X but no Y and not Y but no X.
 Identify reasoning fallacies committed in the argument.

Explanation: The argument says that only usage of mobile phones causes attention deficit, or only
X is Y, where X is mobile phone usage and Y is attention deficit. Only X is Y does not mean All
X is Y; option (1) makes this mistake and tries to weaken the argument by showing that Some
X is not Y. Only X is Y means that All Y is X; to weaken this is it should be shown that Some Y
is not X; option (2) does this. Options (3) and (4) are only indirectly related to the argument and
do not weaken them in any way. Hence, (2).

90
Chapter VA
CRITICAL REASONING 1 4.3

Parallel Reasoning Questions

THEORY
Example

PRACTICE EXERCISE CLASS EXERCISE


Developed countries have no right to ask developing countries to cap their carbon emissions in
order to curtail global warming since the former have been instrumental in releasing large amounts
of carbon to fuel their development and bringing the environment to its current stage.
Which of the following statements uses a line of reasoning that is most similar to the argument
above?
(1) The green movement which seeks to save the environment by urging companies to practise
sustainable manufacturing is forgetting that such products will be more expensive, thus mar-
ginalizing the poor.
(2) It is unfair to ask drivers to compulsorily wear seat belts, since the cost of not wearing seat
belts is always borne exclusively by the drivers themselves.
(3) The ban on smoking in public places is unwarranted since non-smokers can choose not to
go to places that allow smoking in their premises.
(4) He has no right to ask me to not to pay bribes to reduce corruption, since he is part of a
generation which grew up paying bribes to get things done, thus institutionalizing it.

Strategy
 Convert the whole argument into a simple one-line statement. For example, it is ok to do X
since even if I do not do it, some else will do it.
 Try to fit each of the options into the one line statement that mirrors the logic of the para-
graph.
 Reject options that do not fit into the one-line statement.

Explanation: The argument is of the following form: A cannot ask me not to do X in order to
decrease Y, since A himself is responsible for causing Y. It can be seen that only (4) can be fit
into the form of reasoning the argument uses. Hence, (4).

91
CATapult
VERBAL REASONING

Explain/Resolve Questions

Example
Ask CEOs about their top priorities and inevitably they will cite talent as one of their top priorities.
If this is the case, how do we explain the enormous popularity of Dilbert and The Office, which
so eloquently describe the stultifying effect of our work environments on talent?
Which of the following provides the best explanation for the paradox cited above?

(1) Executives tend to focus on talent acquisition and retention, but do not invest much time on
talent development throughout the firm.
(2) Executives tend to not offer their employees significant pay rises in the first year to offset
the raises given to new recruits vis-à-vis their previous jobs.
(3) Dilbert and The Office are exaggerations and not an accurate depiction of the effect that
workplace environment has on talent.
(4) Most employees tend to have an inflated sense of their own talent that is often at odds with
the ways their superiors perceive them.

Strategy
 Clearly define the two premises that are contradictory. For example, X is supposed to cause
an increase in Y but it is causing a decrease in Y.
 Look for options that clearly offer an explanation for the paradoxical relationship between
the two premises.
 More often, the correct option is related to the way X and Y have been defined or generally
understood.

Explanation: The paradox: CEOs (executives) say that talent is their top requirement; yet employees
feel that the workplace environment is killing their talent. So the problem lies either in the CEO’s
interpretation of talent or the employees’ evaluation of the workplace environment. Option (1) best
explains the paradox, CEOs focus on hiring talent but not on talent development, which is why
employees feel that their talent is being killed. Option (2) diverts the issue by talking about pay and
does not address the talent paradox. Option (3) states that Dilbert and The Office are exaggerations
but that does not mean that the paradox does not exist. Option (4) makes a generalization about
most employees; also the paradox is not that their superiors do not recognize their subordinates’
talent, but that the workplace environment does not foster or harness it, which it is supposed to
do if talent was a top priority. Hence, (1).

92
Chapter VA
CRITICAL REASONING 1 4.3

Flawed Reasoning

THEORY
Example

PRACTICE EXERCISE CLASS EXERCISE


Statistics reveal that there are significantly more purchasers of video games in the 35 to 50 age-
group than in the 15 to 30 age-group, thus refuting the popular misconception that it is the latter
demographic group that is most addicted to video games.
The argument above is flawed because it:

(1) assumes that both demographic groups at equally proficient at playing video-games.
(2) ignores the fact that videogames can be played irrespective of one’s age.
(3) assumes that the purchases and the players of the game are the same.
(4) discounts the fact that older people might be purchasing video-games to appear to be in
sync with the times.

Strategy
 Focus on the relationship between the premise and the conclusion and try to find a mismatch.
 Look for the reasoning fallacies listed in the next section.

Explanation: The argument above makes the mistake of assuming that the people who purchase
the product are the people who use the product. Adults might be the main purchasers of ice-
creams but the consumers can be their children at home; similarly men can be main purchasers
of diamonds but women will be the main users. Option (3) identifies this flaw. The rest of the
options do not precisely capture this flaw of reasoning. Hence, (3).

93
CATapult
VERBAL REASONING

CLASS EXERCISE

Directions: Each passage below is followed by a question. Read the passage carefully and answer
the question that follows.

1. Financial dissatisfaction might negatively influence tax morale. Such dissatisfaction might
create a sense of distress, especially when taxes have to be paid and there is a discrepancy
between the actual and the aspired financial situation.
Which of the following can be concluded from the above statements?
1) Taxes might be perceived as a strong restriction that increases financial dissatisfaction
and thus the likelihood of tax dishonesty.
2) Factors that are more closely related to taxation might have a stronger impact on tax
morale than other factors.
3) If tax revenues were spent more in accordance with the preference of tax payers, tax
morale might improve.
4) Measures to prevent tax evasion need to become more stringent and punishment for the
guilty should be more severe.

2. According to Plato, the critic functions as a kind of philosophical traffic cop, admitting certain
“useful” kinds of literature to the republic but banishing the rest. Archetypal critics believe
that humans experience reality in terms of certain basic fears, desires, images (symbols) and
stories (myths). They conclude that audiences will react to such writing patterns forcefully
and almost automatically and that critics should therefore study the ways such patterns affect
writers, texts and readers.
Which of the following is assumed by the archetypal critics?
1) They assume that writers will inevitably employ such patterns.
2) They assume that no other patterns have such a strong impact on readers.
3) They assume that it is their duty to banish worthless literature.
4) They assume that a person’s fears and desires shape the individual he is to become.

3. About 96% of Scandinavian moths have ears tuned to the ultrasonic pulses that bats, their
predators, emit. But the remaining 4% do not have ears and are deaf. However, they have
a larger wingspan than the hearing moths, and also have higher wing-loadings—the ratio
between a wing’s area and its weight—meaning higher manoeuvrability.
Which one of the following can be best inferred from the above passage?
1) A higher proportion of deaf moths than hearing moths fall prey to bats.
2) Deaf moths may try to avoid bats by frequent changes in their flight direction.
3) Deaf moths are faster than hearing moths, and so are less prone to becoming a bat’s
dinner than hearing moths.
4) The large wingspan enables deaf moths to better receive and sense the pulses of their
bat predators.

94
Chapter VA
CRITICAL REASONING 1 4.3

4. Although in the limited sense of freedom regarding appointments and internal working, the

THEORY
independence of the Central Bank is unequivocally ensured, the same cannot be said of its
right to pursue monetary policy without co-ordination with the central government. The role
of the Central Bank has turned out to be subordinate and advisory in nature.

PRACTICE EXERCISE CLASS EXERCISE


Which one of the following best supports the conclusion drawn in the passage?
1) A decision of the chairman of the Central Bank to increase the bank rate by two percent-
age points sent shock-waves in industry, academic and government circles alike.
2) The government has repeatedly resorted to monetisation of debt despite the reservations
of the Central Bank.
3) The Central Bank does not need the central government’s nod for replacing soiled currency
notes.
4) The inability to remove coin shortage was a major shortcoming of this government.
(Past CAT question)

5. The offer of the government to make iodised salt available at a low price of one rupee per
kilo is welcome, especially since the government seems to be so concerned about the ill
effects of non-iodised salt. But it is doubtful whether the offer will actually be implemented.
Way back in 1994, the government, in an earlier effort, had prepared reports outlining three
new and simple but experimental methods for reducing the costs of iodisation to about five
paise per kilo. But these reports have remained just these—reports on paper.
Which one of the following, if true, most weakens the author’s contention that it is doubtful
whether the offer will be actually implemented?
1) The government proposes to save on costs by using the three methods it has already
devised for iodisation.
2) The chain of fair-price distribution outlets now covers all the districts of the state.
3) Many small-scale and joint-sector units have completed trials to use the three iodisation
methods for regular production.
4) The government which initiated the earlier effort is in place even today and has more
information on the effects of non-iodised salt.
(Past CAT question)

6. As medicine and life extension advances, the life expectancy of the population increases
somewhat each year, and this process may accelerate given new technologies or new knowl-
edge. The longer you live, the more medical advances will occur during your lifetime which
extend your life expectancy. If the increase in life expectancy becomes greater than one
year longer life/year lived then the break-even point is reached and there is a finite chance
of living indefinitely.
Which of the following is a situation analogous to the one depicted in the passage?
1) In a cost/benefit analysis the break-even point is reached when the benefit of making a
decision becomes greater than the cost of making the decision.
2) At boiling point the break-even point is reached when water is just about to be converted
to vapour.
95
CATapult
VERBAL REASONING

3) In a fusion reaction the break-even point is reached when additional energy produced by
the reaction becomes greater than that used to drive the reactor.
4) In a thermal plant the break-even point is reached when the water level in the reactor
becomes greater than that supplied to the turbines.

7. Over the last years, Bank of Bharat has seen the number of its retail customer accounts drop
by over 40 percent. Over the same period, the share price of Bank of Bharat has increased
by more than 80 percent. This has amazed a few investors, who believe that a bank’s share
price should drop if its number of customers drops.
Which of the following, if true over the last years, best accounts for the observed movement
in the price of Bank of Bharat’s equity shares?
1) Two years ago, Securities and Exchange Board started an investigation on the bank for
accounting irregularities, but last year the company was cleared of all charges.
2) The bank recently implemented a highly publicized program for free home loans.
3) The bank has been switching its customer base from retail customers to commercial
customers, which now accounts for over 75 percent of the bank’s revenues.
4) There have been many new banks, which have entered retail banking business over the
last 5 years.
5) The bank is known to be the best paymaster in the industry.

8. A research study recorded that the number of unemployed educated youth was equal to the
number of unemployed uneducated youth. It was concluded by the researchers that being
educated does not enhance the probability of being employed.
Which of the following information would be required to validate the above conclusion?
1) The number of unemployed educated and uneducated people in other age groups.
2) The number of organisations employing youth.
3) The percentage of unemployment in educated youth versus percentage of unemployment
in uneducated youth.
4) The percentage increase in the number of educated youth in the current year versus last
year.

9. These days industries have to follow rigid environmental standards while building new plants.
However, standards that must be met can change and construction that was once acceptable
can now be halted if it conflicts with newly-passed regulations. In such a case new factories
will not be built.
Which of the proposals, if accepted, would reduce the threat to the construction of new
factories?
1) Environmental regulations must be published as soon as they are made.
2) The government should pay to bring up recently constructed factories to new environmental
standards.
3) Industries must immediately comply to new environmental standards.
4) Environmental standards for any factory should not be altered once construction is un-
derway.

96
Chapter VA
CRITICAL REASONING 1 4.3

10. A couple of years ago, the government introduced the self-employment scheme to guarantee

THEORY
the graduate youth an alternative to government jobs. Today, over 1,50,000 twenty-five to
thirty-year-olds are still queuing outside employment exchanges in the hope of an office job.

PRACTICE EXERCISE CLASS EXERCISE


Each of the following, if true, could account for the above except:
1) The programme is uninteresting and not challenging enough to attract the youth.
2) Some prefer the security that a government job offers.
3) The number of 25 to 30-year-old graduates has grown over the last couple of years.
4) The graduates are unaware of the potential benefits of the programme.

11. CitiBank will be sending specially-bound gift copies of a new book, Bollywood: Popular Indian
Cinema, to its most favoured NRI account holders all over the world. ‘We wanted to give
them something special, something Indian, and most Indians love Bollywood.’
What assumptions are implicit in this passage?
1) NRIs love similar things that Indians love.
2) NRIs love Hindi films more than anything else from India.
3) NRIs accounts are the most favoured accounts for CitiBank.
4) Bollywood: Popular Indian Cinema is the best book on Indian films.

12. Risks of cardiovascular disease have been associated, in previous studies, with both saturat-
ed fats and trans-fatty acids.  The controversy, so it goes, involves the argument over which
one is worse.  Studies funded by the margarine industry have declared trans-fats safer than
saturated fats; studies funded by the butter industry show the opposite.
Which of the statements is definitely not true as per the passage?
1) Studies regarding cardiovascular disease are fraught with controversy.
2) The margarine and butter industries are competitors.
3) Research data is unreliable.
4) Earlier, cardiovascular disease was not attributed to trans-fatty acids.

13. The Union government has decided to remove all customs duties on second-hand computers
and peripherals like printers, scanners, monitors, etc., when imported by a recognized school
run on a non-commercial basis. The exemption is presently available to only government-run
schools.
The Union government’s decision would prove to be counterproductive if:
1) government-run schools are well funded and most of them have patrons giving huge do-
nations.
2) indigenous PCs and peripherals are of a superior quality and cost less.
3) government-run schools appoint the worst of teachers.
4) Both (2) and (3).

97
CATapult
VERBAL REASONING

14. Growth is a process of change: it requires flexibility, adaptability, and the willingness to ex-
periment. Above all, it is a process of trial and error that requires an effective system for
ruthlessly weeding out errors and for generously backing successful experiments. But cen-
tralized economic planning tends to be cumbersome and rigid.
Which of the following, is the most likely conclusion of the above passage?
1) Growth cannot be brought about by rigid and inefficient experimentation.
2) Successful experiments are not given enough encouragement in centralized economic
planning.
3) Any process of trial and error that does not provide space for mistakes is unlikely
to be an efficient model of planning.
4) Growth cannot be wrought in an environment like the one provided by centralized economic
planning.

15. Cigarettes constitute a mere 20% of tobacco consumption in India, and fewer than 15% of
the 200 million tobacco users consume cigarettes. Yet these 15% contribute nearly 90% of
the tax revenues to the Exchequer from the tobacco sector. The punitive cigarette taxation
regime has kept the tax base narrow, and reducing taxes will expand this base.
Which one of the following best bolsters the conclusion that reducing duties will expand the
tax base?
1) The cigarette manufacturers’ association has decided to indulge in aggressive promotion.
2) There is a likelihood that tobacco consumers will shift to cigarette smoking if cigarette
prices were to reduce.
3) The cigarette manufacturers are lobbying for a reduction on duties.
4) An increase in duties on non-cigarette tobacco may lead to a shift in favour of cigarette
smoking.
(Past CAT question)

16. Thomas Malthus, the British clergyman turned economist, predicted that the planet would
not be able to support the human population for long. His explanation was that human
population grows at a geometric rate, while the food supply grows only at an arithmetic rate.
Which one of the following, if true, would not undermine the thesis offered by Malthus?
1) Population growth can be slowed down by the voluntary choices of individuals and not
just by natural disasters.
2) The capacity of the planet to feed a growing human population can be enhanced through
biotechnological means.
3) Human systems, and natural systems like food supply, follow natural laws of growth which
have remained constant, and will remain unchanged.
4) Human beings can colonize other planetary systems on a regular and on-going basis to
accommodate a growing population.
(Past CAT question)

98
Chapter VA
CRITICAL REASONING 1 4.3

17. The company’s coffee crop for 1998-99 totalled 8079 tonnes, an all-time record. The

THEORY
increase over the previous year’s production of 5830 tonnes was 38.58%. The previous
highest crop was 6089 tonnes in 1970-71. The company had fixed a target of 8000 tonnes
to be realized by the year 2000-01, and this has been achieved two years earlier, thanks to

PRACTICE EXERCISE CLASS EXERCISE


the emphasis laid on the key areas of irrigation, replacement of unproductive coffee bushes,
intensive refilling and improved agricultural practices. It is now our endeavour to reach the
target of 10000 tonnes in the year 2001-02.
Which one of the following would contribute most to making the target of 10000 tonnes in
2001-02 unrealistic?
1) The potential of the productivity enhancing measures implemented up to now has been
exhausted.
2) The total company land under coffee has remained constant since 1969 when an estate
in the Nilgiri Hills was acquired.
3) The sensitivity of the crop to climatic factors makes predictions about production uncertain.
4) The target-setting procedures in the company have been proved to be sound by the
achievement of the 8000 tonne target.
(Past CAT question)

18. Animals in general are shrewd in proportion as they cultivate society. Elephants and beavers
show the greatest signs of this sagacity when they are together in large numbers, but when
man invades their communities they lose all their spirit of industry. Among insects, the la-
bours of the bee and the ant have attracted the attention and admiration of naturalists, but
all their sagacity seems to be lost upon separation and a single bee or ant seems destitute
of every degree of industry. It becomes the most stupid insect imaginable, and it languishes
and soon dies.
Which of the following can be inferred from the above passage?
1) Humankind is responsible for the destruction of the natural habitat of animals and insects.
2) Animals, in general, are unable to function effectively outside their normal social environ-
ment.
3) Naturalists have great admiration for bees and ants, despite their lack of industry upon
separation.
4) Elephants and beavers are smarter than bees and ants in the presence of human beings.
(Past CAT question)

99
CATapult
VERBAL REASONING

PRACTICE EXERCISE

Directions: Each passage below is followed by a question. Read the passage carefully and answer
the question that follows.

1. One can make an analogy comparing the law and order scene today with that of a fishing
net. It may catch the salmon and mackerel but always lets the whales and the sharks break
through.
The author most probably believes:
1) The law must be brutal and use the theory ‘guilty before proven otherwise’.
2) Capital punishment is the only way to deter crime
3) The author is very concerned about environmental hazards that fish face.
4) The more powerful members of society invariably get away with their crimes.

2. Mr. Gates is a futuristic high technology visionary first and a philanthropist next. The devel-
oping world looks to people like him to blend social development and information technology.
The synergy can only have a positive fall-out for the disadvantaged whom Mr. Gates seeks
to help.
Mr. Gates’s strengths in technology are:
1) rivalled by his belief on philanthropy.
2) a useful asset to the disadvantaged.
3) likely to prove more useful if combined with social development.
4) Both (2) and (3).

3. Indians are not as creative as they should be. It goes back to their education system based
on the memorization of facts. That is important, but we tend to overemphasize it. What we
need is right-brain thinking, questioning and out-of-the-box ideas. Innovation is more than
creativity. Creativity is more about good ideas, but innovation is about bringing this creativity
into the market, which is entrepreneurial activity. On the entrepreneurial side we have some
strengths. But, creativity is perhaps where we need to develop more, since as a country, we
are not as innovative as we could be.
The argument presupposes which of the following?
1) Creativity is necessary for being more entrepreneurial as a nation.
2) Entrepreneurship is necessary for being more creative as a nation.
3) Creativity is necessary for being more innovative as a nation.
4) All of the above.

100
Chapter VA
CRITICAL REASONING 1 4.3

4. Even educated professionals may know the basics but see no need to keep up to date –

PRACTICE EXERCISE CLASS EXERCISE THEORY


having no idea of the interest rates on their credit cards, the fees on their mutual funds or
how their investments are doing. But in both America and Britain low personal saving rates
(negative in America, indeed) and record numbers of personal bankruptcies do not bode well.
If people are to take charge of their pensions, shouldn’t they know a little more?
Which of the following is probably the conclusion towards which the author is moving?
1) Educated professionals are finding it increasingly difficult to understand the intricacies of
the savings and investments scenario.
2) The government in America and Britain should take corrective measures to prevent the
increasing events of personal bankruptcies.
3) People should increase their financial literacy, in order to take prudent decisions on fi-
nancial matters.
4) Educated professionals should spread the word about better financial planning.

5. The current spate of sting operations against politicians only serves to boost the viewership
of the channels. Little or no money is ever recovered. The common man, crushed under the
burden of taxes, gains nothing from such exposes. If TV channels are genuinely interested in
addressing corruption, they need to target the various departments of municipal corporations.
Which of the following assumption is made in the above passage?
1) Sting operations are the best means to keep a check on corruption.
2) The common man deals more often with municipal bodies rather than those operating at
higher levels of government.
3) The need to create awareness among the viewers is important to tackle the issue of
corruption.
4) Sting operations are effective as they make the politicians cough up the money they had
embezzled.

6. One is surprised by the hue and cry created over the issue of the use of human bones in
making ayurvedic preparations at Ramdev pharmacy. The Indian politicians indeed maintain
a double standard – they can tolerate the pesticides used in Coke and Pepsi, but they are
allergic to useful traces of animal organs in Indian ayurvedic medicines.
Which of the following statements is most likely to weaken the above argument?
1) Coke and Pepsi are consumed by a large number of people as compared to ayurvedic
medicines which are consumed by a relatively smaller section of Indians.
2) Medicines play a more vital role in saving human lives and maintaining a healthy body as
against beverages.
3) The quantity of pesticides used in Coke and Pepsi has been approved by the Food and
Drug Administration of India as being safe for human consumption.
4) Beverage companies are multinationals and thereby exercise a greater economic influence
than Ramdev pharmacy.

101
CATapult
VERBAL REASONING

7. When Herodotus wrote the history of the ancient world, he mixed the lives of the famous
with those of the ordinary, for, he wanted not only to record the events that shaped his world
but also to give his readers a taste of life in past times and far-away places.
Which of the following can be inferred from the above?
1) Herodotus performed the tasks of both historian and journalist.
2) Only historians are equipped to reconstruct times and social lives.
3) Herodotus’ history was based on scanty historical evidence and more on hearsay and
gossip.
4) Herodotus lived in interesting times.

8. The incidence of stress-related diseases in management workers is increasing at an alarming


rate. Between 1990 and 1995, the increase in lost working days in industry for men was up
by more than 13% for cardiac disease, 12% for cerebrovascular disease, 16% for psychosis,
more than 4% for hypertensive disease and 3% for neurosis.
For women workers, comparable figures were a 25% increase for cardiac disease, 73% for
cerebrovascular disease, 1% for psychosis, more than 3% for hypertensive disease, 2% for
neurosis and 6% for ulcers.
Prime causes of these increases tend to be corporate merger mania – takeover, acquisitions,
amalgamations, buyouts and co-ventures.
Which of the following conclusions cannot be deduced from the above?
1) More women suffer from cerebrovascular diseases related to stress than men.
2) Stress in industry has a direct effect on the health of management staff.
3) More and more working days are being lost due to stress-related disease.
4) The rate at which the incidence of cardiac disease for men is increasing is less than the
rate for women.

9. From 1920 to 1950, the amount of food production per worker and per hour increased two
fold. From 1950 to 1980, food production per worker and per hour increased 2.3 times.
Each of the following, if true, could help to account for this trend except:
1) The number of farm workers increased.
2) The use of mechanical technology in food production increased.
3) The use of chemical fertilisers increased.
4) Introduction of genetically modified crops resulted in greater yields.

102
Chapter VA
CRITICAL REASONING 1 4.3

10. Depression is explained as the deepest crest of sadness. It usually occurs during drastic times

PRACTICE EXERCISE CLASS EXERCISE THEORY


of people’s lives – after a loss (death, for example), relationship problems, school, grades,
stress, etc. Anything can trigger depression – big or small. It depends on the person, the
situation and the events that are happening within the person’s life.
Which of the following is the author least likely to agree with?
1) Depression is caused by events that disturb the sufferer emotionally.
2) All kinds of anxiety can culminate in depression.
3) “Drastic times” are events that have the potential to shake you up.
4) Failure in examinations drives all people to depression.

11. California is home to thousands of health clubs where people go to get rid of their energy.
Let us capture that energy by adding an electric generator to every treadmill. Health clubs
would then become mini-power plants and working out would reduce your electricity bill as
well as your weight.
Which of the following, if true, would most cast doubts on the conclusion in the passage?
1) It would be expensive to transmit the power generated from such a method.
2) Such an idea may not necessarily work in areas where there aren’t many health clubs.
3) The reduction in electricity bill may not be very much considering the small amounts of
power the treadmills will be able to generate.
4) Not all health clubs are equipped with sophisticated instruments like treadmills.

12. In one-third of the schools inspected last year, there was “substantial underachievement” in
the teaching of computing. Why does teaching computing remain such an uphill struggle for
many schools after years on the curriculum? Though IT infiltrates every aspect of everyday
life, applying it effectively to lessons is another matter.
Which of the following would constitute the most important reason for the underachievement
in teaching computing?
1) School teachers are essentially novices and are not competent enough to teach young
students.
2) Most students are computer-literate and do not find computing courses very interesting.
3) Computer infrastructure at many schools is not up-to-date, with schools having to operate
on a limited budget.
4) Teaching at schools is not a high-paying job and therefore many people prefer other ca-
reers to teaching.

103
CATapult
VERBAL REASONING

13. It’s an idea as old as fashion retail itself – big companies hire young, talented designers to
create for lines that are targeted to the not-quite-ready-for designer set. Ideally it’s a win-
win situation for everyone involved: the designers get a salary with which they can start their
own lines.
Which of the following statements is likely to logically conclude the passage?
1) It used to be one of fashion’s little secrets but today’s young egalitarian designers are
proud of creating clothes for every segment of the market.
2) The labels get new talent, while consumers get designer products for less.
3) Even ordinary people get to wear labels at a discounted price.
4) However, some of the new designers can never realize their goals.

14. Opponents of laws that prevent heart patients from participating in any adventure sports
argue that society should accord people the right to take risks as long as they do not harm
others.
Which of the following reasons would induce the opponents of the law to accept it?
1) Adventure sports have, on many occasions, led to the death of participants.
2) The insurance companies will have to pay if these patients get injured or killed.
3) Suicide is considered illegal and adventure sports are akin to suicide for heart patients.
4) None of the above.

15. Technical experts flout a fundamental scientific guideline – good applied science requires
a high degree of certainty about the basic facts at hand, and especially their meaning. We
have not yet reached this point with regard to our understanding of the cause and effects of
earthquakes. How then can we allow experts to certify a Tehri dam safe in spite of its being
built over a known earthquake zone?
The writer is mistrustful of expert opinion on the Tehri dam because:
1) the experts have a vested interest in supporting the Tehri dam project.
2) the experts do not understand the ecology of the Tehri dam site.
3) there is not enough knowledge about earthquakes to decide the issue of safety.
4) the experts do not take the issue of safety of residents seriously.

16. The walkers that many parents buy for their infants may do more harm than good, according
to the Academy of Pediatrics, which has decided on banning their manufacture and sale.
There is no evidence that these devices help infants to learn to walk independently; on the
contrary, they say, they can delay normal motor development.
Which of the following effectively questions the decision taken by the Academy of Pediatrics?
1) Apart from one or two reports of serious injury, the majority of infants who have walkers
learn to walk eventually.
2) For infants who show mental and physical lag in development, walkers help them become
self-reliant.
3) A statutory warning would be more effective in tackling the situation.
4) None of the above.
104
Chapter VA
CRITICAL REASONING 1 4.3

17. Singapore is now an affluent and rapidly ageing society. It is also now searching for ways

PRACTICE EXERCISE CLASS EXERCISE THEORY


to regain competitiveness in the face of globalization and competition. Even as the need for
adequate social security provision is growing, Singapore’s strong revealed preference for use
of the cultural provident fund for socio-economic engineering and for political control are
creating many dilemmas centring around governance, design of schemes, investment policies
and whether to shift to a more appropriate multi-tier system that takes care of such concerns
in a more effective manner.
The premise for the author’s assertion that Singapore is looking for a change in its way of
functioning is:
1) Singapore is seeking a more effective way of governance to meet its social obligations
within the country and to regain its competitive edge in the global set-up.
2) Over the years, apart from retirement benefits, there have been other financing arrange-
ments such as housing and health care benefits which have also eaten into the finances
of the government.
3) From the way retirement benefits are dispensed with in Singapore, it can be assumed
that Singapore does not look after its aged very well and is hoping to change that image
with a new multi-tier system in force.
4) The dilemmas emerging from the current system of governance can only be resolved by
substituting the old single-tier system with a new-multi-tieredone.

18. Nemoria arizonaria, a kind of inchworm found on oak trees in the southwestern United States
and Mexico, produces two broods a year. When the first brood hatches in the spring, the oaks
are covered with fuzzy, spiky flowers called catkins. The caterpillars eat these flowers and
end up looking like the catkins. A few months later, a second brood hatches. By this time,
the catkins are gone, and the new brood of caterpillars eat leaves. Instead of resembling
catkins, the summer brood ends up looking like oak twigs.
Which of the following best explains this strange phenomenon?
1) Twigs provide a better camouflage than leaves.
2) The butterfly that the caterpillar finally metamorphoses into looks a bit like catkins and
a bit like leaves.
3) With the changing season each brood of caterpillars changes its appearance to blend in
better with the changing surroundings.
4) The form that a caterpillar takes is determined by what it eats.

105
CATapult
VERBAL REASONING

19. Cultured pearls are pearls grown under tightly controlled conditions by pearl farmers. When
people hear cultured, they tend to think fake. But they are no more inauthentic than an ap-
ple picked from a tree in an orchard. The seed may not have landed there by accident and
grown into a tree unnoticed, but that doesn’t make the fruit any less real.
What is this argument trying to prove?
1) An artificially grown thing can be even more authentic than one that grows naturally on
its own.
2) That which grows naturally is generally considered more authentic than that which is
grown artificially.
3) The authenticity of a thing is not dependent on whether it was deliberately or accidentally
grown.
4) The difference between a fake thing and an authentic one is like the difference between
a deliberately and an accidentally grown thing.

20. Charles Darwin’s hypothesis was that all present-day life traces back to one or a few original
progenitors. Modern biologists are on the same page when they point to the near universality
of the genetic code as strongly favouring the hypothesis of universal common ancestry over
the hypothesis of multiple ancestors. The shared code would be a surprising coincidence
if different groups of organisms stemmed from different start-ups. It would be much more
probable if all current life traced back to a single origination.
If a form of life were discovered that had a very different sort of genetic code than the rest,
how would this affect the above argument?
1) It would weaken the argument.
2) It would strengthen the argument.
3) It would have no effect on the argument.
4) Its effect cannot be determined from the given information.

21. India is strangely deprived of wild, native coniferous trees. Conifers grow very well in India –
on plantations. But apart from a few Eurasian types in the Himalayas, the only living native
conifer is Nageia wallichiana of the Southern Hemisphere podocarp family in the Western
Ghats in the southwest of the country.
Which of these is most likely to be the reason for the lack of native conifers in India?
1) Conifers are native to colder climates, and cannot grow in tropical countries like India.
2) Conifers do not grow well in the type of soil found throughout most of India.
3) Conifers need regular rainfall, so they are unsuited to India, where it rains almost exclu-
sively during the monsoon months.
4) Non-coniferous trees are more commonly grown in India instead of conifers, as the former
are more important in Indian culture.

106
CATapult
VERBAL REASONING

VA-4.4 CRITICAL REASONING 2


THEORY

Fallacies in Reasoning
Fallacies are common errors in reasoning that will undermine the logic of the argument. Fallacies
can be either illegitimate arguments or irrelevant points. Some of the common fallacies are:

Hasty generalization
(Dicto Simpliciter, also called ‘Jumping to Conclusions’ or ‘Converse Accident’)

Hasty generalization occurs when assumptions about a whole group or range of cases are based
on a sample that is inadequate (usually because it is atypical or too small). Stereotypes about
people, for example: Film stars are immoral, Wealthy people are snobs, etc. are a common example
of the principle underlying hasty generalization.

Missing the point


(Ignoratio Elenchi or ‘Irrelevant Conclusion’)

The premises of an argument may support a particular conclusion—but not the conclusion that
the arguer actually draws.

Example: I say we should support Reservation. Upper castes have run the country for years. They
run most of the government and industry today. You can’t deny that this sort of discrimination is
intolerable.
(The author has proven that there is discrimination, but not that Reservation will end that
discrimination or that we should support Reservation.)

False Cause
(Post hoc ergo propter hoc)

This fallacy gets its name from the Latin phrase post hoc, ergo propter hoc, which translates as
‘after this, therefore because of this’. In post hoc if event B occurs after event A, the arguer
assumes that event A has caused event B. A temporal relationship is taken to be cause-effect.
But sometimes two events that seem related in time aren’t really related as cause and event. That
is, correlation isn’t the same thing as causation.

Example: The shutdown of internet in the state has contributed to a decrease in crime.
The shutdown of internet might or might not be a factor in the decreasing crime, but the argument
hasn’t shown us that one caused the other.
107
CATapult
VERBAL REASONING

Slippery slope

The arguer claims that a sort of chain reaction, usually ending in some dire consequence, will take
place, but there’s really not enough evidence for that assumption. The arguer asserts that if we
take even one step onto the ‘slippery slope,’ we will end up sliding all the way to the bottom; he
or she assumes we can’t stop partway down the hill.

Example: Legalizing prostitution is undesirable because it would cause more marriages to break up,
which would in turn cause the breakdown of the family, which would finally result in the destruction
of civilization.

Appeal to authority
(Ad verecundiam)

The ad verecundiam fallacy concerns appeals to authority or expertise. Fundamentally, the fallacy
involves accepting as evidence for a proposition the pronouncement of someone who is taken to
be an authority but is not really an authority —when, for example, celebrities endorse commercial
products or social movements. Similarly, when there is controversy, and authorities are divided, it
is an error to base one’s view on the authority of just some of them.

Appeal to popularity
(Ad populum)

The fallacy ad populum is similar to the ad verecundiam, the difference being that the source
appealed to is popular opinion, or common knowledge, rather than a specified authority.

Example: Iphones are the most popular handsets in the market today. Why don’t you buy an iphone?
It is the best phone you can buy today.

Often in arguments like this, if the premises are generally true, they may provide only scant support
for their conclusions because something that is widely practised or believed is not compelling
evidence that it is true or that it should be done. There are few subjects on which the general
public can be said to hold authoritative opinions. Another version of the ad populum fallacy is
known as ‘playing to the gallery’ in which a speaker seeks acceptance for his view by arousing
relevant prejudices and emotions in his audience in lieu of presenting it with good evidence.

108
Chapter VA
CRITICAL REASONING 2 4.4

Appealing to feelings or prejudices

THEORY
(Ad hominem and tu quoque)

PRACTICE EXERCISE CLASS EXERCISE


Like the appeal to authority and ad populum fallacies, the ad hominem (‘against the person’) and tu
quoque (‘you, too!’) fallacies focus our attention on people rather than on arguments or evidence.
In both of these arguments, the conclusion is usually “You shouldn’t believe someone’s argument.”
The reason for not believing someone is that he/she is either a bad person (ad hominem) or a
hypocrite (tu quoque). In an ad hominem argument, the arguer attacks his or her opponent instead
of the opponent’s argument. In a tu quoque argument, the arguer points out that the opponent has
actually done the thing he or she is arguing against, and so the opponent’s argument shouldn’t
be listened to.

Example:
Imagine that your parents have explained to you why you shouldn’t smoke, and they’ve given a lot
of good reasons—the damage to your health, the cost, and so forth. You reply, “I won’t accept your
argument, because you used to smoke when you were my age. You did it, too!”
The fact that your parents have done the thing they are condemning has no bearing on the
premises they put forward in their argument (smoking harms your health and is very expensive),
so your response is fallacious.

Straw man

One way of making our own arguments stronger is to anticipate and respond in advance to the
arguments that an opponent might make. In the straw man fallacy, the arguer sets up a weak
version of the opponent’s position and tries to score points by knocking it down. But just as being
able to knock down a straw man (like a scarecrow) isn’t very impressive, defeating a watered-down
version of your opponent’s argument isn’t very impressive either. Straw man fallacy, occurs when
an opponent’s point of view is distorted in order to make it easier to refute.

For example, suppose your argument is: ‘Industrialization is the cause of global warming.’ The
opponent would resort to straw man fallacy and state that you believe that all ills that beset
mankind are due to industrialization and prove that it is false; hence global warming is not due
to industrialization. And even if you also believed that “all ills that beset mankind are due to
industrialization”, your original argument that “Industrialization is the cause of global warming” is
not false.

109
CATapult
VERBAL REASONING

Red herring

Red herring is going off on a tangent to distract one away from the argument.

Example:
Son: “Dad, it’s really hard to make a living on my salary.” Father: “Consider yourself lucky, son.
When I was your age, my salary was less than one tenth of what you are getting today.”
When we lay it out this way, it’s pretty obvious that the arguer went off on a tangent—the fact
that dad’s salary was one tenth of the son’s salary sidetracks the issue that son is unable to
make ends meet today.

False dichotomy

In false dichotomy, the arguer sets up the situation so it looks like there are only two choices.
The arguer then eliminates one of the choices, so it seems that we are left with only one option:
the one the arguer wanted us to pick in the first place. But often there are really many different
options, not just two—and if we thought about them all, we might not be so quick to pick the
one the arguer recommends.

Example: “We can either stop using cars or destroy the earth.”

Begging the question

A complicated fallacy; it comes in several forms and can be harder to detect than many of the
other fallacies we’ve discussed. Basically, an argument that begs the question asks the reader
to simply accept the conclusion without providing real evidence; the argument either relies on a
premise that says the same thing as the conclusion (‘circular reasoning’), or simply ignores an
important (but questionable) assumption that the argument rests on. Sometimes people use the
phrase ‘beg the question’ as a sort of general criticism of arguments, to mean that an arguer hasn’t
given very good reasons for a conclusion, but that’s not the meaning we’re going to discuss here.

Example: Active euthanasia is morally acceptable. It is a decent, ethical thing to help another human
being escape suffering through death.
Premise: It is a decent, ethical thing to help another human being escape suffering through death.
Conclusion: Active euthanasia is morally acceptable.

If we ‘translate’ the premise, we’ll see that the arguer has really just said the same thing twice:
‘decent, ethical’ means pretty much the same thing as ‘morally acceptable’, and ‘help another
human being escape suffering through death’ means something pretty similar to ‘active euthanasia’.
So the premise basically says, ‘active euthanasia is morally acceptable’, just like the conclusion
does. The arguer hasn’t yet given us any real reasons why euthanasia is acceptable; instead, she
has left us asking “well, really, why do you think active euthanasia is acceptable?” Her argument
‘begs’ (that is, evades) the real question.

110
Chapter VA
CRITICAL REASONING 2 4.4

CLASS EXERCISE

Directions: Identify the fallacies, if any, in the statements given below.


Fallacy 1:
Studies have shown that women who were taking combined Hormone Replacement Therapy (HRT)
also had a lower-than-average incidence of Coronary Heart Disease (CHD). Leading doctors now
propose that HRT is protective against CHD.

Fallacy 2:
The stock markets in the country crashed immediately after the results of the election were
declared. It is evident that the markets are not in favour of the new government that the people
have elected.

Fallacy 3:
My uncle drank alcohol all his life and never suffered from a single stroke or liver disease. He died
at ninety, when he got hit by a truck, so drinking alcohol is not as injurious as it is made out to be.

Fallacy 4:
She is the most intelligent person I know because she is much smarter than everyone else I know.

Fallacy 5:
It is safer to fly a plane than to drive a motorbike since there are fewer airplane accidents than
motorbike accidents.

Fallacy 6:
His demands for raising the taxes do not make any economic sense because he is a lazy and
vain man!

Fallacy 7:
If you were a patriotic Indian, you would vote for the Aam Aadmi Party.

Fallacy 8:
Smoking may be harmful for smokers, but what other employment can we provide to the millions
of workers employed by tobacco companies?

Fallacy 9:
If we give in to our employers’ demands today, tomorrow they will make us do everything they
want. We will practically be their slaves!

111
CATapult
VERBAL REASONING

Directions: Each passage below is followed by a question. Read the passage carefully and answer
the question that follows.

1. Organic farming, on an average, has lower input costs than conventional farming because it
excludes or strictly limits the use of pesticides and insecticides. But the yields are lower than
conventional farming since it is very knowledge-intensive and many farmers are still relatively
inexperienced with respect to the principles of organic farming.
Which of the following best supports the idea that organic farming can be profitable?
1) Many organic farms are coming up not only in developed countries, where there is a
movement towards sustainable development, but also in developing countries.
2) The yields from conventional agriculture have reached their limits and will not increase
further.
3) Spurred by the movement towards environmentally sustainable and pesticide-free food,
many consumers and restaurants are willing to pay high prices for organic products.
4) Prolonged use of pesticides is known to make pests resistant to them and call for increased
dosage and hence increased costs.

2. Normally, an increase in the price of a product decreases its sales except when the price
increase accompanies an improvement in the product. Veblen goods are commodities for
which people’s preference for buying them increases with an increase in their price, even if
there is no change in the product itself.
Which of the following, if true, best explains the anomaly described above?
1) The market for Veblen goods is characterized by an extremely wide range of competing
products.
2) Consumers selecting Veblen goods in a store often use the price charged as their main
guide to the product’s quality.
3) Retailers and producers of Veblen goods can generally increase the sales of a particular
product temporarily by introducing a price discount.
4) Consumers who purchase Veblen goods regularly generally have strong opinions about the
brands they prefer.

112
Chapter VA
CRITICAL REASONING 2 4.4

3. In order to reduce non-biodegradable waste, the municipality of a certain town is promoting

THEORY
the widespread use of paper rather than plastic. The municipality plans to ban the sale of
disposable plastic goods for which substitutes made of paper exist. The municipality argues
that paper goods are environmentally preferable, since most paper is entirely biodegradable.

PRACTICE EXERCISE CLASS EXERCISE


Which of the following, if true, indicates that the plan to ban the sale of disposable plastic
goods is ill-suited to the municipality’s environmental goals?
1) The paper factory at which most of the townspeople are employed plans to increase
production of biodegradable paper goods.
2) After other towns enacted similar bans on the sale of plastic goods, the environmental
benefits were not discernible for several years.
3) Since most townspeople prefer plastic goods to paper goods in many instances, they are
likely to purchase them in neighbouring towns where plastic goods are available for sale.
4) Products other than those derived from wood pulp are often used in the manufacture of
paper goods that are entirely biodegradable.

4. One of the major problems faced by companies is the absentee rate. Studies have shown
that the more frequently employees exercise during working hours each week, the fewer
sick days they take. Even employees who exercise only once a week during working hours
take less sick time than those who do not exercise. Therefore, if companies started fitness
programs the absentee rate in those companies would decrease significantly.
Which of the following, if true, most seriously weakens the argument above?
1) Employees who exercise during working hours occasionally fall asleep for short periods
of time after they exercise.
2) Employees who are frequently absent are the least likely to cooperate with or to join a
corporate fitness program.
3) Employees who exercise in their company’s fitness program use their working time no
more productively than those who do not exercise.
4) Employees who exercise during working hours take slightly longer lunch breaks than em-
ployees who do not exercise.

5. There are no unselfish acts; even acts of philanthropy give mental satisfaction to the donor
and thus are, in effect, selfish acts.
Which of the following arguments most resembles the above argument in its line of reasoning?
1) People who adopt children are benefitting as much as the children whom they adopted,
since the pleasure from having parents and having children is one and the same.
2) Even abstaining from voting or not being interested in politics is a political act, since it
expresses one’s stance with respect to the process of collective decision-making.
3) Possessing firearms means assenting to the right to use them, since there is no reason
why one would have them if one would not use them.
4) To take up helping others as a way life, one should first be no longer in need of help
oneself.

113
CATapult
VERBAL REASONING

6. During the last financial year, the profits of a car manufacturer fell drastically; yet its stock
price rose, contrary to the general trend of stock prices moving in the same direction as
profits.
Which of the following best explains the apparent discrepancy stated above?
1) The company’s profits fell by a lower rate than that of its competitors.
2) The company invested in a new production facility that was expected to double its capacity
and this reflected positive long-term prospects of the company.
3) The automobile market grew by a greater percentage last year, compared to the year
before that.
4) With the economy poised to grow at a healthy rate next year, automobile sales are ex-
pected to also grow considerably.

7. A meritocracy, where the best and most hardworking minds stand to gain the most monetarily,
is the only way to ensure that each person contributes his/her best to society.
The above argument is flawed because it assumes that:
1) Many people are interested in contributing their best to society.
2) Money is the only factor that will motivate the best and most hardworking minds.
3) Society needs people to contribute their best to it.
4) Society needs all people to contribute their best to it.

8. Management experts have long predicted the demise of the standard 9-to-5 workday. Thanks
to the Internet and mobile technology, we can now work where and when we want, they
argue. The reality is that while flexible work arrangements have become more popular, few
companies have an official policy or program. Traditionally, managers were reluctant to have
people work remotely because of lack of trust. Even those bosses who trust their employ-
ees worry about allowing productivity to decline. Still, more managers and organisations are
seeing the benefits of non-traditional schedules.
Which of the following best supports the idea of businesses encouraging non-traditional
schedules?
1) Non-traditional schedules can be introduced on a trial and error basis before businesses
can formulate a formal policy.
2) Research has shown that when people are given the flexibility they need, they meet goals
more easily, they’re absent or tardy less often, and their morale goes up.
3) Not all employees will want a flexible schedule; studies have a shown that a significant
number of people still prefer a traditional schedule.
4) Businesses need to assess the impact of flexible schedules before they offer the option
to their employees.

114
Chapter VA
CRITICAL REASONING 2 4.4

9. Group discussions (GDs) are an essential part of the selection process for management

THEORY
schools since they help institutes decide whether candidates have the requisite interpersonal
and communication skills for a career in management.
The argument above is flawed because:

PRACTICE EXERCISE CLASS EXERCISE


1) It uses interpersonal and communication skills interchangeably without accounting for the
difference in their meanings.
2) It assumes that people who do well in group discussions can maintain the same level of
performance throughout their careers.
3) It assumes that the kind of the interpersonal and communication skills required for a career
in management are similar as well as limited to those brought out in group discussions.
4) It assumes that candidates who do not exhibit interpersonal and communication skills in
the group discussions can never develop them later.

10. With more and more people eating out and growing increasingly obese, several state gov-
ernments are trying to help people eat healthier by forcing fast-food restaurants to divulge
nutritional information on their menus.
Which of the following, if true, casts most serious doubts on the effectiveness of the sug-
gested plan of action to make people eat healthier?
1) Studies have shown that people tend to eat fast food for convenience rather than for
taste.
2) Studies have shown that eating habits die hard and adding the calorie count to menus
has no impact on diners’ choices.
3) As more and more states debate such laws, food vendors are pre-emptively modifying
their menus to both lighten existing options and add healthier foods.
4) Food choices are cultural choices and the changing food preferences reflect a change in
culture rather than a change in awareness about health.

11. It has long been the natural order that the teen and young adults should be dishonest in
their dealings with their parents – for example, loudly announcing that they’re going studying
at Ciara’s house only to shed their uniforms in the nearest bush and go drinking in a field.
As it was, and always will be. They are much more forthright with their older siblings though,
who always seem to know what they are up to. The parents, however, remain woefully un-
informed.
Which of the following has been assumed to arrive at the above conclusion?
1) Teens are more forthright with their older siblings than their parents because the gener-
ation gap is less with the former.
2) Parents take it for granted that their children will lie to them and hide things from them.
3) Teenagers don’t want their parents to know what they are up to only when they are up
to something forbidden.
4) Parents cannot get to know what kids are actually up to, unless their kids tell them.

115
CATapult
VERBAL REASONING

12. In the book of John (4:24), the scriptures state: “The Supreme Being is Spirit: and they that
worship him must worship him in spirit and in truth”. Again, in the book of Genesis (1:2),
the scriptures state: “And the earth was without form, and void; and darkness was upon the
face of the deep. And the Spirit of the Supreme Being moved upon the face of the waters”.
Which of the following conclusions is best supported by the information above?
1) The scriptures believe that it was the Supreme Being that formed the earth.
2) From the perspective of religion, just as it is from the perspective of science, the earth
existed before any living entity.
3) From the perspective of religion, the term ‘spirit’ can be used to identify the existence of
different types of entities.
4) The scriptures talk about how the Supreme Being came into existence and how he created
man.

13. How I Met Your Mother’s (HIMYM) title sequence starts with a dated image—of a 20-something
Lily Aldrin holding a disposable camera. But that datedness actually fits the show perfectly.
Since 2005, the year HIMYM premiered, sitcoms have gotten pretty innovative. 30 Rock infused
the format with unprecedented levels of snarky dialogue and clever satire. Single-camera
shows like The Office and Modern Family introduced new stylistic sensibilities and satirized
reality TV tropes. Non-network comedies like Louie and Girls used stark realism to achieve
biting poignancy. And all this while, HIMYM has been the old soul of network comedy, and
that’s why it’s so beloved—even if critics often scoffed at it.
Which of the following inferences cannot be drawn from the argument above?
1) The audience craved new sitcom genres around 2005.
2) HIMYM is not a single-camera show.
3) The datedness of HIMYM is what made it work.
4) A disposable camera is considered an old-fashioned device today.

14. Using new computer software to observe and record people’s faces, scientists mapped no
fewer than 21 emotional states, including apparently contradictory examples such as “happily
disgusted” and “sadly angry”. The research more than triples the number of known emotional
facial expressions and could help medical specialists improve the diagnosis and treatment of
psychiatric disorders.
The above conclusion depends on which of the following assumptions?
1) These 21 emotions are expressed in nearly the same way by everyone, irrespective of
their mental health.
2) The six basic emotions – happy, sad, fearful, angry, surprised and disgusted – did not
substantially cover the range of feelings we convey with our facial expressions.
3) Tagging prominent landmarks for facial muscles, such as the corners of the mouth or the
outer edges of the eyebrows, will enable scientists to match emotions to movement.
4) The brain is like a computer and scientists want to decode the algorithm in our brains
that makes us recognise facial expressions.

116
Chapter VA
CRITICAL REASONING 2 4.4

15. If we reduce the number of hours we spend in front of our television or computer screens,

THEORY
and step out of the house and do some physical activities, very soon we will start enjoying
our outdoor pursuits and will not want to be restricted to the four walls of our homes for
long.

PRACTICE EXERCISE CLASS EXERCISE


Which of the following mimics the reasoning of the main argument?
1) If Sheetal starts studying for her exam early this time, rather than wait till the last minute,
she will score well and that will motivate her to do even better next time.
2) If organisations spend more on their employees than they do on marketing, their employ-
ees will be more than happy to act as endorsers for the organisations.
3) If you stop fighting with your little sister and talk to her instead, you will realize she is
a wonderful person and you will cease to have any problems with her.
4) If a farmer overfeeds his cows he will soon realise that instead of giving more milk they
will bloat and eventually die of suffocation caused by the pressure of unreleased frothy
gas on their heart or lungs.

117
CATapult
VERBAL REASONING

PRACTICE EXERCISE-1

Directions: Read the short passages and answer the questions that follow.

1. Scientists studying how diseases spread believe that there are many parallels between com-
puter viruses and biological ones. Contrary to the idea that computer viruses immediately
explode into a pandemic, the scientists found the infection rate starts out very slowly among
a small group of friends. A computer virus can exhibit clique behaviour, with pairs of connect-
ed individuals sharing many common neighbours, reducing the opportunities for secondary
infection events.
Which of the following can be logically inferred from the passage?
1) Certain computer viruses, which can mail themselves from one computer to another, may
spread very fast.
2) A disease is also in the nature of a program whose objective is to reproduce itself as
many times as possible.
3) Contagious diseases can now be cured by studying how computer viruses spread among
Internet users.
4) Diseases may spread more slowly when contact is mainly local, compared to well-mixed
populations.

2. The convergence of media and technology, long predicted but not yet fulfilled, is at last
showing signs of happening – with high-speed Internet access making much of it possible.
With more households going in for broadband, faster Internet connections are changing the
movie, music, telephone, computer and cable businesses. The battles brought on by these
changes are likely to occupy the media and technology industries in the coming years.
Which one of the following does NOT illustrate the battle between media and technology?
1) Record companies ratcheted up the copyright battles in 2004 by suing people who share
music files over the Internet.
2) The movie industry is searching for technologies that will make copying movies more dif-
ficult, even as high-speed Internet connections make downloading movies more feasible.
3) Electronics manufacturers are beginning the fight over the technical standard that will
apply to the next generation of DVD’s for use with high-definition television.
4) Music aficionados are concerned that the current popularity of MP3 players is resulting in
the deterioration of the sound quality of music.

118
Chapter VA
CRITICAL REASONING 2 4.4

3. Moral action was to flow from reason and not be controlled by any external power. Therefore,

PRACTICE EXERCISE CLASS EXERCISE THEORY


the Confucian idea of group can be called ethical and normative. These notions of group and
ethics tended to relate the characters of individuals from different social levels and were of
great perduring force. This enabled feudal economies to develop and flourish. On the basis of
this education in a strong sense of the group, there emerged in Chinese history many people
with lofty ideas. This notion of group became a pillar for the harmony of families, the unity
of the state and the survival of the nation.
Which of the following options would weaken the argument made in the passage above?
1) Confucianism was marked by a strong sense of hierarchical and patriarchal relations.
2) Should an individual’s words and deeds violate the rules of his social status the individual
must be severely punished.
3) The idea of the group and ethics as a whole did not strive for individual rights, suppressed
individuality and deprived inferiors of rights, which, in turn, generated social instability.
4) The obligations one was called to fulfil became tools to preserve the privileges of the
upper hierarchy.

4. The problem comes from the sharing of the pie. The pie may grow faster with appropriate-
policies, but some may get a smaller share or even less than before in absolute terms. For
example, land reforms may benefit a country, but large landowners are bound to lose. Trade
liberalization may enhance efficiency and promote growth, but producers and workers in hith-
erto protected and inefficient sectors are likely to suffer. Reducing high public expenditure
per student in higher education and correspondingly increasing the public outlay on primary
education are likely to yield high dividends in terms of human skill formation and growth.
Which of the following would be a strain in which the author would have continued?
1) But those at whom such a move is directed will still work as child labourers without
attending school.
2) Yet it places a burden on the middle class which are the greatest beneficiaries of higher
education.
3) But the poor will still not get jobs.
4) All of the above.

119
CATapult
VERBAL REASONING

5. Joshua Silver, a socially conscious Oxford University physics professor, has invented a pair of
corrective glasses that can be adjusted on the spot to fit 90% of prescriptions. The glasses
have hollow fluid-filled lenses whose curvature can be altered pneumatically with the turn
of a knob. Silver has dubbed his invention Adaptive Spectacles, and he hopes they can be
distributed in parts of the world where optometrists and prescription lenses are hard to come
by.
Inspite of this great breakthrough Adaptive Spectacles did not create as much as a dent in
the market. Which of the following, if true, could explain this?
1) The knob can be adjusted only at an opthalmologist’s dispensary.
2) Since fluid expands with increase in temperature, a fluctuation in the temperature affects
the spectacles’ adaptability.
3) The fluid used in the lenses has to be changed after a given period of time as it tends
to get milky, affecting vision.
4) All of the above.

6. Nearly all scientists agree that global warming is taking place. But, scientists will also readily
tell you that our weather forecasting tools are too primitive for accurate forecasts 100 years
ahead. They use simplistic computer models, not because these tell the truth, but because
they have nothing better. These typically predict some warming. But they represent assump-
tions, not scientific truth.
Which of the following, if true, would most weaken the argument against accurate forecasts?
1) 100 countries across the globe recorded abnormality in seasons but no forecast had been
made.
2) The weather forecasting tools used today were unheard of even a few decades ago.
3) A 100 year period forecast is based on computer simulations using millions of variables
that reflect real complexities.
4) A number of forecasts made a decade ago for the current decade have proved to be
accurate.

120
Chapter VA
CRITICAL REASONING 2 4.4

7. Farmers have failed as stewards of the UK countryside. The last century saw increased pol-

PRACTICE EXERCISE CLASS EXERCISE THEORY


lution, destruction of hedgerows, declining song-bird numbers. The farming industry receives
£3 billion in subsidies every year (DEFRA). Moreover, it is cheaper and more efficient to grow
food abroad, in places like Eastern Europe. It is suggested, the land would be better used
for tourism, recreation and conservation.
Which statement, if true, would suggest that the UK should be using its countryside to grow
food?
1) Current agricultural practices date back to the post-war period. Food security and increased
efficiency is less of an issue in today’s society.
2) Farming contributes £6.65 billion to the British economy every year whereas countryside
tourism is currently worth £1.2 billion.
3) Currently 70% of food in UK is imported. Only 30% of the food consumed comes from
the UK.
4) Farmers are not the only ones to blame for the pollution and other problems affecting
UK’s countryside.

8. Followers can be defined by their behaviour – doing what others want them to do. They are
low in the hierarchy and have less power, authority and influence than their superiors. They
generally go along to get along, particularly with those in higher positions. In the workplace,
they may comply so as not to put money or stature at risk. In the community, they may
comply to preserve collective stability and security – or simply because it’s the easiest thing
to do. In other words, subordinates follow their leaders all the time.
Which of the following, if true, would undermine the argument in the passage?
1) When the leader has gone astray, it has been observed that the followers have decided the
course of action by withholding support from the bad leaders and throwing their weight
behind good ones.
2) The collapse of communism and other totalitarian regimes shows that followers need not
always be the dependent partner in the leader-follower relationship.
3) Followers now want power for themselves and so think of themselves as free agents, not
as dependent underlings.
4) In an era dominated by knowledge workers rather than manual workers, expertise more
often trumps position, indicating that the line dividing leaders from followers is blurring.

121
CATapult
VERBAL REASONING

9. At a time when companies should be taking bigger – but smart – innovation risks, their bias
is in the other direction. From 1990 to 2004, the percentage of major innovations in devel-
opment portfolios dropped from 20.4 to 11.5 – even as the number of growth initiatives
rose. The result is internal traffic jams of safe, incremental innovations that delay all projects,
stress organizations and fail to achieve revenue goals.
Which of the following is a conclusion that can be drawn from the passage?
1) Minor innovations eat into the profit-making goals of a company and impede overall prog-
ress of the company.
2) Companies should increase the proportion of major innovations while carefully managing
the risk.
3) Companies focus on minor innovations instead of major innovations, but these rarely gen-
erate the growth companies seek.
4) Little innovations are necessary for continuous improvement but don’t give companies a
competitive edge.

10. Global financial markets froze in the morning New York trade on Wednesday as projections
of a Republican sweep of the White House and Congress came unstuck over a recount in the
pivotal state of Florida, leaving the presidential election result in balance. The dollar firmed
but pared earlier gains against the Euro after the White House race was put on hold.
In the event of a clear Republican win in America, which of the following predictions about
the dollar is most likely to come true?
1) It will be at an advantage against the Euro.
2) It will rise to some advantage but will eventually slide.
3) It will be stable.
4) It will continue to be volatile and unstable.

11. Inefficiency and its natural corollary, corruption still persists, and nothing symbolizes this
better than the stacks of files that continue to choke the system. But the department of
Income Tax is moving towards computerization in a bid to speed up processes and improve
productivity. But are these developments enough? Why not go a step forward and free the
individual from the whole cumbersome process of filing returns?
Which of the following would most effectively meet the author’s aim?
1) Allowing the individual to make payments through the internet and thus avoiding paper-
work.
2) Introducing a tax on consumption at the point of sale of goods and abolishing income
tax.
3) Introducing a system whereby the government and not the individual is responsible for
correct evaluation and payment of income tax.
4) All of the above.

122
Chapter VA
CRITICAL REASONING 2 4.4

12. The adversarial machinery deserves enormous respect. It has been refined over centuries as

PRACTICE EXERCISE CLASS EXERCISE THEORY


the most thorough, even elegant, formal means of discerning truth and arriving at effective,
balanced judgments in matters of conflict of interest or ethics. It is also extremely expensive,
time-consuming, unpleasant and alienating. What is called adversarialism may be defined as
a profound tendency on the part of individuals, organizations, or societies to routinely resort
to adversarial machinery when it is unnecessary and unproductive, even uncivil. But why?
Why on earth would anyone – particularly an organization – want to employ procedures that
are so unpleasant and alienating, time consuming and expensive?
Which of these statements would best answer the query raised by the passage?
1) People are inherently destructive in nature – and court methods that delay, cost more or
are unpleasant in nature.
2) People are islands and want to be alienated from other human beings.
3) People want to get rid of tension fast and this is the proper and seemingly safest way
to proceed.
4) People do not believe in litigation or democratic processes.

13. Three airlines – IA, JA and SA – operate on the Delhi-Mumbai route. To increase the number
of seats sold, SA reduced its fares and this was emulated by IA and JA immediately. The
general belief was that the volume of air travel between Delhi and Mumbai would increase
as a result.
Which of the following if true would add credence to the general belief?
1) Increase in profitability of three airlines.
2) Extension of the discount scheme to other routes.
3) A study that shows that air travellers in India are price-conscious.
4) A study that shows that as much as 80% of air-travel in India is company-sponsored.
(Past CAT question)
14. According to McNeill, a Brahmin priest was expected to be able to recite at least one of the
Vedas. The practice was essential for several centuries when the Vedas had not yet been
written down. It must have had a selective effect, since priests would have been recruited
from those able or willing to memorize long passages. It must have helped in the dissem-
ination of the work, since a memorized passage can be duplicated many times.
Which one of the following can be inferred from the above passage?
1) Reciting the Vedas was a Brahmin’s obligation.
2) The Vedic priest was like a recorded audio cassette.
3) McNeil studied the behaviour of Brahmin priests.
4) Vedic hymns had not been scripted.
(Past CAT question)

123
CATapult
VERBAL REASONING

15. Developed countries have made adequate provisions for social security for senior citizens.
State insurers (as well as private ones) offer medicare and pension benefits to people who
can no longer earn. In India, with the collapse of the joint family system, the traditional
shelter of the elderly has disappeared. And a State faced with a financial crunch is not in a
position to provide social security. So, it is advisable that the working population give serious
thought to building a financial base for itself.
Which one of the following, if it were to happen, weakens the conclusion drawn in the above
passage the most?
1) The investible income of the working population, as a proportion of its total income, will
grow in the future.
2) The insurance sector is underdeveloped and trends indicate that it will be extensively
privatized in the future.
3) India is on a path of development that will take it to a developed country status, with all
its positive and negative implications.
4) If the working population builds a stronger financial base, there will be no need to revive
the joint family system.
(Past CAT question)

16. Various studies have shown that our forested and hilly regions and, in general, areas where
biodiversity as reflected in the variety of flora - is high, are the places where poverty appears
to be high. And these same areas are also the ones where educational performance seems
to be poor. Therefore, it may be surmised that, even disregarding poverty status, richness in
biodiversity goes hand in hand with educational backwardness.
Which one of the following statements, if true, can be said to best provide supporting evi-
dence for the surmise mentioned in the passage?
1) In regions where there is little variety in flora, educational performance is seen to be as
good as in regions with high variety in flora, when poverty levels are high.
2) Regions which show high biodiversity also exhibit high educational performance, at low
levels of poverty.
3) Regions which show low biodiversity reveal high levels of poverty and poor educational
performance.
4) In regions where there is low biodiversity, at all levels of poverty, educational performance
is seen to be good.
(Past CAT question)

124
Chapter VA
CRITICAL REASONING 2 4.4

17. The audience of CEOs and finance managers many of whom have joined the growing list

PRACTICE EXERCISE CLASS EXERCISE THEORY


of firms offering stock options to staff – would’ve been uneasy listening to Joel Stern at a
seminar on Economic Value Added (EVA) in the capital last week. The founder-partner of
consultancy firm Stern Stewart and the reputed academic who conceived EVA wasn’t a per-
son they could ignore. It isn’t easy to build accountability for performance in a stock option
plan, said Stern. As a result, it gives rise to the free-rider problem. People avoid work and
ride the gravy train secure in the knowledge that their co-workers work hard and drive up
share values, thereby making everyone richer. Stock options have other pitfalls. For instance,
many employees don’t understand how their own work impacts the firm’s performance, leave
alone the share value. ‘So,’ they question, ‘I may work hard, but if the market doesn’t like
the stock, I’m sunk.’
What is the point that the author is trying to make in this passage?
1) Stock options are not motivating enough for the employees to give their best to their
work.
2) Most employees know nothing about the stock market.
3) Stock options only give rise to the free rider problem.
4) Offering employees stock options can sometimes be detrimental to both the firm and its
employees.

18. Reductionism implies that real stuff lies at the bottom, at the level of mere matter: with the
help of this bottom level, every higher form of empirical nature can be fully explained. A few
reductionists assert that not only human consciousness, but also human culture as a whole,
can be explained materialistically. According to them, our genes hold our social life and all
of human culture on a leash. One may infer from the passage that a reductionist would be
most unlikely to:
1) overestimate the importance of the material aspect of life.
2) view human culture as a natural fallout of man’s innate tendencies.
3) view the progress of mankind as a measure of his spiritual progress.
4) disbelieve in the existence of the human soul.

125
CATapult
VERBAL REASONING

19. China’s national legislature moved to amend the Constitution to protect private property rights,
the first time the Communist Party has formally protected private wealth since coming to
power 55 years ago. The change, expected to be enacted early next year, is a milestone in
China’s 25-year economic reform effort. It marks a victory for advocates of China’s emerging
class of entrepreneurs, who have argued for years that the Marxist Constitution discriminates
against them and gives leeway to the police and the courts to seize their property according
to party dictates.
In which case would this legislation be invalid?
1) Government leaders have long warned against steps that would make it easier for well-es-
tablished people to take control of public property and treat it as their own.
2) The changes do not have a direct impact on China’s peasant class. Farm land is still owned
and controlled by the State and leased to farmers.
3) The amendment specifies that only legally obtained property will be thus protected, but
the Communist Party itself determines what is considered ‘legally obtained’.
4) In spite of this amendment, the Communist Party still turns a blind eye to what the police
do on their own initiative.

20. More UK farms are switching to organic manageement. There is a growing demand for organic
produce. In fact, even the take up of government organic farming schemes would indicate
that farmers are in favour. Between 1999 and 2001, more than 2,000 farmers converted
their land to organic farms. Environmentally as well, organic farming improves biodiversity,
both on the farm itself and in uncultivated areas nearby.
Which statement, if true, would suggest that switching to organic farming is in fact bad?
1) The Food Standards Agency has not identified any health benefits from switching to organic
food.
2) Sustainable crops (such as willow plantations that are cultivated for biomass fuel) can
match organic farms in terms of biodiversity.
3) Since organic farming is a relatively new phenomenon, it is difficult to gauge its long-term
benefits and ill-effects accurately.
4) With growing world populations, it is vital that a farmland produces as much food as
possible, but as organic farming is less efficient, more wilderness will be lost.

126
Chapter VA
CRITICAL REASONING 2 4.4

PRACTICE EXERCISE-2

Directions: The passages given below are followed by questions. Choose the best answer for each
question. (Past CAT Questions)

1. It is sometimes mooted that there can be democracy in a two-party system. That would be
correct if politics were a game like cricket or football; but politics is not a sport.
Which of the following would strengthen the argument?
1) A two-party system functions well.
2) Politics is a dirty game.
3) Two political parties limit the choice of the voters.
4) None of these.

2. It is sometimes mooted that there can be democracy in a two-party system. That would be
correct if politics were a game like cricket or football; but politics is not a sport.
Which of the following would weaken the argument?
1) The game of politics is played like any other game, for example, football.
2) Politics is not a sport.
3) Political parties struggle for power.
4) None of these

3. There can be no civilization without music, dance or art, for one is not fully, vibrantly alive
without them.
Which of the following would weaken the argument?
1) Music is the life of man.
2) Living persons like music.
3) Art has no relation with civilisation.
4) None of these

4. There can be no civilization without music, dance or art, for one is not fully, vibrantly alive
without them.
Which of the following would strengthen the argument?
1) Music, dance and art are human activities.
2) Only the vibrantly alive can contribute to civilisation.
3) Music injects new life in man.
4) None of these.

127
CATapult
VERBAL REASONING

5. We have planned development with a view to raising the standard of living of our teeming
millions. Hence, our economic development is inspired by social justice.
Which of the following will weaken the argument?
1) Without economic development, the standard of living cannot be raised.
2) Social justice implies economic prosperity.
3) Development cannot be planned.
4) Planning for development requires expert inputs.

6. We have planned development with a view to raising the standard of living of our teeming
millions. Hence, our economic development is inspired by social justice.
Which of the following will strengthen the argument?
1) Social justice can be done by raising the standard of living.
2) Economic planning is necessary for every state.
3) For economic development production should be increased.
4) None of these

7. We will have to take more interest in hydroelectric projects. As the prices of oil have in-
creased, it has become vital that such renewable sources of energy are tapped.
Which of the following will weaken the argument?
1) Generation of hydroelectric power is more costly than oil.
2) OPEC increased oil prices.
3) Without energy we cannot manage.
4) None of these.

8. Unless you devote your whole life to it, you will never learn to speak the language of an-
other country to perfection; you will never know its people and its literature with complete
intimacy.
Which of the following is likely to undermine the above argument?
1) I can speak ten foreign languages already.
2) I do not travel to foreign countries.
3) I am happy with the languages I know and do not need any other languages.
4) I should spend time to understand my own people and literature first, only then can I
appreciate other languages and cultures.

128
Chapter VA
CRITICAL REASONING 2 4.4

9. The writer can only be fertile if he renews himself and he can only renew himself, if his soul

PRACTICE EXERCISE CLASS EXERCISE THEORY


is constantly enriched by fresh experience.
Which of the following is most likely to support the above thought?
1) Only out of fresh experience can the writer get germs for new writing.
2) The writer can meet new people.
3) The writer must see new places.
4) None of these

10. But because the idea of private property has been permitted to override with its selfishness
the common good of humanity it does not follow that there are no limits within which that
idea can function for the general convenience and advantage.
Which of the following is most likely to weaken the argument?
1) All the people of the society should progress at an equitable rate and there should be no
disparities and private property does bring about a tremendous disparity.
2) One should not strive for the common good of humanity at all; instead one should be
concerned with maximising one’s own wealth.
3) One should learn from the experiences of former communist nations and should not repeat
their mistakes at all.
4) Even prosperous capitalist countries like the USA have their share of social problems.

11. Now the audience is a very curious animal. It is shrewd rather than intelligent. Its mental
capacity is less than that of its most intellectual members. If these were graded from A to
Z, decreasing with succeeding letters to the zero of the hysterical shop-girl, I should say its
mental capacity would come around about the letter O.
According to the above statements:
1) Some members in the audience are more intelligent than any of its other members.
2) The net intelligence of the audience is a little less than average.
3) Neither (1) nor (2)
4) Both (1) and (2)

12. I have been studying it, consciously and subconsciously, for forty years and I still find men
unaccountable; people I know intimately can surprise me by some action of which I never
thought them capable or by the discovery of some trait that exhibits a side of themselves
that I never even suspected.
The idea in this sentence can be best summarised as …
1) Men are inconsistent and therefore one should not be confident even about one’s closest
friends.
2) Men are unpredictable, one can never tell what they will do next; hence, one should be
very careful in one’s dealings.
3) No matter how closely you know somebody there still exists an unknown facet of his
personality.
4) None of these.
129
CATapult
VERBAL REASONING

13. In order to ease the traffic congestion, the transport planners decided to have a sophisticated
system of elevated monorail travel in the city. However, it was pointed out by somebody that
a metro rail system would be a more effective solution to the traffic problem. The plan was
thus stalled. Moreover, since a budget had not been drawn up for the project, it was deemed
fit to stall the work of the monorail for some time. In the meanwhile, the traffic planners of
the city decided to build an efficient system of subways and flyovers in the city with the aim
of easing the same problem. At the instant when the planners were preparing to award the
contracts to the concerned parties, the transport planners came up with the contention that
the subways interfered with the site of a pillar of the monorail system. The traffic planners
had to give up the idea and think of other possible solutions.
Which of the following can be inferred from the above passage?
1) The city authorities felt that the monorail system was essentially impractical.
2) There is ongoing communication between the two groups of planners in the city.
3) The projects would be stalled for an indefinite period.
4) All of the above

14. The company encourages its managers to interact regularly, without a pre-set agenda, to
discuss issues concerning the company and society. This idea has been borrowed from the
ancient Indian concept of religious congregation, called Satsang. Designations are forgotten
during these meetings; hence, it is not uncommon in these meetings to find a sales engineer
questioning the CEO on some corporate policy or on his knowledge of customers.
Based on the information provided in the above passage, it can be inferred that …
1) The company is concerned about its reputation among its employees.
2) The company believes in fostering the spirit of dialogue without letting it degenerate into
a positioning-based debate.
3) The company had some inter-personnel problems in the past due to which it felt the need
for these corporate Satsangs.
4) All of the above

130
Chapter VA
CRITICAL REASONING 2 4.4

15. From Cochin to Shimla, the new culture vultures are tearing down acres of India’s architectural

PRACTICE EXERCISE CLASS EXERCISE THEORY


treasures. Ancestral owners are often fobbed off with a few hundred rupees for an exqui-
sitely carved door or window, which fetches fifty times that much from foreign dealers, and
yet more from the drawing room sophisticates of Europe and the U.S. The reason for such
shameless rape of the Indian architectural wealth can perhaps, not wrongly, be attributed to
the unfortunate blend of activist disunity and local indifference.
It can be inferred from the above passage that …
1) The environment created by the meeting between activist disunity and local indifference
is ideal for antique dealers to thrive in India.
2) Only Indians are not proud of their cultural heritage and are hungry for the foreign cur-
rency that is easily available through the return of artifacts.
3) Most Indians families have heirlooms which can be sold at high prices to Europeans and
Americans.
4) India provides a rich market for unscrupulous antique dealers.

16. Deepa Mehta’s Fire is under fire from the country’s self-appointed moral police. Their contention
is that the film is a violation of Indian cultural mores and cannot be allowed to influence the
Indian psyche. According to them, such films ruin the moral fabric of the nation. This moral
fabric must be protected and defended against such intrusions at all cost, even at the cost
of cultural dictatorship.
Based on the information in the above passage, it can be inferred that …
1) The assumption underlying the moral police’s critique of Fire is that the Indian audience
is vulnerable to all types of influence.
2) The assumption underlying the moral police’s critique of Fire is that the Indian audience
is impressionable and must be protected against ‘immoral’ influences.
3) The moral police thinks it has the sole authority to pass judgement on films screened in
India.
4) None of the above

17. The dominant modern belief is that the soundest foundation of peace would be universal
prosperity. One may look in vain for historical evidence that the rich have regularly been more
peaceful than the poor, but then it can be argued that they have never felt secure against
the poor; that their aggressiveness stemmed from fear; and that the situation would be quite
different if everybody were rich.
It can be inferred from the above passage that …
1) A lot of aggression in the world stems from the desire of the haves to defend themselves
against the have-nots.
2) Universal prosperity as a fool-proof measure of peace can no longer be accepted.
3) Both (1) and (2)
4) Neither (1) nor (2)

131
CATapult
VERBAL REASONING

18. The effect produced on the mind by travelling depends entirely on the mind of the traveller
and on the way in which he conducts himself. The chief idea of one very common type of
traveller is to see as many objects of interest as he possibly can. If he can only after his
return home say that he has seen such and such a temple, castle, picture gallery, or mu-
seum, he is perfectly satisfied. Far different is the effect of travels upon those who leave
their country with their minds prepared by culture to feel intelligent admiration for all the
beauties of nature and art to be found in foreign lands. When they visit a new place, instead
of hurrying from temple to museum to picture gallery, they allow the spirit of the place to
sink in to their minds, and only visit such monuments as the time they have at their disposal
allows them to contemplate without haste.
It can be inferred from the above passage that …
1) The writer prefers the second type of traveller.
2) The first type of traveller is the lay traveller who does not understand the worth of any
place he travels to.
3) The objective of the second type of traveller is not to see much, but to see well.
4) All of the above.

19. Whether we look at the intrinsic value of our literature, or at the particular situation of this
country, we shall see the strongest reason to think that of all foreign tongues the English
tongue is that which would be the most useful to our native subjects.
It can be inferred that …
1) The speaker is a die-hard colonist.
2) The speaker has the good of the nation at heart.
3) The speaker is addressing an issue related to a colonial empire.
4) None of the above

20. Aspiration is nothing new. Neither is the debate over what the Indian middle class is, what it
wants and what it buys. Since the mid-80s, that has been the focus of the economic policy
papers’ so called pro- and anti-poor budgets and marketing strategies that have successfully
broken the barrier of urban selling and reached deeper into rural India with increasing income
levels and aspirations.
Based on the above passage it can be inferred that …
1) The Indian middle class has been the focus of economic policies for a long time.
2) The Indian middle class has graduated from being the ‘deprived’ middle class to the
‘pampered’ middle class.
3) Both (1) and (2)
4) Neither (1) nor (2)

132

You might also like